Anda di halaman 1dari 258

• 2 Item: 1 of 44 V/lark

. 3 Previous Next Lab Values. Notes Calculator.

• 4
• 5

• 6 A 6-year-old girl is brought to the physician due to pallor, decreased energy, bloody diarrhea, and "spots" over
. 7 her arms and legs that developed over the past 2 days_ Her immunizations are up to date. The patient's past
• 8 medical history is unremarkable and she has been generally healthy. The girl has no fever, vomiting, or joint
• 9 pain. She has had no travel or animal exposures. The patient's temperature is 37.1 C (98.8 F). On
• 10 examination, she appears lethargic and weak; her face and extremities are swollen and she has poor skin
• 11 turgor. The patient has scattered petechiae and a few purpura on her arms. She also has moderate, non-
• 12 localized abdominal tenderness_ Laboratory studies are as follows:
• 13
Hemoglobin 6.4 gJdL
• 14
Platelets 450001p L
• 15
Creatinine 2.3 mg/dL
• 16
Total bilirubin 3 mg/I:IL
• 17
Direct bilirubin 0.2 mg/I:IL
• 18
• 19
Urinalysis
20
Specific gravity 1.025
21
pH 5
• 22
Protein +2
• 23
Blood moderate
• 24
Leukocyte esterase negative
• 25
Nitrites negative
• 26
Bacteria none
• 27
White blood cells 50+1hpf
• 28
Red blood cells 20-301hpf
• 29
Casts hyaline casts
• 30
• 31
Which of the following is the most likely diagnosis?
• 32
• 33
. 34 • A. Hemolytic-uremic syndrome
• 35
• B. Henoch-SchOnlein purpura
• 36
• 37
• C. Immune thrombocytopenic purpura
• 39 D. Post-streptococcal glomerulonephritis
• 39
• E Rocky Mountain spotted fever
• 40
• 41
• 42 0
Feedback Suspend End Block
1
2 Item: 2 of 44 F' Mark
3 Previous Next Lab Values. Notes Calculator.
• 4-
5
A 2-week-old baby girl is brought to the clinic for the evaluation of vaginal discharge. Her mother has noticed a
6
cloudy white vaginal discharge mixed with blood since yesterday. She was born by normal vaginal delivery at
▪ 7
term, and no congenital anomalies were noted at birth. Physical examination reveals a blood-stained,
▪ 8
odorless vaginal discharge. Her vital signs are stable. What is the most appropriate next step in the
▪ 9
management of this patient?
• 10
• 11 • A. Treatment for Chlamydia
• 12
• B. Treatment for HSV-2
• 13
• 14 O C. Rule out vaginal cancer
• 15
• 16 • ID. Reassurance
• 17 • E Treatment for Trichomonas vaginalis
• 18
• 19 O Suspect child abuse
20
21
• 22
• 23
• 24
• 25
• 26
• 27
• 28
• 29
• 30
• 31
• 32
• 33
• 34
• 35
• 36
• 37
• 38
• 39
• 40
• 41
• 42 0
Feedback Suspend End Block
• 1
2 Item: 3 of 44 V/lark
Previous Next Lab Values. Notes Calculator.
• 4
• 5

• 6 A previously healthy 15-year-old boy is brought to the physician for progressive muscle weakness. Over the
. 7 past 2 months, he has had increasing difficulty making facial expressions, including smiling and frowning_ He
• 8 also has had difficulty swallowing but has no pain. The boy takes no medications and his vaccinations are up
• 9 to date. He was adopted as an infant and his biological family history is unknown_ Physical examination
• 10 shows ptosis, temporal wasting, thin cheeks, emaciated extremities, atrophy of the thenar and hypothenar
• 11 eminences, inverted V-shaped upper lip, and testicular atrophy. Delayed relaxation is noted on contraction of
• 12 the thenar and hypothenar muscles_ Testicular volume is small for age. Neurologic examination shows
• 13 normal deep-tendon reflexes and a negative Babinski sign. What is the most likely mode of genetic
• 14 transmission of the patient's condition?
• 15
• 16
• 17 • A. Autosomal dominant
• 18 • B. Autosomal recessive
• 19
• C. De novo mutation
20
21 • ID. X-linked dominant
• 22
• 23
• E X-linked recessive
• 24
• 25
• 26
• 27
• 28
• 29
• 30
• 31
• 32
• 33
. 34
• 35
• 36
• 37
• 38
• 39
• 40
• 41
• 42 0
Feedback Suspend End Block
• 1
2 Item: 4 of 44 F' Mark

• 3 Previous Next Lab Values. Notes Calculator.
4
. 5

• 6 A 6-year-old boy with congenital deafness is brought to the emergency department after fainting. He was
. 7 running with friends in his backyard when he suddenly dropped to the ground. He regained consciousness
• 8 quickly and without confusion. Review of systems is negative for shaking movements, tongue-biting, or
• 9 urinary incontinence. He has no history of similar episodes. The boy has met all developmental milestones
• 10 and takes no medications. His past medical history is significant for bilateral cochlear implantation at age 2
• 11 for sensorineural hearing loss. He had a brother who drowned suddenly at age 7_ Blood pressure is 110/70
• 12 mm Hg and heart rate is 713/min and regular. Physical examination is unremarkable. Serum chemistry is
• 13 normal. Electrocardiogram is obtained_ Which of the following therapies is most appropriate for this patient?
• 14
• 15
• 16 • A. Ethosuximide
• 17 • B. Levetiracetam
• 18
• 19
• C. Propranolol and pacemaker
20 • ID. Quinidine
21
• 22
• E Sotalol
• 23 • F. Verapamil
• 24
• 25
• 26
• 27
• 28
• 29
• 30
• 31
• 32
• 33
• 34
• 35
• 36
• 37
• 38
• 39
• 40
• 41
• 42 0
Feedback Suspend End Block
• 1
• 2 Item: 4 of 44 Mark 11 I
3 Previous Next Lab Values. Notes Calculator.
4 Media Exhibit
• 5
• 6
• 7
• 8



9
10
11
1i,,,,L4..,,,L
I :ck.r' irinn-lr'ilr'Hr-`\-
N-4

• 12
111.. •site—A,—
• 13 n aVL % tF
• 14
• 15
I t
• 16 1 i., •-•,_,I.---,"11_..--..%I
. ',__
• 17 M 'AT V6
• 18
• 19
20
21 1
• 22


23
24
1
• 25



26
27
28
1-Wr--N-11,
-1
• 29
• 30
31
• 32
• 33
• 34
• 35
• 36
• 37
• 38
• 39 ]
• 40
• 41
• 42
1
2 Item: 5 of 44 V/lark
3 Previous Next Lab Values. Notes Calculator.
4

A 3-year-old boy is brought to the physician for evaluation of delayed milestones_ Over the past year, he has
▪ 7 been receiving physical and speech therapies for delays in gross motor skills and speech_ He is currently
▪ 8 able to walk up and down stairs and run well. He has a vocabulary of approximately 25 words and does not
▪ 9 make 2-word sentences. His parents report that he has poor eye contact and does not show interest in
• 10 engaging with other children_ Review of systems is otherwise negative. His vaccinations are up to date. He
• 11 has an uncle with Down syndrome and a cousin with autism. Head circumference, length, and weight are
• 12 each at the 5th percentile_ Examination of the face shows small palpebral fissures, absent philtrum, and a
• 13 thin upper lip border. What is the underlying cause of his problems?
• 14
• 15
• 16
O A. Adverse effects from vaccination during infancy
• 17 O B. Fragile X mental retardation 1 gene mutation
• 18
O C. In utero exposure to alcohol
• 19
20 O D. In utero exposure to cytomegalovirus
21 • E In utero exposure to phenytoin
• 22
In utero exposure to rubella
• 23
• 24
• 25
0
• 26
• 27
• 28 G. Nondisjunction resulting in an extra copy of chromosome 21


29
30
a
• 31
• 32
• 33
• 34
• 35
• 36
• 37
• 38
• 39
• 40
• 41
• 42 411 0
Feedback Suspend End Block
2 Item: 6 of 44 F Mark
3 Previous Next Lab Values. Notes Calculator.
4

A 13-year-old boy is brought to the physician for evaluation of a limp. He has had mild left hip pain for several
weeks_ After the boy fell off his bicycle today, his mother noticed that he was limping and brought him to the
▪ 8 office. His body mass index is 31 kg/m2. He walks with a limp to the examination table, favoring his right leg.
▪ 9 As he sits down, his left leg is rotated externally. There is decreased range of motion and pain with internal
• 10 rotation of his left hip. There is no leg length discrepancy_ Bilateral anteroposterior and frog-leg lateral x-rays
• 11 of both hips are obtained. The frog-leg lateral view is shown below. What is the most likely diagnosis?
• 12
• 13
• 14
• 15
• 16
• 17
• 18
• 19
20
21
• 22
• 23
• 24
• 25
• 26
• 27
• 28
• 29
• 30
• 31
• 32
• 33
• 34
• 35
• 36
• 37
• 38
• 39
• 40
se
• 41
• 42 0
Feedback Suspend End Block
- 1
• 2 Item: 6 of 44 .11 1. Mark
• 3 Previous Next Lab Values. Notes Calculator.
- 4
5
6
7
8
9
• 10
• 11
• 12
• 13
• 14
• 15
• 16
• 17
• 18
• 19
20
21
• 22
• 23
• 24
• 25
• 26
• 27
• 28
• 29
• 30
• 31
• 32
• 33
. 34 • A. Avascular necrosis of the femoral head
• 35
• B. Chronic developmental hip dysplasia
• 36
• 37
• C. Femoral neck stress fracture
• 38 .0 ID. Legg-Calve-Perthes disease
• 39
• E Slipped capital femoral epiphysis
• 40
• 41
• 42 0
Feedback Suspend End Block
1
2 Item: 8 of 44 F' Mark
3 Previous Next Lab Values. Notes Calculator.
4-
5
6 A 9-year-old girl is seen in the office for unsteady gait and weakness in the lower limbs. On examination she is
7 found to have a wide-based gait with constant shifting of position to maintain her balance. There is decreased
8 vibratory and position sense in the lower extremities, and ankle jerks are absent bilaterally. Her feet are
9 deformed with high plantar arches_ MRI of the brain and spinal cord shows marked atrophy of the cervical
• 10 spinal cord and minimal cerebellar atrophy. Nerve conduction velocity results are within normal limits. EKG
• 11 shows T-wave inversions in the inferior and lateral chest leads. The mother says that her other child, a
• 12 3-year-old boy, is completely normal. He shares toys with his sister and plays well with other children at
• 13 playschool. He is able to copy squares and crosses but is unable to copy a triangle. Which of the following
• 14 statements would you advance while discussing the girl's condition with her mother?
• 15
• 16
• 17
A. There is nothing to worry about since it's a non-progressive condition
• 18
• 19 a
20
21
• 22
• 23 B. It is most likely a result of spontaneous mutation & the chances of your boy developing the same
• 24 condition are minimal
• 25
• 26
• 27
• 28 0
• 29
• 30
• 31
• 32
• 33
• 34
C. The EKG abnormalities are most likely due to myocardial ischemia
• 35
• 36
• 37 0
• 38
• 39
• 40
se
• 41
• 42 0
Feedback Suspend End Block
1
2 Item: 8 of 44 F' Mark
3 Previous Next Lab Values. Notes Calculator.
4-
5
6 A 9-year-old girl is seen in the office for unsteady gait and weakness in the lower limbs. On examination she is
7 found to have a wide-based gait with constant shifting of position to maintain her balance. There is decreased
8 vibratory and position sense in the lower extremities, and ankle jerks are absent bilaterally. Her feet are
9 deformed with high plantar arches_ MRI of the brain and spinal cord shows marked atrophy of the cervical
• 10 spinal cord and minimal cerebellar atrophy. Nerve conduction velocity results are within normal limits. EKG
• 11 shows T-wave inversions in the inferior and lateral chest leads. The mother says that her other child, a
• 12 3-year-old boy, is completely normal. He shares toys with his sister and plays well with other children at
• 13 playschool. He is able to copy squares and crosses but is unable to copy a triangle. Which of the following
• 14 statements would you advance while discussing the girl's condition with her mother?
• 15
• 16
• 17
• A. There is nothing to worry about since it's a non-progressive condition
• 18 • E. It is most likely a result of spontaneous mutation & the chances of your boy developing the same
• 19 condition are minimal
20
• C. The EKG abnormalities are most likely due to myocardial ischemia
21
• 22 • ID. The inability of your boy to copy a triangle worries me since it may be an early manifestation of the
• 23 same condition your daughter is suffering from
• 24 E. I advise you to go for prenatal counseling if you desire any future pregnancies
• 25
• 26
• 27
• 28 0
• 29
• 30
• 31
• 32
• 33
• 34
• 35
• 36
• 37
• 38
• 39
• 40
• 41
• 42 0
Feedback Suspend End Block
1
• 2 Item: 9 of 44 F' Mark
• 3 Previous Next Lab Values. Notes Calculator.
• 4-
• 5
6 A 5-year-old boy is brought to the emergency department with blurred vision, right eye pain, and headache_
7 His past medical history includes otitis media, dental caries, eczema, and allergic rhinitis. He is allergic to
insect venom. His temperature is 39.4' C (103° F), blood pressure is 100/70 mm Hg, pulse is 120/min, and
respirations are 20imin. Examination shows conjunctival injection, pain with extraocular movements,
• 10 periorbital edema, limited adduction, and proptosis of the right eye. The left eye is normal. Which of the
• 11 following is the most common predisposing factor for this child's condition?
• 12
• 13
• A. Bacterial sinusitis
• 14
• 15 ▪ B. Dacryocystitis
• 16
0. C. Dental abscess
• 17
• 18 • ID. Hordeolum
• 19
C. E. Impetigo
20
21 • Insect bite
• 22
• G. Orbital trauma
• 23
• 24 • H. Otitis media
• 25 • I. Strabismus surgery
• 26
• 27
• 28
• 29
• 30
• 31
• 32
• 33
• 34
• 35
• 36
• 37
• 38
• 39
• 40
• 41
• 42 0
Feedback Suspend End Block
1
2 Item: 10 of 44 F' Mark
3 Previous Next Lab Values. Notes Calculator.
4-
5
6 A 2-year-old child is brought to the office due to vomiting. He is recovering from an upper respiratory
7 infection, which was treated by his mother with over-the-counter aspirin. an examination he is irritable,
8 lethargic, agitated and uncooperative. His height, weight and head circumference are at the 50th percentile.
9 The pupils are equal and have a sluggish reaction to light The sclera is non-icteric. The neck is supple and
without adenopathy. The abdomen is flat, with normal bowel sounds. The lab findings are as follows:
• 11
Serum bilirubin Normal
• 12
Serum AST and ALT Increased
• 13
Serum ammonia Increased
• 14
Prothrombin time Prolonged
• 15
• 16
• 17
A CT scan of the brain shows cerebral edema. CSF analysis is normal, except for increased pressure. What
• 18
is the most likely diagnosis?
• 19
20 • A. Aseptic meningitis
21
• B. Fteye syndrome
• 22
• 23 • C. Carnitine deficiency
• 24
• D. Sepsis
• 25
• 26 • E Viral encephalitis
• 27
• 28
• 29
• 30
• 31
• 32
• 33
• 34
• 35
• 36
• 37
• 38
• 39
• 40
• 41
• 42 0
Feedback Suspend End Block
• 1
• 2 Item: 11 of 44 F' Mark
▪ 3 Previous Next Lab Values. Notes Calculator.
• 4
▪ 5
▪ 6 A 6-hour-old boy is in the newborn nursery with respiratory distress_ He was born at 39 weeks gestation by
▪ 7 uncomplicated vaginal delivery_ The pregnancy was complicated by pre-eclampsia during the 3rd trimester_
▪ 8 The mother had an elevated 1-hour glucose tolerance screening, but her 3-hour glucose tolerance test was
▪ 9 normal. The infant weighs 1.9 kg (4 lb 4 oz) (<3rd percentile). He has not been able to feed due to
• 10 tachypnea_ Temperature is 372 C (99 F), blood pressure is 6013L1- mm Hg, pulse is 144/min, and respirations
are 68/min. Pulse oximetry shows 96% saturation on room air_ an examination, the infant's skin is plethoric
• 12 and he has mild cyanosis of his lips and tongue. No cardiac murmur is heard. He has a normal Moro reflex
• 13 and normal tone. Examination is otherwise normal. Chest radiograph shows clear lung fields with a normal
• 14 cardiac silhouette. Initial laboratory results are as follows:
• 15
Complete blood count
Hemoglobin 22.8 WI:IL
• 17
Hematocrit 69%
• 18
Platelets 200,004 L
• 19
Leukocytes 5,500/pL
20
Neutrophils 56%
21
Lymphocytes 33%
• 22
Glucose 50 mg/dL
• 23
• 24
• 25
Which of the following is the most likely cause of this patient's respiratory distress?
• 26
• 27 • A. Cyanotic heart disease
• 28
• B. Dehydration
• 29
• 30 • C. Hyperinsulinism
• 31 • D. Polycythemia
• 32
• E Transient tachypnea of the newborn
• 33
• 34
• 35
• 36
• 37
• 39
• 39
• 40
• 41
• 42 0
Feedback Suspend End Block
• 1
• 2 Item: 12 of 44 I'Mark
• 3 Previous Next . Lab Values. Notes Calculator.
• 4
• 5
• 6 A 7-year-old boy is brought to the physician because of fever, malaise, and pain in his right lower leg for the
• 7 last three days. His pain is 7110 in severity, non-radiating, and unrelieved by ibuprofen, heat, or rest He has
• 8 no history of trauma or fracture. There is no family history of sickle cell anemia. His temperature is 39.3'C
• 9 (103"F). Examination of his right leg shows bony point tenderness, erythema, and warmth over a portion of
• 10 the proximal tibia. Blood is drawn for culture and sensitivity studies_ Which of the following organisms is
- 11 most likely to be isolated from his blood?

- 13
- 14 • A. Staphylococcus epidermidis
15
6
O B. Staphylococcus aureus

• 17 O C. Salmonella species
• 18
O D. Klebsiella species
• 19
20 • E Pseudomonas aeruginosa
21 • Group B streptococcus
• 22
• 23 O G. Escherichia coli
• 24
O H. Streptococcus pyogenes
• 25
• 26
• 27
• 28
• 29
• 30
• 31
• 32
• 33
• 34
• 35
• 36
• 37
• 38
• 39
• 40
• 41
• 42 0
Feedback Suspend End Block
• 1
• 2 Item: 13 of 44 F Mark
▪ 3 Previous Next Lab Values. Notes Calculator.
• 4
▪ 5
▪ 6 7 A 16-year-old girl comes to the emergency department with painless, heavy vaginal bleeding for 3 days. Her
▪ 7 menses have been irregular since menarche at age 14, and skip up to 4 months at a time_ Her last menstrual
▪ 8 period was almost 4 months ago. The patient has a steady boyfriend and uses condoms for contraception.
▪ 9 She has no other bleeding problems. Her temperature is 37 C (99 F), blood pressure is 111116CI mm Hg, pulse
• 10 is 90imin, and respirations are 161min_ On speculum examination, she has a moderate amount of blood in
• 11 the vaginal vault and active bleeding through the cervical os_ Bimanual examination shows a uterus of normal
• 12 size and shape with no adnexal masses_ Pregnancy test is negative. Coagulation studies are within normal
13 limits_ Her hemoglobin is 9.8 gidL. Which of the following is the most appropriate next step in management of
• 14 this patient?
• 15
• 115'1
• 17 • A. Emergency dilation and curettage
• 18 • E High-dose estrogen therapy
• 19
• C. High-dose gonadotropin-releasing hormone agonists
20
21 • ID. Packed red blood cell transfusion
• 22
• 23
• E Sonohysterogram
• 24
• 25
• 26
• 27
• 28
• 29
• 30
• 31
• 32
• 33
• 34
• 35
• 36
• 37
• 39
• 39
• 40
• 41
• 42 0
Feedback Suspend End Block
• 1
• 2 Item: 14 of 44 V- Mark
▪ 3 Previous Next Lab Values. Notes Calculator.
• 4-
▪ 5
▪ 6 A 2-hour-old boy has developed worsening cyanosis over the past few minutes. The infant was delivered
▪ 7 vaginally at term after an uncomplicated pregnancy. Birth weight was 4.1 kg (9 lb)_ Apgar scores were 7 and
▪ 8 8 at 1 and 5 minutes, respectively. Oxygen is administered by hood but does not relieve the cyanosis.
▪ 9 Further examination shows tachypnea, subcostal retractions, a normal first heart sound, a single and loud
• 10 second heart sound, and no murmur_ Which of the following is the most likely cause of the infant's cyanosis?
• 11
• 12
• 13
• A. Atrial septal defect
• 14 • 13. Coarctation of the aorta
• 15
• 16 'I
0 C. IDextrocardia
• 17 O ID. Hypoplastic left heart syndrome
• 18
• 19 O E Patent ductus arteriosus
20 • F. Tetralogy of Fallot
21
• 22
• G. Transposition of the great vessels
• 23 • H. Ventricular septal defect
• 24
• 25
• 26
• 27
• 28
• 29
• 30
• 31
• 32
• 33
• 34
• 35
• 36
• 37
• 39
• 39
• 40
• 41
• 42 0
Feedback Suspend End Block
• 1
• 2 Item: 15 of 44 V-• Mark
▪ 3 Previous Next Lab Values. Notes Calculator.
• 4
5
▪ 6 A 3-week-old boy in the neonatal intensive care unit has increased gastric residual volume and vomiting_ He
▪ 7 was born at 28 weeks gestation by vaginal delivery due to cervical incompetence_ Birth weight was 900 g (2
▪ 8 Ib). He initially required mechanical ventilation but extubated to nasal cannula oxygen last week. He has been
▪ 9 advancing on continuous nasogastric formula feeds and gaining an average of 30 g each day. His
• 10 temperature is 35.6 C (96 F), blood pressure is 85/45 mm Hg, pulse is 14 /min, and respirations are 34/min.
• 11 Pulse oximetry is 97% on 2 L oxygen. Examination shows a lethargic neonate with abdominal distension.
• 12 Laboratory studies show leukocytosis and metabolic acidosis. Imaging would most likely show which of the
• 13 following?
• 14
I 15
• 16
• A. Abdominal x-ray with air in the bowel wall and portal veins
• 17 • B. Abdominal x-ray with air in the stomach and duodenum but no distal air
• 18
• C. Abdominal ultrasound with thickened pylorus muscle
• 19
20 • 11 Contrast enema with narrow rectosigmoid and dilation of the rest of the colon
21
E Head ultrasound with hemorrhage into the lateral ventricles
• 22
• 23
• 24
0
• 25
• 26
• 27
• 28
• 29
• 30
• 31
• 32
• 33
• 34
• 35
• 36
• 37
• 38
• 39
• 40
• 41
• 42 0
Feedback Suspend End Block
• 1
• 2 Item: 16 of 44 V- Mark
▪ 3 Previous Next Lab Values. Notes Calculator.
• 4-
▪ 5
▪ 6 A 2-year-old boy is brought to the emergency department after developing coughing and shortness of breath a
▪ 7 few minutes after eating peanuts. He has a history of eczema treated with topical hydrocortisone_ His father
▪ 8 has a history of asthma. His temperature is 37.2° C (99° F), blood pressure is 100/70 mm Hg, pulse is
▪ 9 128/min, and respirations are 30imin_ Pulse oximetry is 91% on room air_ Diffuse wheezing and retractions
• 10 are present His voice is hoarse and he complains of itchiness_ Skin examination is shown in the photograph
• 11 below.
• 12
• 13
• 14
• 15
I 16
• 17
• 18
• 19
20
21
• 22
• 23
• 24
• 25
• 26
• 27
• 28
• 29
• 30
• 31
• 32
• 33
• 34
• 35
• 36
• 37
• 39
• 39
• 40
v
• 41
• 42 0
Feedback Suspend End Block
• 1
• 2 Item: 16 of 44 .11M Mark
▪ 3 Previous Next Lab Values. Notes Calculator.
• 4- A
▪ 5
▪ 6
▪ 7
▪ 8
▪ 9
• 10
• 11
• 12
• 13
• 14
• 15
I 16
• 17
• 18
• 19
20
21
• 22
• 23
• 24
• 25
• 26
• 27
• 28
• 29 ..lorhich of the following is the best initial treatment for this patient?
• 30
• 31
0 A. Endotracheal intubation
• 32
• 33
O B. Intramuscular epinephrine
• 34 O C. Intravenous methylprednisolone
• 35
O D. Inhaled albuterol
• 36
• E. Oral diphenhydramine
• 37
• 39 • Rigid bronchoscopy
• 39
O G. Topical hydrocortisone
• 40
• 41
• 42 0
Feedback Suspend End Block
• 1
• 2 Item: 17 of 44 VI/lark
▪ 3 Previous Next Lab Values. Notes Calculator.
• 4-
▪ 5
▪ 6 A 6-year-old boy is brought to the pediatrician with leg pain_ Over the last few months, he has complained of
▪ 7 pain in both legs that occurs only at night The pain is worst in his thighs although he occasionally has pain in
▪ 8 his calves as well. The episodes last a few hours each and improve with massage and over-the-counter
▪ 9 medications. The pain has remained unchanged over the past few months. The patient is able to walk, run,
• 10 and play at school without any complaints, and his mother has noticed no change in his activity level. He has
• 11 no fever, chills, or weight loss and there is no history of recent trauma. On examination, the boy has full range
• 12 of motion of the hips and knees bilaterally. No swelling or tenderness to palpation is noted and he has a
• 13 normal gait His mother is very worried because his 9-year-old cousin died of leukemia 6 months earlier.
• 14 Which of the following is the most appropriate next step in management of this patient?
• 15
• 16
• A. Blood cultures
• 18
• B. Bone scan
• 19
20 • C. Complete blood count
21
• D. Observation and reassurance
• 22
• 23 E Plain radiographs
• 24
• Psychiatric evaluation
• 25
• 26
• 27
• 28
• 29
• 30
• 31
• 32
• 33
• 34
• 35
• 36
• 37
• 39
• 39
• 40
• 41
• 42 0
Feedback Suspend End Block
• 1
• 2 Item: 18 of 44 F' Mark .4111°.
▪ 3 Previous Next Lab Values. Notes Calculator.
• 4
▪ 5
▪ 6 A 16-year-old girl is brought to the emergency department by her 18-year-old boyfriend due to nausea,
▪ 7 vomiting, and severe pain in her lower right quadrant She also complains of some anorexia_ The patient has
▪ 8 had these symptoms for almost 12 hours. Her last menstrual period was 2 weeks ago, and she has not been
▪ 9 sexually active in the past 2 weeks. Her temperature is 38.1' C (100.6" F), blood pressure is 110/713 mm Hg,
• 10 pulse is 88/min, and respirations are 12/min. Examination shows diffuse discomfort in her abdomen that is
• 11 increased in the right lower quadrant without guarding or rebound tenderness_ Pregnancy test, workup for
• 12 sexually transmitted diseases, and pelvic examination are negative. Abdominal and pelvic computed
• 13 tomography scans show nonspecific inflammation in the area of the appendix but are not definitive for
• 14 appendicitis. The patient's condition remains stable with improved pain control. However, it is recommended
• 15 that she be admitted to the hospital for further observation, with consideration for possible exploratory
• 16 laparotomy in the next 24 hours if she worsens_ This is discussed with the patient and her boyfriend. The
• 17 physician inquires about obtaining consent for further medical care. The patient says that she lives with her
parents but does not want them called as "they would find out I've been spending time with my boyfriend, and
19 they don't approve of him." Who is the appropriate person to sign the consent form for admission and further
20 medical care?
21
• 22
• 23
• A. Parent of the adult boyfriend
• 24 • B. Patient
• 25
• C. Patient's adult boyfriend
• 26
• 27 • 11 Patient's parent
• 28
• E Physician in the emergency department
• 29
• 30
• 31
• 32
• 33
• 34
• 35
• 36
• 37
• 38
• 39
• 40
• 41
• 42 0
Feedback Suspend End Block
1
2 Item: 19 of 44 V-• Mark
3 Previous Next Lab Values. Notes Calculator,
4-
5
6 A mother brings her 3-year-old son to the pediatrician because he has had 7 days of fever and a painful
7 swollen lymph node in his right groin. This is the boy's sixth episode of lymph node swelling; previous
8 episodes resolved after drainage and prolonged antibiotic therapy. The child also has a past medical history
9 significant for pneumonia at 1 months of age that required chest tube placement for drainage. His maternal
10 uncle died in childhood from recurrent infections_ On examination, the boy is at the 5th percentile for both
11 height and weight His temperature is 385°C (1013°F) There is an enlarged, tender lymph node in the right
12 inguinal area with erythema and warmth of the overlying skin. There are several healed incisions in the
13 inguinal regions bilaterally, as well as around the neck from old drainage sites. Laboratory analyses reveal the
14 following:
15
Hematocrit 35%
16
Platelet count 350,000imm3
17
18
Leukocyte count 17000/mm3
Segmented neutrophils 65%
Bands 10%
Lymphocytes 25%
• 22
• 23 Gram stain of fluid aspirated from the affected lymph node reveals numerous bacteria-filled segmented
• 24 neutrophils. Cultures grow S. aureus. What is the most likely mechanism underlying these findings?
• 25
• 26
O A. Adenosine deaminase deficiency
• 27
• 28
O B. Complement consumption
• 29 O C. Defective opsonization
• 30
O ID. Destruction of CD4+ lymphocytes
• 31
• 32
• E Developmental arrest in B lymphocyte maturation
• 33 • E Dysmorphogenesis of the 3rd and 4th pharyngeal pouches
• 34
O G. Impaired oxidative metabolism within phagocytes
• 35
• 36
• 37
• 38
• 39
• 40


41
42
Feedback.
IE
Suspend
a
End Block
• 1
• 2 Item: 20 of 44 r Mark
▪ 3 Previous Next Lab Values. Notes Calculator.
• 4-
▪ 5
▪ 6 The following vignette applies to the next 2 items.
▪ 7
▪ 8 A 4-day-old full-term boy is brought to the physician for his first office visit after an uncomplicated vaginal
▪ 9 delivery and newborn nursery course. The mother's blood type is A positive, and prenatal laboratory results
• 10 and ultrasound were normal_ Birth weight was 3A kg (7 lb 8 oz) and length was 48.5 cm (18 in). He has been
• 11 exclusively breastfed since birth and nurses for 10 minutes on each breast every 4 hours. The infant passed
• 12 several dark-brown, sticky meconium stools during the first 2 days of life but has had only smears of dark
• 13 yellow stool today. He has 3 wet diapers each day_ For the past day, his diapers have appeared as shown
• 14 below.
• 15
• 16
• 17
• 18
• 19

• 22
• 23
• 24
• 25
• 26
• 27
• 28
• 29
• 30
• 31
• 32
• 33
• 34
• 35
• 36
• 37
• 38
• 39 The neonate's current weight is 3.15 kg (6 lb 15 oz). He has scleral icterus and jaundice of the face, chest,
• 40 Ann Ahrinmpn Thp rpqi- of hiq nhuqirql pr2min2tinn is nnrrnql I 2hnr-21-nr-u rpci iltc 2r-p ac fnlInum-
• 41
• 42 11 . 0
Feedback Suspend End Block
• 1
• 2 Item: 20 of 44 F' Mark
▪ 3 Previous Next Lab Values. Notes Calculator.
• 4-
▪ 5
▪ 6
▪ 7
▪ 8
▪ 9
• 10
• 11
• 12
• 13
• 14
• 15
• 16
• 17
• 18
• 19

• 22
• 23 The neonate's current weight is 3.15 kg (6 lb 15 oz). He has scleral icterus and jaundice of the face, chest,
• 24 and abdomen. The rest of his physical examination is normal. Laboratory results are as follows:
• 25
Total bilirubin 15 mg/I:IL
• 26 Direct bilirubin 0.9 mgldL
• 27
• 28
Item 1 of 2
• 29
• 30 Which of the following is the most likely cause of this infant's hyperbilirubinemia?
• 31
• 32
• 33
• A. Alloimmune hemolytic disease
• 34 • B. Biliary atresia
• 35
• C. Breast milk jaundice
• 36
• 37 • D. Breastfeeding failure jaundice
• 38 .0 E. Dubin-Johnson syndrome
• 39 • Galactosemia
• 40
• 41
• 42 0
Feedback Suspend End Block
• 1
• 2 Item: 21 of 44 V-Mark
▪ 3 Previous Next Lab Values. Notes Calculator.
• 4-
▪ 5
▪ 6 Item 2 of 2
▪ 7
▪ 8 Which of the following is the best next step in management of this patient?
▪ 9
• 10
• 11 • A. Administer Rho(D) immune globulin to the infant's mother
• 12
• B. Breastfeed 15 minutes per side every 2-3 hours
• 13
• 14 • C. Continue current feeding regimen, follow-up in 1 week
• 15
• D. Phototherapy and exchange transfusion
• 16 •
- 17 • E. Switch to a cow's milk-based formula
- 18
• F. Switch to soy formula
- 19

• 22
• 23
• 24
• 25
• 26
• 27
• 28
• 29
• 30
• 31
• 32
• 33
• 34
• 35
• 36
• 37
• 38
• 39
• 40
• 41
• 42 0
Feedback Suspend End Block
1
2 Item: 22 of 44 V-• Mark
3 Previous Next Lab Values. Notes Calculator,
4-
5
6 A 5-year-old boy is brought to the physician for evaluation of left hip pain after his parents saw him limping this
7 morning. He spent time on the playground yesterday but his parents do not think he was injured then_ The
8 boy complains that the pain in his hip worsens when he moves or walks. For the past 3 days, he has had a
9 runny nose and congestion. Review of systems is otherwise negative. His grandmother has rheumatoid
10 arthritis treated with methotrexate. His temperature is 37_2' C (99° F), blood pressure is 100/65 mm Hg,
11 pulse is 92/min, and respirations are 18/min. Physical examination shows a well-appearing child with clear
12 rhinorrhea and intermittent dry cough. Lungs are clear to auscultation bilaterally_ His left hip is slightly
13 abducted and externally rotated with mildly decreased range of motion. He is able to stand and bear
14 weight The remainder of his examination is normal_ X-rays of both hips are normal. His laboratory results
15 are as follows:
16
Complete blood count
17
Hemoglobin 12.5 g/dL
18
Platelets 281,000/mm3
19
Leukocyte count 8,500/mm3
Neutrophils 30%
Eosinophils 1%
Lymphocytes 64%
• 23
Monocytes 5%
6 24
Erythrocyte sedimentation rate 30 mm/h
6 25
C-reactive protein 9 mg/L (N: 8 mg/L)
• 26
• 27
• 28
Which of the following is the best next step in management of this patient?
• 29
• 30 A. Ibuprofen, rest, and follow-up in 1 week
• 31
▪ B. Intravenous antibiotics
• 32
• 33 ▪ C. Magnetic resonance imaging of the left hip
• 34 ▪ D. Serum antinuclear antibodies
• 35
E Synovial fluid aspiration of the left hip
• 36
• 37
• 38
• 39
• 40


41
42 11
11
Feedback.
IE
Suspend
a
End Block
• 1
• 2 Item: 23 of 44 F' Mark
▪ 3 Previous Next Lab Values. Notes Calculator.
• 4-
▪ 5
▪ 6 A 7-year-old boy is brought to the clinic due to malaise and headache for the past 2 weeks. Three weeks ago,
▪ 7 he went on a hiking trip with his family during which his mother found a tick attached to his thigh. The mother
▪ 8 removed the tick with tweezers, and a red "bump' remained at the site where the tick was attached. One
▪ 9 week later, a red ring developed around the "bump' and has since grown larger. The rash is slightly itchy but
• 10 not painful_ The patient has a history of eczema treated intermittently with topical hydrocortisone. Last year,
• 11 he had tines pedis that resolved with terbinafine. Examination shows a nontender rash, approximately 8 cm
• 12 in diameter, on his right thigh. The remainder of his examination is normal. What is the most appropriate next
• 13 step in management of this patient?
• 14
• 15
• 16 • A. Intravenous ceftriaxone
• 17 • B. Oral amoxicillin
• 18
• 19 • C. Oral doxycycline
20 • D. Oral itraconazole
21
• 22 • E. Topical hydrocortisone
23
• 24
• 25
• 26
• 27
• 28
• 29
• 30
• 31
• 32
• 33
• 34
• 35
• 36
• 37
• 39
• 39
• 40
• 41
• 42 0
Feedback Suspend End Block
• -1
• 2 Item: 23 of 44 V-Mark 111
.
▪ 3 Previous Next Lab Values Notes Calculator
• 4- Media Exhibit
▪ 5
▪ 6 Media 1 of 1
▪ 7
I
▪ 8
▪ 9
• 10
• -1 -1
• 12
• 13
• 14
• 15
• 16
• 17
• 19
• 19
20
21

• 24
• 25
• 26
• 27
• 28
• 29
• 30
• 31
• 32
• 33
• 34
• 35
• 36
• 37
• 39
• 39
• 40
• 41
• 42 14. 0
Feedback Suspend End Block
• 1
• 2 Item: 24 of 44 V- Mark
▪ 3 Previous Next Lab Values. Notes Calculator.
• 4-
5
▪ 6 A 34-year-old woman who describes herself as a "holistic healer" brings her two children aged four years and
▪ 7 three years to the pediatrician for routine physical examinations_ This is a first visit as they have recently
▪ 8 moved from another state. The medical records for the children indicate that no immunizations have been
▪ 9 given_ When the mother is questioned about this, she proudly replies, "My children are wonderfully healthy on
• 10 their own and have no need for these artificial vaccines." The principles and benefits of immunization are
• 11 discussed at length, as well as the risks inherent in not being immunized. The physician recommends that the
• 12 children be given all age-appropriate vaccinations today. The mother remains convinced that immunizations
• 13 cause more harm than good, and she steadfastly refuses to allow her children to be vaccinated. What is the
• 14 most appropriate next step?
• 15
• 16
• 17 • A. Obtain a court order for immunization of children
• 18 • B. Proceed with immunizations today
• 19
• C. Inform mother that she will be reported to the local health department
20
21 • ID. Document in the medical chart that the risks and benefits of vaccination have been explained
• 22
E Request to speak with her husband
• 23
• 24 0
• 25
• 26
• 27
• 28
• 29
• 30
• 31
• 32
33
34
• 35
• 36
• 37
• 38
• 39
• 40
• 41
• 42 0
Feedback Suspend End Block
• 1
• 2 Item: 25 of 44 F' Mark
▪ 3 Previous Next Lab Values. Notes Calculator.
• 4-
▪ 5
▪ 6 An 18-year-old woman comes to the pediatrician with her 3-day-old daughter for her first newborn visit and
▪ 7 has questions about breastfeeding. She has been breastfeeding exclusively, but says that it is painful and
▪ 8 believes that she is not producing enough milk. She would like to stop breastfeeding and switch to formula.
▪ 9 Which of the following statements regarding breastfeeding is correct?
• 10
• 11
• 12 • A. Breastfeeding is associated with increased risk of obesity in the infant
• 13 • B. Breastfeeding reduces the risk of endometrial cancer in the mother
• 14
• 15 • C. Breastfeeding reduces the risk of otitis media in the infant
• 16
• ID. Nutritional supplementation is not needed with exclusive breastfeeding
• 17
• 18 • E There is no need for contraception with exclusive breastfeeding
• 19
20
21
• 22
• 23
• 24

• 26
• 27
• 28
• 29
• 30
• 31
• 32
• 33
• 34
• 35
• 36
• 37
• 39
• 39
• 40
• 41
• 42 0
Feedback Suspend End Block
• 1
• 2 Item: 26 of 44 F' Mark
▪ 3 Previous Next Lab Values. Notes Calculator.
• 4-
▪ 5
▪ 6 A 12-year-old boy is brought to the physician because of right groin pain, knee pain, and limping. He has had
▪ 7 these symptoms for the past 2 weeks_ He is at the 90th percentile for weight and €0th percentile for height
▪ 8 He is afebrile, and his other vital signs are within normal limits. Examination shows that the range of motion of
▪ 9 the right knee joint is within normal limits but hip movements are restricted and the right foot points outward.
• 10 There is external rotation of the right thigh on flexion of the hip. After confirming the diagnosis, which of the
• 11 following is the most appropriate management?
• 12
• 13
• 14
• A. Aspiration and microscopic examination of the hip joint synovial fluid
• 15 • B. Closed reduction of the hip joint
• 16
• 17 • C. Conservative management with rest and analgesics
• 18 11 Immediate osteotomy of the femoral neck
• 19
• E Surgical pinning of the femoral head
20
21
• 22
• 23
• 24
• 25

• 27
• 28
• 29
• 30
• 31
• 32
• 33
• 34
• 35
• 36
• 37
• 33
• 39
• 40
• 41
• 42 0
Feedback Suspend End Block
1
• 2 Item: 27 of 44 F' Mark .4111°.
3 Previous Next Lab Values. Notes Calculator.
• 4-
5
6 A 4-month-old boy is brought to the physician for routine evaluation. He was born at 36 weeks gestation by
▪ 7 cesarean section due to shoulder dystocia. His birth weight was 4 kg (8 lb 13 oz). He spent 3 weeks in the
▪ 8 neonatal intensive care unit for management of hypoglycemia and poor feeding secondary to macroglossia.
▪ 9 The patient has had no significant issues since discharge home. He is currently at the 99th percentile for
• 10 weight, length, and head circumference. Physical examination shows an enlarged tongue and a reducible
• 11 umbilical hernia. His right upper and lower extremities are significantly larger in circumference than the left
• 12 extremities. The remainder of his examination is normal. In addition to routine vaccinations, which of the
• 13 following is the best next step in management of this patient?
• 14
• 15
• 16 • A. Abdominal ultrasound
• 17 • B. Brain magnetic resonance imaging
• 18
• 19
• C. Referral for herniorrhaphy
20 • D. Serum glucose level
21
• 22
• E Thyroid-stimulating hormone level
• 23 • F. Urine homovanillic and vanilmandelic acid
• 24
• 25
• 26

• 28
• 29
• 30
• 31
• 32
• 33
• 34
• 35
• 36
• 37
• 38
• 39
• 40
• 41
• 42 0
Feedback Suspend End Block
• 1
• 2 Item: 28 of 44 V- Mark
▪ 3 Previous Next Lab Values. Notes Calculator.
• 4-
▪ 5
▪ 6 A 6-week-old boy is brought to the physician by his parents for evaluation of inconsolable crying. The boy has
▪ 7 cried "almost nonstop" for up to 4 hours every evening over the past 3 weeks_ The parents say that pacifiers
▪ 8 do not calm him, and they feel overwhelmed and frustrated. Otherwise, he is happy and alert during the rest
▪ 9 of the day. The infant was born full term by uncomplicated spontaneous vaginal delivery and is growing well.
• 10 He breastfeeds every 2 hours and has 2-3 soft stools daily. The child's physical examination is
• 11 unremarkable. The parents are concerned that "something is wrong" with their child. Which of the following
• 12 is the most appropriate response?
• 13
• 14
• 15
• A. "A lactose-free formula will help reduce the pain."
• 16 • B. "I will order a skeletal survey to evaluate for trauma_"
• 17
• 18
• C. will order an abdominal ultrasound to rule out intussusception_"
• 19 • D. "I will prescribe ranitidine for gastroesophageal reflux."
20
21
• E "Let's review some techniques for soothing your baby.'
• 22
• 23
• 24
• 25
- 25
- 27

30
31
32
33
34
• 35
• 36
• 37
• 39
• 39
• 40
• 41
• 42 0
Feedback Suspend End Block
• 1
• 2 Item: 29 of 44 F' Mark
▪ 3 Previous Next Lab Values. Notes Calculator.
• 4-
▪ 5
▪ 6 A 17-year-old boy is brought to the emergency department by his father after the boy began threatening him at
▪ 7 home. Over the last several months, the father reports, the boy has been increasingly abusive_ He was
▪ 8 recently involved in a fist fight at school. The boy states that there is nothing wrong. He is otherwise healthy.
▪ 9 He denies alcohol use, but does admit to occasional marijuana use. On examination he has acne on his
• 10 forehead and back and his hairline is receding. There is palpable tissue underneath his nipples bilaterally.
• 11 Heart and lung exams are normal_ What substance is this boy most likely abusing?
• 12
• 13
• 14
• A. Anabolic steroids
• 15 • B. Cocaine
• 16
• 17
C. Heroin
• 18 • 11 Methamphetamine
• 19
• E Phencyclidine
20
21 • F. Spray paint
• 22
• 23
• 24
• 25
• 26
• 27
• 28

31
32
33
34
• 35
• 36
• 37
• 38
• 39
• 40
• 41
• 42 0
Feedback Suspend End Block
• 1
• 2 Item: 30 of 44 F' Mark
▪ 3 Previous Next Lab Values. Notes Calculator.
• 4-
▪ 5
▪ 6 A 5-month-old full-term boy is brought to the physician for fever. He was doing well until this morning, when
▪ 7 he felt warm to his mother. He has had a "runny nose" and intermittent coughing but otherwise has been
▪ 8 breastfeeding well. His 3-year-old brother had an upper respiratory tract infection last week. The infant takes
▪ 9 vitamin ID, and his vaccinations are up to date_ His temperature is 38. 9 C (102 F), blood pressure is 90160
• 10 mm Hg, pulse is 120/min, and respirations are 32/min. Pulse oximetry shows an oxygen saturation of 96%.
• 11 Examination shows crusted rhinorrhea at the nares. The patient's chest radiograph is shown below.
• 12
• 13
• 14
• 15
• 16
• 17
• 18
• 19
20
21
• 22
• 23
• 24
• 25
• 26
- 27
- 28
▪ 29

• 31
• 32
• 33
• 34
• 35
• 36
• 37
Which of the following is marked on this patient's chest radiograph?
• 38
• 39
• 40 0 A_ Hilar lvmohadenooathv se
• 41
• 42 0
Feedback Suspend End Block
• 1
• 2 Item: 30 of 44 .11M Mark
-<1

▪ 3 Previous Next Lab Values. Notes Calculator.


• 4 Examination shows crusted rhinorrhea at the nares. The patient's chest radiograph is shown below.
▪ 5
▪ 6
▪ 7
▪ 8
▪ 9
• 10
• 11
• 12
• 13
• 14
• 15
• 16
• 17
• 18
• 19
20

• 22
• 23
• 24
• 25
• 26
- 27
- 28
• 29

Which of the following is marked on this patient's chest radiograph?


• 31
• 32
• 33 • A. Hilar lymphadenopathy
• 34 • B. Right atrium
• 35
• C. Right middle-lobe infiltrate
• 36
• 37
• ID. Right upper-lobe atelectasis
• 38 • E Right upper-lobe infiltrate
• 39
• F. Thymus
• 40
• 41
• 42 0
Feedback Suspend End Block
• 1
• 2 Item: 31 of 44 F' Mark
▪ 3 Previous Next Lab Values. Notes Calculator.
• 4-
▪ 5
▪ 6 A 3-day-old boy is in the neonatal intensive care unit for management of prematurity. For the past few days,
▪ 7 he has had decreased spontaneous movements, decreased tone, seizures, and rapidly increasing head
▪ 8 circumference. The boy was delivered vaginally at 3D weeks gestation by a multiparous woman with cervical
▪ 9 incompetence. Birth weight was 1.36 kg (3 lb). Prenatal laboratory studies and ultrasounds were normal.
• 10 Rupture of membranes occurred 3 hours prior to birth. Amniotic fluid was clear, and there was no maternal
• 11 fever. The neonate's vital signs show intermittent bradycardia and apnea_ Examination shows a lethargic
• 12 neonate with a weak and high-pitched cry, tense fontanels, and generalized hypotonia. Complete blood count
• 13 shows anemia C-reactive protein is normal_ Head ultrasound is shown below.
• 14
• 15
• 16
• 17
• 18
• 19
20
21
• 22
• 23
• 24
• 25
• 26
• 27
• 28
29
• 30

• 32
• 33
• 34
• 35
• 36
• 37
• 38
• 39
What is the most likely cause of the patient's condition? v
• 40
• 41
• 42 0
Feedback Suspend End Block
• 1
• 2 Item: 31 of 44 V-• Mark
▪ 3 Previous Next Lab Values. Notes Calculator.
• 4 shows anemia. C-reactive protein is normal_ Head ultrasound is shown below.
▪ 5
▪ 6
▪ 7
▪ 8
▪ 9
• 10
• 11
• 12
• 13
• 14
• 15
• 16
• 17
• 18
• 19
20
21
• 22
• 23
• 24
• 25
• 26
• 27
• 28 I
29
• 30 What is the most likely cause of the patient's condition?

• 32
A. Arnold-Chiari malformation
• 33
• 34 • B. Choroid plexus cysts
• 35
• C. Craniopharyngioma
• 36
• 37
• D. Dandy-Walker malformation
• 38 • E. Intraventricular hemorrhage



39
40
41
• Meningitis

• 42 0
Feedback Suspend End Block
• 1
• 2 Item: 32 of 44 F' Mark
▪ 3 Previous Next Lab Values. Notes Calculator.
• 4-
▪ 5
▪ 6 A 6-year-old female with Down syndrome is brought to the physician for behavioral changes. Over the past
▪ 7 few weeks, she has begun refusing to do her usual activities. Her parents also report that she seems dizzy
▪ 8 and state that she has developed urinary incontinence. On examination, she is hypotonic but hyperreflexic
▪ 9 with a positive Babinski reflex_ Her gait is ataxic. Which of the following is the most likely diagnosis?
• 10
• 11
• 12 • A. Alzheimer disease
• 13
• B. Atlantoaxial instability
• 14
• 15 • C. Hypothyroidism
• 16
• 17 • D. Mental retardation
• 18 • E Spinal cord infarction
• 19
20
21
• 22
• 23
• 24
• 25
• 26
• 27
• 28
• 29
H
• 31
16
• 31

• 33
• 34
• 35
• 36
• 37
• 38
• 39
• 40
• 41
• 42 0
Feedback Suspend End Block
1
• 2 Item: 33 of 44 F' Mark
3 Previous Next Lab Values. Notes Calculator.
• 4-
5
6 A 2-year-old boy is brought by his mother to the emergency department because of a high-grade fever which
▪ 7 "does not go away" with acetaminophen. For the last four days, the child has been very irritable and is crying
▪ 8 a lot. He is also pulling his ear and not eating well. He has been generally well, other than the occasional sore
▪ 9 throat this season_ His temperature is 38.8° C (102_2a F), blood pressure is 90/60 mm Hg, pulse is 119/min,
• 10 and respirations are 24/min. He appears well nourished, but is irritable_ Physical examination reveals
• 11 enlarged cervical lymph nodes and splenomegaly. The tympanic membranes are inflamed. CBC shows:
• 12
WEC 81,1DD mm3
• 13
Hemoglobin ED g/dL
• 14
Hematocrit 25%
• 15
Platelets 16,000 mm3
• 16
Blast forms 80%
• 17
Prolymphocytes 10%
• 18
Lymphocytes 10%
• 19
20
21
The blast cells have condensed nuclear chromatin, small nucleoli and scant agranular cytoplasm_
• 22
Subsequent histochemical staining reveals strongly positive periodic acid Schiff (PAS) reaction. No Auer rods
• 23
were seen. Which of the following is the most likely diagnosis?
• 24
• 25 .0 A. Burkitt lymphoma
• 26
▪ B. Acute myelocytic leukemia
• 27
• 28 ▪ C. Prolymphocytic leukemia
• 29 .0 ID. Acute lymphoblastic leukemia
• 30
E Myelodysplastic syndrome
• 31
• 32

• 34
• 35
• 36
• 37
• 38
• 39
• 40
• 41
• 42 0
Feedback Suspend End Block
• 1
• 2 Item: 34 of 44 V-• Mark
▪ 3 Previous Next Lab Values. Notes Calculator.
• 4-
▪ 5
▪ 6 A 2-year-old boy is brought to the emergency department due to a cough and a "whistling" sound with
▪ 7 breathing. Two days ago, he developed rhinorrhea, fever, a hoarse cry and a progressively worsening, harsh.
▪ 8 tarky" cough. His immunizations are up-to-date. His 6-year-old brother also has cold symptoms. His
▪ 9 temperature is 3L5° C (99.5° F), pulse is 14G/min, and respirations are 36/min. On examination, he is alert.
• 10 in mild respiratory distress, has a dry barking cough, hoarse cry, and some clear mucoid rhinorrhea_ His
• 11 pharynx is slightly injected, but without enlargement or asymmetry. The lungs are clear on auscultation.
• 12 Lateral neck x-rays reveal a mildly narrowed subglottic region. What is the most likely diagnosis?
• 13
• 14
• 15 O A. Epiglottis
• 16 O B. Croup
• 17
• 18 O C. Laryngotracheobronchopneumonitis
• 19 O D. Foreign body aspiration
20
21 • E Laryngeal diphtheria
• 22
• 23
• 24
• 25
• 26
• 27
• 28
• 29
• 30
1:=
31
• 32
• 33

e
• 35
r• 36 I
• 37
• 38
• 39
• 40
• 41
• 42 0
Feedback Suspend End Block
• 1
• 2 Item: 35 of 44 F' Mark
▪ 3 Previous Next Lab Values. Notes Calculator.
• 4-
▪ 5
You are called to examine a newly delivered infant, who is 41 weeks gestation and a product of an
▪ 6
uncomplicated pregnancy. Physical examination reveals a matted mass of edematous bowel loops
▪ 7
protruding from the abdomen_ There is no covering over the bowel loops. The umbilical cord appears
▪ 8
normal. Which of the following is the best next step in the management of this patient?
▪ 9
• 10 • A. Intravenous nutrition
• 11
• B. Sterile wrapping of exposed bowel
• 12
• 13 • C. Immediate surgery
• 14
• D. Look for associated anomalies
• 15
• 16 E Initiate broad-spectrum antibiotics
• 17
• 18
• 19
20
21
• 22
• 23
• 24
• 25
• 26
• 27
• 28
• 29
• 30
• 31
• 32
• 33
• 34

• 36
• 37
• 39
• 39
• 40
• 41
• 42 0
Feedback Suspend End Block
• 1
• 2 Item: 36 of 44 F' Mark U.P
▪ 3 Previous Next Lab Values, Notes Calculator.
• 4-
▪ 5
▪ 6 An 8-year-old African American child is brought to the office for the evaluation of a 1-day history of fever and
▪ 7 back pain. He has sickle cell disease, and has had 5 hospitalizations for similar painful crises. His laboratory
▪ 8 report shows normocytic anemia, reticulocytosis and leukocytosis. What finding is most likely to be present
▪ 9 on this patient's peripheral smear?
• 10
• 11
• 12 • A. Bite cells
• 13 • B. Helmet cells
• 14
• 15 • C. Howell Jolly bodies
• 16
• ID. Heinz bodies
• 17
• 18 • E Basophilic stippling
• 19
20
21
• 22
• 23
• 24
• 25
• 26
• 27
• 28
• 29
• 30
• 31
• 32
• 33
• 34
35
I 36
37
38
39
• 40
• 41
42 0
Feedback Suspend End Block
• 1
• 2 Item: 37 of 44 F' Mark
▪ 3 Previous Next Lab Values. Notes Calculator.
• 4-
▪ 5
▪ 6 A 1-day-old girl is in the newborn nursery with swollen hands and feet She was born at term by
▪ 7 uncomplicated vaginal delivery_ Her mother had mild pre-eclampsia and swelling of her hands and feet during
▪ 8 pregnancy. The pregnancy was otherwise uncomplicated. Vital signs are normal. Examination shows a girl
▪ 9 with a short webbed neck, dysplastic nails, and bilateral, non-pitting carpal and pedal edema_ Ultrasound of
• 10 the abdomen shows a horseshoe kidney_ Which of the following is the most likely cause of the edema?
• 11
• 12
• 13 • A. Decreased albumin synthesis
• 14 • B. Decreased cardiac contractility
• 15
• 16 • C. Dysgenesis of the lymphatic network
• 17 • D_ Increased capillary permeability
• 18
• 19 • E Increased sodium retention
20 • F. Increased urinary loss of protein
21
• 22
• 23
• 24
• 25
• 26
• 27
• 28
• 29
• 30
• 31
• 32
• 33
• 34
• 35
• 36
a 37
• 38
• 39
• 40
• 41
• 42 0
Feedback Suspend End Block
• 1
• 2 Item: 38 of 44 F' Mark
▪ 3 Previous Next Lab Values. Notes Calculator.
• 4-
▪ 5
▪ 6 A 14-year-old girl is brought to the physician for evaluation of facial puffiness, fatigue, and decreased appetite
▪ 7 for the past few days. The patient recently immigrated from China to the United States. Temperature is 36.7
▪ 8 C (98 F), blood pressure is 110/70 mm Hg, pulse is 80/min, and respirations are 18/min. Physical
▪ 9 examination shows periorbital and pretibial edema. Serum laboratory results are as follows:
• 10
Creatinine 0.9 mg/dL
• 11
Albumin 2.2 mg/dL
• 12
Total bilirubin 0.5 mg/dL
• 13
Aspartate aminotransferase 56 LW_
• 14
Alanine aminotransferase 64 LIIL
• 15
Alkaline phosphatase 97 LIIL
• 16
HRsAg Positive
• 17
HBeAg Positive
• 18
Anti-HEsAg antibodies Negative
• 19
Anti-HCV antibodies Negative
20
Anti-HIV antibodies Negative
21
• 22
• 23
Urinalysis shows 4+ proteinuria, no red blood cells, and no casts. Which of the following is the most likely
• 24
diagnosis in this patient?
• 25
• 26 • A. Focal segmental glomerulosclerosis
• 27
• B. Membranoproliferative glomerulonephritis
• 28
• 29 • C. Membranous nephropathy
• 30 • D. Minimal change disease
• 31
• E. Poststreptococcal glomerulonephritis
• 32
• 33
• 34
▪ 35
▪ 36
- 37
• 38
- 39
- 40
▪ 41
▪ 42 0
Feedback Suspend End Block
A'S
• 1
• 2 Item: 39 of 44 F Mark
▪ 3 Previous Next Lab Values. Notes Calculator.
• 4-
▪ 5
▪ 6 A 1-week-old boy in the neonatal intensive care unit has central cyanosis. He was born at 40 weeks gestation
▪ 7 to a healthy 25-year-old woman who had an uncomplicated pregnancy and delivery. Family history is notable
▪ 8 for a maternal uncle who died of hypoplastic left heart syndrome shortly after birth. Auscultation shows a III
▪ 9 holosystolic murmur that is loudest at the left lower sternal border_ The lungs are clear bilaterally. Chest
• 10 radiograph reveals decreased pulmonary vascular markings and a normal-sized heart. Left axis deviation and
• 11 tall, peaked P waves are shown in the electrocardiogram below.
• 12
• 13
• 14
• 15
• 16
• 17
• 18
• 19
20
21
• 22
• 23
• 24
• 25
• 26
• 27


28
29
II a VI_ V2
(u
• 30
• 31
• 32
• 33
• 34

n ty,
III V6

I
se

0
Feedbac k Suspend End Block
• 1
• 2 Item: 39 of 44 V-• Mark -<1
U.P
▪ 3 Previous Next Lab Values, Notes Calculator.
• 4
▪ 5
▪ 6
▪ 7
▪ 8
▪ 9
• 10
• 11
• 12
• 13
• 14
• 15 aVL VZ
• 16
• 17 _.....L...L...L L_.L_ L_ i 1 ! I 1 .1 .1 .1 .1 .1 .1 I .1 i t

• 18
• 19
20
21
• 22
• 23
• 24
• 25
• 26
• 27
• 28
USIOLEWorld, LLC
• 29
• 30 What is the most likely diagnosis in this patient?
• 31
• 32
• 33
• A. Complete atrioventricular canal defect
• 34 • B. Ebstein's anomaly
• 35
• C. Tetralogy of Fallot
• 36
• 37 • D. Total anomalous pulmonary venous return
• 38 E Tricuspid valve atresia
• Truncus arteriosus
• 40
• 41
• 42 0
Feedback Suspend End Block
• 1
• 2 Item: 40 of 44 F' Mark
▪ 3 Previous Next Lab Values. Notes Calculator.
• 4-
▪ 5
▪ 6 A 7-year-old boy is brought to the emergency department for a suspected fracture. His mother says that he
▪ 7 was running around the house and fell on the carpet an hour ago. The boy has been crying and complaining
▪ 8 of thigh pain since the fall. He did not hit his head or lose consciousness. His past medical history is notable
▪ 9 for mild hearing loss and multiple fractures after seemingly minor injuries. On physical examination, he has
• 10 decreased muscle tone throughout The right thigh is markedly tender to palpation with obvious deformity.
• 11 Multiple bruises in various stages of healing are present on his extremities. His eye examination is shown
• 12 below.
• 13
• 14
• 15
• 16
• 17
• 18
• 19
20
21
• 22
• 23
• 24
• 25
• 26
• 27
• 28
• 29
• 30
• 31
• 32
• 33
• 34
• 35
• 36
• 37 1.1.Thich of the following is most likely associated with this patient's underlying condition?
• 39
• 39
0 A. Aortic root dilatation se
r

41
42 411 0
Feedback Suspend End Block
A'S
• 1 I
• 2 Item: 40 of 44 V- Mark
▪ 3 Previous Next Lab Values. Notes Calculator.
I I ILJ3L Ii, LL/114-. LI II I I 14-. I iii IL LI ii i I is I IlLilro—uly TYILI I uuviuu.a L.14-.1L/I I I IlLy.
• 4
Multiple bruises in various stages of healing are present on his extremities. His eye examination is shown
▪ 5
below.
▪ 6
▪ 7
▪ 8
▪ 9
• 10
• 11
• 12
• 13
• 14
• 15
• 16
• 17
• 18
• 19
20
21
• 22
• 23
• 24
• 25
• 26
• 27
• 28
• 29
• 30 Which of the following is most likely associated with this patient's underlying condition?
• 31
• 32
• 33
• A. Aortic root dilatation
• 34 • B. Intellectual disability
• 35
• C. Opalescent teeth
• 36
• 37
• ID. Retinal hemorrhages
• 30 E Subdural hematoma
39
• Vitamin ID deficiency
• 40 Lai
• 41
411 0
Feedback Suspend End Block
• 1
• 2 Item: 41 of 44 F' Mark
▪ 3 Previous Next . Lab Values. Notes Calculator.
• 4-
▪ 5
▪ 6 A 6-week-old boy is brought to the physician with persistent, worsening vomiting over the past 2 weeks. The
▪ 7
emesis occurs with every feed, is nonbilious, and is projectile in nature_ The emesis has persisted even
▪ 8 though his mother has tried small, frequent feeds and holding the infant upright after feeds. On physical
▪ 9 examination, peristaltic waves are seen over the upper abdomen, and an olive-shaped mass is palpated in the
• 10 right upper quadrant Laboratory results are as follows:
• 11
Serum chemistry
• 12
Sodium 133 mEq/L
• 13
Potassium 2_8 mEq/L
• 14
Chloride 90 mEq!L
• 15
Bicarbonate 36 mEq!L
• 16
Blood urea nitrogen 18 mg/dL
• 17
Creatinine 0_6 mg/I:IL
• 18
Glucose 100 mg/dL
• 19
20
21
Which of the following is the most appropriate next step in management of this patient?
• 22
• 23 O A. Elective surgery later in childhood
• 24
O B. Immediate surgery
• 25
• 26 C. Intravenous hydration and potassium replacement
• 27 • ID. Oral metoclopramide
• 28
• 29
• E Switch to a hydrolyzed formula
• 30
• 31
• 32
• 33
34
35
36
37
38
39
• 40

0
Feedback Suspend End Block
1
• 2 Item: 42 of 44 V- Mark
▪ 3 Previous Next Lab Values. Notes Calculator.
• 4-
▪ 5
▪ 6 A newborn boy is in the delivery room with respiratory distress. He was born vaginally at 28-weeks gestation
7 due to maternal pre-eclampsia. Examination shows a cyanotic tachypneic boy with intercostal and subcostal
8 retractions and nasal flaring. On lung auscultation, coarse breath sounds are heard bilaterally. Respiratory
9 support with continuous positive airway pressure is provided. The patient is admitted to the neonatal intensive
• 10 care unit where he is intubated due to worsening respiratory status. An orogastric tube is placed to
• 11 decompress the stomach. A chest radiograph is obtained. In addition to prematurity, which of the following
• 12 is a risk factor for the development of respiratory distress syndrome?
• 13
• 14
• 15 • A. Antenatal corticosteroids
• 16 • B. Intrauterine growth restriction
• 17
• 18 • C. Maternal diabetes mellitus
• 19 • ID. Maternal hypertension
20
21 • E Prolonged rupture of membranes
• 22 • F. Vaginal delivery
• 23
• 24
• 25
• 26
• 27
• 28
• 29
• 30
• 31
• 32
• 33
• 34
• 35
• 36
• 37
• 39
• 39
• 40
• 41
0
Feedback Suspend End Block

• 2 Item: 42 of 44 V-Mark
Previous Next Lab Values
FF.
Notes
11
.
Calculator
▪ 3
• 4- Media Exhibit
▪ 5
▪ 6 Media 1 of 1
▪ 7
▪ 8
▪ 9
• 10
• -1 -1
• 12
• 13
• 14
• 15
• 16
• 17
• 19
• 19
20
21
• 22
• 23
• 24
• 25
• 26
• 27
• 28
• 29
• 30
• 3-1
• 32
• 33
34
35
36
37 V
39
• 39
• 40
• 41
a 42 0
Suspend End Block
Feedback
• 1
• 2 Item: 43 of 44 F' Mark
▪ 3 Previous Next Lab Values. Notes Calculator.
• 4-
▪ 6
▪ 6 An overweight 12-year-old boy presents with left knee pain that has been going on intermittently for the past
▪ 7 three months. Physical activity, especially stair climbing, exacerbates the pain. The boy's mother also points
▪ 8 out that he has been limping recently. On physical examination, his anterior left hip is moderately tender to
▪ 9 palpation, and when he is asked to stand on his left leg, the right half of his pelvis tilts downward. Which of the
• 10 following best explains this finding?
• 11
• 12
• 13 • A. Tensor fascia lata weakness
• 14 • B. Psoas muscle weakness
• 15
• 16 • C. Quadratus lumborum weakness
• 17 • D. Quadriceps muscle weakness
• 18
• 19 • E. Gluteus muscle weakness
20
21
• 22
• 23
• 24
• 25
• 26
• 27
• 28
• 29
• 30
• 31
• 32
• 33
• 34
• 35
• 36
• 37
• 38
• 39
• 40
• 41
.
• 11 1.. 0
Feedback. Suspend End Block
1
2 Item: 44 of 44 V-Mark
3 Previous Next Lab Values. Notes Calculator,
4-
5 A 14-month-old boy is admitted to the hospital for treatment of pneumonia. He has a fever, cough, and
6 increased work of breathing. Sick contacts include multiple children in day care who have had a "cold." His
7 mother is concerned that he is "frequently ill" as he had 3 episodes of bronchiolitis during infancy and
8 pneumococcal pneumonia 2 months ago that required intubation and hospitalization in the intensive care
9 unit He also had tympanostomy tubes placed recently for recurrent otitis media. He takes no medications
10 and his vaccinations are up to date. Weight is 7 kg (15.4 Ib, <3rd percentile). Temperature is 39.5 C (103.2
11 F), blood pressure is 90/50 mm Hg, pulse is 128/min, and respirations are 38/min. On examination, the
12 patient appears listless and is in mild respiratory distress_ Both ear canals contain purulent
13 drainage. Crackles are heard in the left lung base. Laboratory results are shown below.
14 Complete blood count
15 Hemoglobin 12.6 g/dL
16 Platelets 260,000/pL
17 Leukocytes 9,000/pL
18 Neutrophils 40%
19 Lymphocytes 50%
20
21 Immunoglobulins
22 IgG 250 mg/dL (normal = 700-1500 mg/dL)
23 IgA 24 mg/dL (normal = 60-400 mg/dL)
24 IgM 450 mg/dL (normal = 60-300 mg/dL)
25
26 CD4/CD8 ratio 2.2 (normal = 1-4)
27 HIV-1 antibody negative
28
29 Which of the following is the most likely diagnosis?
30
31
O A. Bruton agammaglobulinemia
32
33 O B. Common variable immunodeficiency
34 O C. Hyper-IgM syndrome
35
O ID. IgA deficiency
36
37 O E. Munchausen syndrome by proxy
38 • Selective IgG subclass deficiencies
39
O G. Transient hypogammaglobulinemia of infancy
40
41
42 1111
Feedback.
IE

Suspend
a
End Block
• 2 Item: 1 of 44 F' Mark
. 3 Previous Next Lab Values. Notes Calculator.

• 4
• 5

• 6 A n-year-old girl is brought to the physician due to pallor, decreased energy, bloody diarrhea, and "spots" over
. 7 her arms and legs that developed over the past 2 days_ Her immunizations are up to date. The patient's past
• 8 medical history is unremarkable and she has been generally healthy. The girl has no fever, vomiting, or joint
• 9 pain_ She has had no travel or animal exposures. The patient's temperature is 37_1 C (98.8 F). On
• 10 examination, she appears lethargic and weak; her face and extremities are swollen and she has poor skin
• 11 turgor. The patient has scattered petechiae and a few purpura on her arms. She also has moderate, non-
• 12 localized abdominal tenderness_ Laboratory studies are as follows:
• 13
Hemoglobin 6.4 g/dL
• 14
Platelets 45,000/pL
• 15
Creatinine 2.3 mgldL
• 16
Total bilirubin 3 mgidL
• 17
Direct bilirubin 02 nigidL
• 18
• 19
Urinalysis
20
Specific gravity 1.025
21
pH 5
• 22
Protein +2
• 23
Blood moderate
• 24
Leukocyte esterase negative
• 25
Nitrites negative
• 26
Bacteria none
• 27
White blood cells 50+1hpf
• 28
Red blood cells 20-30/hpf
• 29
Casts hyaline casts
• 30
• 31
• 32
Which of the following is the most likely diagnosis?
• 33
. 34 A. Hemolytic-uremic syndrome [69136]
• 35
• Henoch-Schbnlein purpura [21%]
• 36
• 37 C. Immune thrombocytopenic purpura [8%]
• 39 D. Post-streptococcal glomerulonephritis [2'3C]
• 39
E Rocky Mountain spotted fever [0°•0] se
• 40
• 41
• 42 End Block
Feedback
• 2 Item: 1 of 44 V- Mark
• 3 Previous Next Lab Values. Notes Calculator.
• 4 A. Hemolytic-uremic syndrome [69%] A
• 5
B. Henoch-SchOnlein purpura [21%]
• 6
• 7 C. Immune thrombocytopenic purpura [8%]
• 8 D. Post-streptococcal glomerulonephritis [2%]
• 9
E Rocky Mountain spotted fever [0%]
• 10
• 11
• 12 Explanation:
• 13
• 14
• 15 Hemolytic-uremic syndrome
• 15
_
• 17
• 19 Enterocolitis from Shiga toxin-producing bacteria
Etiology
• 19 (E coil 0157:HI, Shigelfa) most common
20
21
• 22 • Diarrhea (often bloody)
• 23 • Lethargy, irritability, pallor
Presentation
• 24 • Bruising or petechiae
• 25 • Oliguria, edema
• 26
. ,
• 27
• 28 • Hemolytic anemia
• 29 • Thrombocytopenia
• 30
Laboratory
F indings • 4 Creatinine, hematuria, proteinuria, casts (due
• 31
to glomen_ilar hemolysis)
• 32
• t Bilirubin (clue to hemolysis)
• 33
• 34
• 35 • Fluid & electrolyte management
• 36
Treatment • Blood transfusions
• 37
• 39
• Dialysis
• 39
QUSMLEWorldr LLC
• 40
• 41
• 42 End Block
Feed back
: 1 of 44 r VMark
▪ 3 Previous Next Lab Values. Notes Calculator.

• 4-
▪ 5 Hemolytic-uremic syndrome
▪ 6
_
▪ 7
▪ 8 Enterocolitis from Shiga toxin-producing bacteria
Etiology
▪ 9 (E coli 0157:H7, Shigeita) most common
• 10
• 11
• 12 • Diarrhea (often bloody)
• 13 • Lethargy, irritability, pallor
Presentation
• 14 • Bruising or petechiae
• 15 • Oliguria, edema
• 16 . . ,
• 17
• 18 • Hemolytic anemia
• 19 • Thrombocytopenia
20
Laboratory
Findings • 4 Creatinine, hematuria, proteinuria, casts (due
21
to glomerular hemolysis)
• 22
• 1 Bilirubin (due to hemolysis)
• 23
• 24 " ,
• 25 • Fluid & electrolyte management
• 26 Treatment • Blood transfusions
• 27
• Dialysis
• 28
• 29
IDLISPilLEWorkir LLC
• 30
• 31 Hemolytic-uremic syndrome (HUS) is a clinical syndrome consisting of hemolytic anemia,
• 32 thrombocytopenia, and acute renal failure. It is most commonly seen in toddlers. More than 90% of HUS
• 33 occurs due to shiga-toxin producing diarrheogenic pathogens such as Escherichia coil 0157:H7 or
• 34 Shigetta_ Approximately 10% of HUS cases are due to infection with Streptococcus pneumornae; these
• 35 patients have pneumonia or meningitis instead of diarrhea. In both settings, the toxins enter the systemic
• 36 circulation and injure the endothelial cells in the kidney.
• 37
• 39 Children with HUS generally have fatigue/lethargy and pallor due to the hemolysis. In addition, the hemolysis
• 39 often leads to some degree of hyperbilirubinemia. If the central nervous system is affected, irritability or
se
• 40 hpqriqrhp Mali 11P hr-pQpnt Prhhr-pQQiup renal inwhlwpmpnt Ipqriq to riprnmqqpri I irinp r71Itn1It qnri nrncc nr
• 41
• 42 End Block
Feedback
Item: 1 of 44 V-Mark
▪ 3 Previous Next Lab Values. Notes Calculator.
• 4- Shigelia_ Approximately 1G% of HUS cases are due to infection with Streptococcus pneumornae; these
▪ 5 patients have pneumonia or meningitis instead of diarrhea In both settings, the toxins enter the systemic
▪ 6 circulation and injure the endothelial cells in the kidney.
▪ 7
▪ 8 Children with HUS generally have fatigue/lethargy and pallor due to the hemolysis. In addition, the hemolysis
▪ 9 often leads to some degree of hyperbilirubinemia. If the central nervous system is affected, irritability or
• 10 headache may be present Progressive renal involvement leads to decreased urine output and gross or
• 11 microscopic hematuria. As the kidney function worsens, fluid overload develops (eg, pulmonary edema,
• 12 congestive heart failure)_
• 13
• 14 Once HUS is diagnosed, the management is supportive. Fluid and electrolyte balance must be maintained,
• 15 and anemia and thrombocytopenia should be managed with conservative transfusions. Approximately half of
• 16 all children with HUS will require dialysis for their acute renal failure. With supportive care, the mortality of
• 17 HUS is <5%.
• 18
• 19 (Choice B) Henoch-SchOnlein purpura presents with purpura on the legs and buttocks in the setting of a
20 normal platelet count In this scenario, the thrombocytopenia, lack of joint pain, distribution of the purpura in
21 the upper extremities, and the history of preceding diarrhea are more suggestive of HUS.
• 22
(Choice C) Immune thrombocytopenic purpura can present with thrombocytopenia and purpura. However,
• 23
• 24
anemia and renal failure are not seen in immune thrombocytopenic purpura.
• 25
(Choice 0) Post-streptococcal glomerulonephritis (P N) can present with hematuria and signs of volume
• 26
overload after infection with group A Streptococcus. If present, thrombocytopenia and anemia are generally
• 27
mild, and renal failure is unusual in PSGN.
• 28
• 29 (Choice E) Rocky Mountain spotted fever (RMSF) can cause a vasculitis, including petechiae, anemia,
• 30 thrombocytopenia, and hematuria. Children with RMSF are almost invariably febrile. In addition, this patient
• 31 does not have an exposure history that would place her at risk for RMSF or other tick-borne infections.
• 32
• 33 Educational objective:
• 34 Hemolytic-uremic syndrome is a clinical syndrome of hemolytic anemia, thrombocytopenia, and acute renal
• 35 failure. It is most often caused by Shiga-toxin-mediated endothelial cell damage, after infection with
• 36 Escherichia cob. 0157:H7 or Shigella_ Treatment is supportive; approximately 5G% of children with
• 37 hemolytic-uremic syndrome require dialysis.
• 39


39
40 TReferences: se
• 41
• 42 End Block
Feedback
• 2 Item: 1 of 44 V-• Mark
• 3 Previous Next Lab Values. Notes Calculator.
• 4 congestive near[ failure).
▪ 5
▪ 6
Once HUS is diagnosed, the management is supportive. Fluid and electrolyte balance must be maintained,
and anemia and thrombocytopenia should be managed with conservative transfusions. Approximately half of
▪ 7
all children with HUS will require dialysis for their acute renal failure. With supportive care, the mortality of
▪ 8
HUS is <5%.
▪ 9
• 10
(Choice B) Henoch-SchOnlein purpura presents with purpura on the legs and buttocks in the setting of a
• 11
normal platelet count In this scenario, the thrombocytopenia, lack of joint pain, distribution of the purpura in
• 12
the upper extremities, and the history of preceding diarrhea are more suggestive of HUS.
• 13
• 14 (Choice C) Immune thrombocytopenic purpura can present with thrombocytopenia and purpura. However,
• 15 anemia and renal failure are not seen in immune thrombocytopenic purpura.
• 16
• 17 (Choice 0) Post-streptococcal glomerulonephritis (P N) can present with hematuria and signs of volume
• 18 overload after infection with group A Streptococcus. If present, thrombocytopenia and anemia are generally
• 19 mild. and renal failure is unusual in PSGN.
20
21 (Choice E) Rocky Mountain spotted fever (RMSF) can cause a vasculitis, including petechiae, anemia,
• 22 thrombocytopenia, and hematuria. Children with RMSF are almost invariably febrile. In addition, this patient
• 23 does not have an exposure history that would place her at risk for RMSF or other tick-borne infections.
• 24
• 25 Educational objective:
• 26 Hemolytic-uremic syndrome is a clinical syndrome of hemolytic anemia, thrombocytopenia, and acute renal
• 27 failure. It is most often caused by Shiga-toxin-mediated endothelial cell damage, after infection with
• 28 Escherichia cob. 0157:H7 or Shigella_ Treatment is supportive; approximately 50% of children with
• 29 hemolytic-uremic syndrome require dialysis.
• 30
• 31 References:
• 32
1. HUS and TTP in children
• 33
• 34 2. Update on Streptococcus pneumoniae associated hemolytic uremic syndrome
• 35 3. Shiga toxins and the pathophysiology of hemolytic uremic syndrome in humans and
• 36 animals.
• 37
• 39
• 39
Copyright © Mond Last updated: [11/11/2014]
• 40
• 41
• 42 End Block
Feedback
1
2 Item: 2 of 44 V-Mark
▪ 3 Previous Next Lab Values. Notes Calculator.
• 4
▪ 5
A 2-week-old baby girl is brought to the clinic for the evaluation of vaginal discharge. Her mother has noticed a
▪ 6
cloudy white vaginal discharge mixed with blood since yesterday. She was born by normal vaginal delivery at
▪ 7
term, and no congenital anomalies were noted at birth. Physical examination reveals a blood-stained,
▪ 8
odorless vaginal discharge. Her vital signs are stable. What is the most appropriate next step in the
▪ 9
management of this patient?
• 10
• 11 A. Treatment for Chlamydia [213q
• 12
• 13 B. Treatment for HSV-2 [1%]
• 14
• 15 C. Rule out vaginal cancer [31C]
• 16
• 17
• ID. Reassurance [82%]
• 18
• 19
E Treatment for Trichomonas vaginalis [1%]
20
21
• 22
• Suspect child abuse [10%]
• 23
• 24
• 25 Explanation:
• 26
• 27 Female infants less than 3 months of age sometimes develop vaginal spotting or bleeding. Maternal
• 28 estrogens can cross the placenta and enter the fetal blood stream before birth, thereby causing a pubertal
• 29 effect in the newborn which disappears as soon as the hormone is cleared from the infant's circulation.
• 30
• 31 (Choice A) Chlamydial infection in the newborn occurs through transmission from the genital tract of infected
• 32 mothers. Infected infants may present with pneumonia and conjunctivitis. Vaginal discharge is not a feature
• 33 of chlamydial infection in the newborn.
• 34
• 35 (Choice B) Most herpes infections in the newborn are caused by HSV-2. Infection occurs while the infant
• 36 passes through the infected birth canal during delivery. This presents as a disseminated form of
• 37 infection, localized central nervous system disease, or localized infection of the skin, eyes, and mouth.
• 39
(Choice C) Vaginal cancer usually occurs in women over the age of 5G, with the exception of women who
• 39
were exposed in utero to diethylstilbestrol. In this group of women, vaginal cancer occurs at an earlier age
• 40
• 41
• 42 End Block
Feedback
1
2 Item: 2 of 44 A V-Mark
▪ 3 Previous Next Lab Values. Notes Calculator.
• 4
▪ 5
▪ D. Reassurance [82%]
▪ 6
▪ 7 E Treatment for Trichomonas vaginalis [1%]
▪ 8
▪ 9 • Suspect child abuse [10%]
• 10
• 11
• 12
Explanation:
• 13
• 14
Female infants less than 3 months of age sometimes develop vaginal spotting or bleeding. Maternal
• 15
estrogens can cross the placenta and enter the fetal blood stream before birth, thereby causing a pubertal
• 16
effect in the newborn which disappears as soon as the hormone is cleared from the infant's circulation.
• 17
• 18 (Choice A) Chlamydial infection in the newborn occurs through transmission from the genital tract of infected
• 19 mothers. Infected infants may present with pneumonia and conjunctivitis. Vaginal discharge is not a feature
20 of chlamydial infection in the newborn.
21
• 22 (Choice B) Most herpes infections in the newborn are caused by HSV-2. Infection occurs while the infant
• 23 passes through the infected birth canal during delivery. This presents as a disseminated form of
• 24 infection, localized central nervous system disease, or localized infection of the skin, eyes, and mouth.
• 25
• 26 (Choice C) Vaginal cancer usually occurs in women over the age of 50, with the exception of women who
• 27 were exposed in utero to diethylstilbestrol. In this group of women, vaginal cancer occurs at an earlier age
• 28 (Le., under the age of 30), but still not in the neonatal period.
• 29
• 30 (Choice El Trichomonas infection is a sexually transmitted disease that is common in adolescents and in
• 31 sexually active males and females.
• 32
(Choice F) This child's discharge is physiological. There is no odor or discoloration. There is no reason to
• 33
suspect abuse in this child_
• 34
• 35
Educational Objective:
• 36
Vaginal discharge in the newborn is due to the effects of maternal estrogens. In such cases, reassurance of
• 37
the mother is all that is required.
• 39
• 39
Copyright © UWorld Last updated: [9/15/2014]
• 40 IR
• 41
• 42 End Block
Feedback
1
Item: 3 of 44 V-• Mark
Previous Next Lab Values. Notes Calculator.

▪ 5
▪ 6 A previously healthy 15-year-old boy is brought to the physician for progressive muscle weakness. Over the
▪ 7 past 2 months, he has had increasing difficulty making facial expressions, including smiling and frowning_ He
▪ 8 also has had difficulty swallowing but has no pain_ The boy takes no medications and his vaccinations are up
▪ 9 to date. He was adopted as an infant and his biological family history is unknown_ Physical examination
• 10 shows ptosis, temporal wasting, thin cheeks, emaciated extremities, atrophy of the thenar and hypothenar
• 11 eminences, inverted V-shaped upper lip, and testicular atrophy. Delayed relaxation is noted on contraction of
• 12 the thenar and hypothenar muscles_ Testicular volume is small for age. Neurologic examination shows
• 13 normal deep-tendon reflexes and a negative Babinski sign. What is the most likely mode of genetic
• 14 transmission of the patient's condition?
• 15
• 16
A. Autosomal dominant [29%]
• 17
• 18 • Autosomal recessive [1916]
• 19 C. De novo mutation [1%]
20
X-linked dominant [1196]
21
• 22 E X-linked recessive [35'36]
• 23
• 24
Explanation:
• 25
• 26
• 27 Muscular dystrophies
• 28
• 29 Diagnosis Duchenne Becker Myotonic
• 30
• 31 Autosomal
• 32 dominant
• 33 expansion of a
• 34 X-linked recessive deletion of dystrophin CTG trinucleotide
• 35 Genetics
gene on chromosome Xp21 repeat in DMPK
• 36
gene on
• 37
chromosome 19q
• 38
• 39
13.3
• 40
se
• 41
• 42 End Block
Feedback
-1
: 3 of 44 r Flilark
Previous Next Lab Values, Notes Calculator.

▪ 5
Muscular dystrophies
_
▪ 6
Diagnosis Duchenne Becker Myotonic
▪ 7
▪ 8
Autosomaf
0
dominant
• 10
11
expansion of a
X-linIced recessive deletion of dystrophin CTG trinucleotide
• 12 Genetics
13 gene on chromosome Xp21 repeat in DMPK
• 14 gene on
15 chromosome 19q
• 13.3
17
• 18 • Onset: Age 3-5
• Onset: Age 5-15 • Onset: Age 12-
19
• Progressive 30
20 • Milder weakness
Clinical weakness,
21 compared to • Facial weakness,
Gower
• 22 presentation Duchenne hand grip
• 23
maneuver, calf
muscular rnyotonia,
• 24 pseudohyper-
dystrophy dysphagia
• 25 trophy
• 26
. —
▪ '7
Z e Arrhythmias
• 28 • Intellectual • Cataracts
• 29 Comorbidities disability Cardiornyopathy • Balding
• 30
• 31
• Cardiomyopathy • Testicular
• 32 atrophyfinfertility
• 33
• 34 • Wheelchair-
dependent by
• 36 adolescence Death from
Death by age 40-50 respiratory or heart
Prognosis • Death by age 20-
• 38 from heart failure failure depending
30 from on age of onset
• 39
• 40
respiratory or v
• 4-1
• 42 End Block,
Feedback
• 1
2 Item: 3 of 44 VI/lark U.P
Previous Next Lab Values, Notes Calculator.
• 4
SM LEWorld, LLC
• 5
6
• This patient's presentation is classic for myotonic muscular dystrophy type 1 (Steinert disease). It is the most
. 7
common adolescent/adult-onset muscular dystrophy, affecting —1 in 8000 persons in the United States. An
• 8
autosomal dominant expansion of trinucleotides on the dystrophia myotonica protein kinase gene causes
• 9
myotonia; weakness of skeletal, smooth, and cardiac muscles; and problems in multiple other organ
• 10
systems (Table).
• 11
• 12 Myotonia (delayed muscle relaxation) is most notable when the patient is unable to release the hand after a
• 13 handshake (grip myotonia}_ Skeletal muscle weakness is prominent in the face, forearms, hands, and ankle
• 14 dorsiflexors (eg, bilateral foot drop). Dysphagia, the most dangerous smooth muscle manifestation,
• 15 significantly increases the risk of aspiration pneumonia. Cardiac involvement includes conduction problems
• 16 and arrhythmias. Other manifestations include cataracts, testicular atrophy/infertility, frontal baldness, and
• 17 insulin resistance.
• 18
• 19 (Choice B) Congenital muscular dystrophies and some types of limb-girdle muscular dystrophies are
20 inherited in an autosomal recessive manner.
21
• 22 (Choices C and E) Duchenne and Becker muscular dystrophies are X-linked recessive myopathies,
• 23 although some cases are sporadic.
• 24
• 25 (Choice 0) Fragile X syndrome is an X-linked dominant disorder characterized by macroorchidism, not
• 26 testicular atrophy. Boys with fragile X syndrome can have low muscle tone but not the significant facial
• 27 weakness and grip myotonia seen in myotonic muscular dystrophy_ There are no known X-linked dominant
• 28 muscular dystrophies_
• 29
Educational objective:
• 30
Myotonic muscular dystrophy is an autosomal dominant disorder characterized by grip myotonia (delayed
• 31
muscle relaxation), facial weakness, foot drop, dysphagia, and cardiac conduction anomalies. Other
• 32
problems include cataracts, testicular atrophy/infertility, and baldness.
• 33
. 34
• 35 References:
• 36 1. The myotonic dystrophies: molecular, clinical, and therapeutic challenges.
• 37
• 38
• 39
Copyright © UWorld Last updated: [11/11/2014]
• 40 Ii
• 41
• 42 End Block
Feedback
2 Item: 4 of 44 V-• Mark -4(1
Previous Next Lab Values. Notes Calculator.

6 A 6-year-old boy with congenital deafness is brought to the emergency department after fainting. He was
7 running with friends in his backyard when he suddenly dropped to the ground. He regained consciousness
8 quickly and without confusion_ Review of systems is negative for shaking movements, tongue-biting, or
▪ 9 urinary incontinence. He has no history of similar episodes. The boy has met all developmental milestones
• 10 and takes no medications. His past medical history is significant for bilateral cochlear implantation at age 2
• 11 for sensorineural hearing loss. He had a brother who drowned suddenly at age 7. Blood pressure is 110/70
• 12 mm Hg and heart rate is 70imin and regular. Physical examination is unremarkable. Serum chemistry is
• 13 normal. Electrocardiogram is obtained_ Which of the following therapies is most appropriate for this patient?
• 14
• 15
A. Ethosuximide [9%]
• 16
• 17 B. Levetiracetam [5%]
• 18 Propranolol and pacemaker [50%]
• 19
ID. Quinidine [12%]
20
21 E. Sotalol [12%]
• 22 • Verapamil [13%]
• 23
• 24
• 25 Explanation:
• 26
• 27
• 28
Causes of QT prolongation
• 29
• 30 • Hypocalcemia
• 31
Electrolyte
• Hypokalemia
• 32 derangements
• Hypomagnesemia
• 33
• 34
• 35 • Macrolide antibiotics
• 36
• 37
• Fluoroquinolone antibiotics
• 38 • Psychotropic medications
• 39 • Antipsychotics
-,__ ._ I, _ __.,_, . se
• 40
• 41
• 42 End Block
Feedback
2 Item: 4 of 44 F'Mark
3 Previous Next Lab Values, Notes Calculator.
A

6
Causes of QT prolongation
,
▪ 8 • Hypocalcernia
0
Electrolyte
• Hypokalemia
▪ 10 derangements
• Hypomagnesemia
11
• 12
13 • Macrolide antibiotics
• 14
15
• Fluoroquinolone antibiotics
• • Psychotropic medications
17 • Antipsychotics
• 18
• Tricyclic antidepressants
19
• Selective serotonin reubtake inhibitors
20
21 Acquired • Opioids
• 22 • Methadone
• 23 Medication- • Oxycodone
• 24 induced
• 25 • Antiemetics
• 26 • Ondansetron
• 27 • Granisetron
• 28
• Antiarrhythrnics
• 29
• 30 • Quinidine
• 31 • Procainarnide
• 32 • Flecainide
• 33 • Arniodarone
• 34
• Sotalol
• 36
• Jervell and Lange-Nielsen syndrome (autosomal recessive)
• 38
Inherited
• Romano-Ward syndrome (autosonnal dominant)
• 39
• 40 OILJSIALEWorld. LLC
v
• 41
• 42 End Block,
Feedback
• 1
• 2 Item: 4 of 44 F'Mark
• 3 Previous Next Lab Values. Notes Calculator.
4
• Sotalol
• 5
• 6
• 7 • Jervell and Lange-Nielsen syndrome (autosomal recessive)
• 8
Inherited
• Romano-Ward syndrome (autosonnal dominant)
• 9
• 10 CIIJSMLEWoridr LLC
• 11
• 12 Depolarization and repolarization of the ventricles occur during the QT interval, the duration of which varies by
• 13 heart rate but is normally <44Q ms in males and <4&I ms in females. Prolonged QT intervals are most
• 14 commonly acquired by medication side effect or electrolyte derangements but can sometimes be inherited
• 15 (Table). This patient's family history of sudden death, congenital sensorineural deafness, and QT interval
• 16 of -'600 ms are concerning for Jervell and Lange-Nielsen syndrome, an autosomal recessive congenital long
• 17 QT syndrome caused by molecular defects in potassium channels_ Patients with congenital long QT
• 18 syndrome are at high risk of syncope, life-threatening ventricular arrhythmias such as torsade de pointer,
• 19 and sudden death_
20
21 Treatment consists of refraining from vigorous exercise; avoiding medications that can lengthen the QT
• 22 interval; maintaining normal levels of calcium, potassium, and magnesium; and pharmacotherapy. Beta
• 23 blockers (eg, propanolol) are class II antiarrhythmics and the medication class of choice to blunt exertional
• 24 heart rate and shorten the QT interval. Symptomatic patients (eg, lightheadedness, palpitations) or those with
• 25 a history of syncope require beta blocker therapy plus long-term pacemaker placement
• 26
(Choice A) Ethosuximide is the first-line anti-epileptic for absence seizures which typically presents at age
• 27
4-1G. However, this diagnosis is unlikely as the loss of consciousness would not be accompanied by loss of
• 28
body tone.
• 29
• 30
(Choice B) Levetiracetam is an anti-epileptic medication. Seizure is unlikely in this patient due to lack of
• 31
jerking motions, tongue-biting, urinary incontinence, or postictal confusion/drowsiness_
• 32
• 33 (Choice ID) Quinidine is a class IA anti-arrhythmic agent that blocks both sodium and potassium channels. It
• 34 is contraindicated as its action on potassium channels can prolong the QT interval.
• 35
• 36 (Choice El Sotalol is the only beta blocker that should be avoided in this patient as it is a class III
• 37 anti-arrhythmic agent that can prolong the QT interval by blocking potassium channels.
• 38
• 39 (Choice F) Beta blockers or calcium channel blockers can be used for treatment of hypertrophic
V
• 40 cardiamvanathv Hnwpvp.r nalnium nhannp.I hInnkprs am not uspful in lona OT sync-1=p.
• 41
• 42 End Block
Feedback
1
2 Item: 4 of 44 V-• Mark -4Z1

3 Previous Next Lab Values. Notes Calculator.


4 I byriururrie 4,dU6eU Uy EflUlt1,441cIf Utlel,L6 If I pUL1661Uf El rdLI F ILA WILE! Guriyermai iuriy L.41

5 syndrome are at high risk of syncope, life-threatening ventricular arrhythmias such as torsade de pointer,
6 and sudden death_
7
Treatment consists of refraining from vigorous exercise; avoiding medications that can lengthen the QT
▪ 8
interval; maintaining normal levels of calcium, potassium, and magnesium; and pharmacotherapy. Beta
▪ 9
blockers (eg, propanolol) are class II antiarrhythmics and the medication class of choice to blunt exertional
• 10
heart rate and shorten the QT interval. Symptomatic patients (eg, lightheadedness, palpitations) or those with
• 11
a history of syncope require beta blacker therapy plus long-term pacemaker placement.
• 12
• 13
(Choice A) Ethosuximide is the first-line anti-epileptic for absence seizures which typically presents at age
• 14
4-10. However, this diagnosis is unlikely as the loss of consciousness would not be accompanied by loss of
• 15
body tone.
• 16
• 17 (Choice B) Levetiracetam is an anti-epileptic medication. Seizure is unlikely in this patient due to lack of
• 18 jerking motions, tongue-biting, urinary incontinence, or postictal confusion/drowsiness_
• 19
20 (Choice 13) Quinidine is a class IA anti-arrhythmic agent that blocks both sodium and potassium channels. It
21 is contraindicated as its action on potassium channels can prolong the QT interval.
• 22
• 23 (Choice E) Sotalol is the only beta blacker that should be avoided in this patient as it is a class III
• 24 anti-arrhythmic agent that can prolong the QT interval by blocking potassium channels.
• 25
• 26 (Choice F) Beta blockers or calcium channel blockers can be used for treatment of hypertrophic
• 27 cardiomyopathy. However, calcium channel blockers are not useful in long QT syndrome_
• 28
Educational objective:
• 29
Patients with prolonged QT intervals are at risk for syncope, ventricular arrhythmias, and sudden cardiac
• 30
death. Those with congenital QT prolongation should avoid electrolyte derangements and medications that
• 31
block potassium channels. Beta blockers with pacemaker placement can prevent cardiac arrest.
• 32
• 33
• 34 References:
• 35 1. The Jervell and Lange-Nielsen syndrome: natural history, molecular basis, and clinical
• 36 outcome.
• 37
• 38 r
• 39
Copyright © UWarld Last updated: [11/11/2014]
• 40
• 41
• 42 End Block
Feedback
Item: 4 of 44 'Mark
Previous Next Lab Values Notes Calculator
Media Exhibit L

6 Long QT
7
8 Long QT
▪ g
• 10
• -1 -1 a‘,„
• 12 vnifi
C)r)
• 13
• 14 1 vs
• 15
• 16
• 17
• 19
altIF '473 V4
• 19
20
21
11
• 22
• 23
• 24
• 25
• 26
• 27
• 28
• 29
• 30
• 31
• 32
• 33
• 34
• 35
• 36
• 37
• 39
• 39
• 40


41
42
Feedback
a
End Block
• 2 Item: 4 of 44 r. Mark
Previous Next Lab Values Notes Calculator
Media Exhibit L

▪ 6 Long QT
7
▪ 8
▪ 9 V3
• 10
• -1 -1
• 12 VI
• 13
• 14
• 15
• 16
• 17
• 19
• 19
20
21
• 22
• 23
RR interval
• 24
• 25
• 26
• 27
• 28
• 29
• 30
• 31
• 32
• 33
• 34
Ca interval
• 35
• 36 USOVILEWorld, LLC
• 37 V

• 39
• 39
• 40


41
42
Feedback
a
End Block
Item: 4 of 44 r Mark
EF.
Previous Next Lab Values Notes Calculator
Media Exhibit

6 Torsade de pointes
7
8
▪ 9
• 10
• -1 -1
• 12
• 13
• 14
• 15
• 16
• 17
• 19
• 19
20
21
• 22
• 23
• 24
• 25
• 26
• 27
• 28
• 29
• 30
• 31
• 32
• 33
• 34
• 35
• 36
• 37
• 39
• 39
• 40
• 41
• 42 End Block
Feedback
1
2 Item: 5 of 44 V/lark
3 Previous Next Lab Values. Notes Calculator.

A 3-year-old boy is brought to the physician for evaluation of delayed milestones_ Over the past year, he has
▪ 7 been receiving physical and speech therapies for delays in gross motor skills and speech_ He is currently
▪ 8 able to walk up and down stairs and run well. He has a vocabulary of approximately 25 words and does not
▪ 9 make 2-word sentences. His parents report that he has poor eye contact and does not show interest in
• 10 engaging with other children_ Review of systems is otherwise negative. His vaccinations are up to date. He
• 11 has an uncle with Down syndrome and a cousin with autism. Head circumference, length, and weight are
• 12 each at the 5th percentile_ Examination of the face shows small palpebral fissures, absent philtrum, and a
• 13 thin upper lip border. What is the underlying cause of his problems?
• 14
• 15
• 16
A. Adverse effects from vaccination during infancy [0%]
• 17 B. Fragile X mental retardation 1 gene mutation [4%]
• 18
C. In utero exposure to alcohol [80%]
• 19
D. In utero exposure to cytomegalovirus [1%]
20
21 E In utero exposure to phenytoin [2%]
• 22
E In utero exposure to rubella [1%]
• 23
• 24
G. Nondisjunction resulting in an extra copy of chromosome 21 [12%]
• 25
• 26
Explanation:
• 27
• 28 Fetal alcohol syndrome (FAS) is one of the leading preventable causes of birth defects and
• 29 neurodevelopmental problems. Although in utero alcohol exposure may result in no apparent sequelae for
• 30 some fetuses, others may suffer from FAS or be stillborn. Women who are pregnant or trying to conceive
• 31 should be advised to abstain completely from alcohol as there is no known safe amount of prenatal
• 32 alcohol consumption.
• 33
• 34 FAS is characterized by 3 pathognomonic facial dysmarphisms:
• 35
1. Small palpebral fissures
• 36
2. Smooth philtrum (vertical groove above the upper lip)
• 37
3. Thin vermilion border
• 38
• 39
• 40 qnri/nr wpinht rirrdu+11 is ririmnrinmiciarl with nprrpntilpc <1 fXth fnr qr1P qnri aPu Mir rnr pnhqly i¢ nftpn
• 41
• 42 End Block
Feedback
2 Item: 5 of 44 V-Mark
-4(1

3 Previous Next Lab Values. Notes Calculator.


4

FAS is characterized by 3 pathognomonic facial dysmarphisms:

7 1. Small palpebral fissures


8 2. Smooth philtrum (vertical groove above the upper lip)
9 3. Thin vermilion border
• 10
• 11 Height and/or weight growth is compromised with percentiles 10th for age and sex. Microcephaly is often
• 12 present, and these children suffer from cognitive and behavioral disorders. The phenotypic range of
• 13 neurodevelopmental problems is wide and includes intellectual disability, attention-deficit hyperactivity
• 14 disorder, social withdrawal, and delays in motor and language milestones_ This child has appropriate
• 15 gross motor skills for a 3-year-old, but his language is comparable to that of an 18-month-old. He also has
• 16 socialization problems resembling autism spectrum disorder. Early diagnosis is critical for affected children
• 17 to benefit from aggressive speech, physical, and occupational therapies_
• 18
• 19 (Choices B and G) Down syndrome is the most common genetic cause of developmental delay and usually
20 results from meiotic nondisjunction of chromosome 21. Fragile X syndrome is the most common X-linked
21 inherited cause of intellectual disability. Both of these syndromes can be distinguished from FAS by careful
• 22 physical examination (Table).
• 23
• 24
• 25
Common causes of intellectual disability
• 26
• 27
Syndrome Key physical findings
• 28
• 29
• 30 Face
• 31
• Smooth philtrum
• 32
Fetal alcohol
• 33 • Thin vermilion border
• 34 syndrome
• Small palpebral fissures
• 35
• Microcephaly
• 36
• 37
• 38
Face
• 39
• Flat facial profile V
• 40
• 41
• 42 End Block
Feedback
1
2 44 M fidlark
3 Previous Next Lab Values. Notes Calculator.
4 syrictrume p
- Small palpebral fissures
• Microcephaly
▪ 7
▪ 8
Face
▪ 9
- Flat facial profile
• 10
• 11 • Slanted palpebral fissures
• 12
• Small low-set ears
• 13
Down
• 14 Body
• 15
syndrome
- Excessive skin at nape of the neck
• 16
• 17 • Single transverse palmar crease
• 18 • Clinodactyly
• 19
. Large space between the first two toes
20
21
• 22 • Face
• 23
• Long narrow face
• 24
• 25 • Prominent forehead and chin
• 26 Fragile X • Large ears
• 27 syndrome • Macrocephaly
• 28
• 29
Body
• 30
• 31 • Macroorchidism
• 32
LISMLIVOorld, LLC
• 33
▪ 34
• 35 (Choice A) There is no evidence that vaccinations cause cognitive or behavioral problems.
• 36
(Choice ID) Cytomegalovirus is the most common congenital viral infection and can also cause
• 37
• 38
developmental delay However, other characteristic problems include sensorineural deafness, blindness,
jaundice, hepatosplenomegaly, and petechiae.
• 39
• 40
se
• 41
• 42 End Block
Feedback
• 1 1
• 2 I
• 3 Previous Next Lab Values Notes Calculator.

syndrome • Macrocephaly
• 6
Body
• 7
• 8 • Macroorchidism
▪ 9
a•u 5,141.LEWorld, LLC
• 10
• 11
• 12 (Choice A) There is no evidence that vaccinations cause cognitive or behavioral problems_
• 13
• 14
(Choice 0) Cytomegalovirus is the most common congenital viral infection and can also cause
• 15
developmental delay_ However, other characteristic problems include sensorineural deafness, blindness,
• 16
jaundice, hepatosplenomegaly, and petechiae.
• 17
(Choice E) Hypoplastic fingers/nails and cleft lip/palate are classic physical findings in fetal hydantoin
• 18
syndrome. Pregnant women on phenytoin during their last trimester often receive prophylactic vitamin K to
• 19
prevent neonatal bleeding as phenytoin may increase the rate of fetal vitamin K degradation_ Phenytoin is not
20
associated with the dysmorphic facies seen in this patient
21
• 22
(Choice F) In addition to developmental delay, congenital rubella syndrome is characterized by sensorineural
• 23
deafness, cataracts, hepatosplenomegaly, and purpura.
• 24
• 25 Educational objective:
• 26 Fetal alcohol syndrome is one of the most common preventable causes of birth defects, behavioral problems,
• 27 and cognitive impairment The pathognomonic facial features are small palpebral fissures, smooth philtrum,
• 28 and thin vermilion border_
• 29
• 30
References:
• 31
• 32 1. A practical clinical approach to diagnosis of fetal alcohol spectrum disorders: clarification
• 33 of the 1996 Institute of Medicine criteria.
▪ 34
2. American Academy of Pediatrics. Committee on Substance Abuse and Committee on
• 35 Children With Disabilities. Fetal alcohol syndrome and alcohol-related
• 36 neurodevelopmental disorders.
• 37
• 38
• 39
Copyright © LIWorld Last updated: [16/11/2014]
• 40
• 41
• 42 End Block
Feedback
Item: 5 of 44 .11M Mark
Previous Next Lab Values Notes Calculator
Media Exhibit

Fetal alcohol syndrome


7
8 Fetal alcohol syndrome
9
• 10
• -1 -1
• 12
• 13
• 14
Microcephaly
• 15
• 16
• 17
• 19
• 19 Small
20 palpebral
21 fissures
• 22
• 23
• 24
• 25
• 26 Smooth
• 27
phi ltrum
• 28
• 29
• 30 Thin
• 31
vermilion
• 32
• 33
border
• 34
• 35
• 36
• 37 V
• 39
• 39
• 40
• 41
• 42 End Block
Feedback
- 1
2 Item: 6 of 44 F Mark
▪ 3 Previous Next Lab Values. Notes Calculator.
- 4
5
6 A 13-year-old boy is brought to the physician for evaluation of a limp. He has had mild left hip pain for several
7 weeks_ After the boy fell off his bicycle today, his mother noticed that he was limping and brought him to the
8 office. His body mass index is 31 kg/m2. He walks with a limp to the examination table, favoring his right leg.
9 As he sits down, his left leg is rotated externally. There is decreased range of motion and pain with internal
• 10 rotation of his left hip. There is no leg length discrepancy_ Bilateral anteroposterior and frog-leg lateral x-rays
• 11 of both hips are obtained. The frog-leg lateral view is shown below. What is the most likely diagnosis?
• 12
• 13
• 14
• 15
• 16
• 17
• 18
• 19
20
21
• 22
• 23
• 24
• 25
• 26
• 27
• 28
• 29
• 30
• 31
• 32
• 33
. 34
• 35
• 36
• 37
• 38
• 39
• 40
se
• 41
• 42 End Block
Feedback
1
2 Item: 6 of 44 F Mark
3 Previous Next Lab Values. Notes Calculator.
4

A. Avascular necrosis of the femoral head [5%]

▪ 8 B. Chronic developmental hip dysplasia [2%]


▪ 9 C. Femoral neck stress fracture [4%]
• 10
ID. Legg-Calve-Perthes disease [9*]
• 11
• 12 E Slipped capital femoral epiphysis [19%]
• 13
• 14
Explanation:
• 15
• 16 Normal Slipped capital femoral epiphysis
• 17
• 18 Stable Unstable
• 19
20
21
• 22
• 23
• 24
• 25
• 26
• 27
• 28
• 29
• 30
• 31
• 32
• 33
• 34
• 35 1J5.141LEWorld, CLC © .2011

• 36
• 37 This patient has slipped capital femoral epiphysis (SCFE), displacement of the capital femoral epiphysis from
• 38 the femoral neck_ This typically presents in obese children age 10-16 years. Adolescent boys are affected
• 39 slightly more often than girls_ Additional risk factors include endocrinopathies (eg, hypothyroidism, growth
• 40 hnrmnna elafir Ira anri rariiatinn hictnnr rhilriran inrith anrinr rine-Nni-hiac- Imr1c.1- In.mra

• 41
• 42 End Block
Feedback
[1:=
• 2 Item: 6 of 44 F'Mark
• 3 Previous Next . Lab Values. Notes Calculator.

- 4 I A

- 5 Normal Slipped capital femoral epiphysis

• 7 Stable Unstable

• 8
• 9
• 10
• 11
• 12
• 13
• 14
• 15
• 16
• 17
• 18
• 19
20
21
• 22
• 23
US.PALEMiid, CLC MI 1
• 24
• 25
• 26 This patient has slipped capital femoral epiphysis (SCFE), displacement of the capital femoral epiphysis from
• 27 the femoral neck_ This typically presents in obese children age 1G-16 years. Adolescent boys are affected
• 28 slightly more often than girls_ Additional risk factors include endocrinopathies (eg, hypothyroidism, growth
• 29 hormone deficiency), renal failure, and radiation history. Children with endocrinopathies almost always have
• 30 bilateral disease and present at an earlier age_ Patients classically present with an insidious onset of dull hip
• 31 or referred knee pain and altered gait with no preceding trauma. Minor trauma, as in this patient, can
• 32 sometimes exacerbate the pain and bring the patient to medical attention. On examination, patients tend to
• 33 hold the affected hip in passive external rotation and exhibit decreased internal rotation, abduction, and
• 34 flexion. Diagnosis is made with plain radiographs of the hip (anteroposterior and frog-leg lateral views), which
• 35 show the posteriorly and inferiorly displaced femoral head. Both hips should be imaged for comparison and
▪ 36 to assess for contralateral displacement. The gold standard treatment is immediate surgical screw fixation
• 37 at the current degree of slippage to avoid the risk of avascular necrosis (AWN).
▪ 38
(Choice A) AVN of the femoral head can be a complication of SCFE as the displaced femoral head can
• 39
disrupt the blood supply.
• 40
• 41
• 42 End Block
Feedback
A
- 1
2 Item: 6 of 44 I'Mark
▪ 3 Previous Next . Lab Values, Notes Calculator.
- 4 slightly more often than girls. Additional risk factors include endocrinopathies (eg. hypothyroidism, growth
5 hormone deficiency). renal failure. and radiation history. Children with endocrinopathies almost always have
. 6 bilateral disease and present at an earlier age. Patients classically present with an insidious onset of dull hip
7 or referred knee pain and altered gait with no preceding trauma. Minor trauma. as in this patient. can
• 8 sometimes exacerbate the pain and bring the patient to medical attention. On examination, patients tend to
• 9 hold the affected hip in passive external rotation and exhibit decreased internal rotation.. abduction, and
• 10 flexion. Diagnosis is made with plain radiographs of the hip (anteroposterior and frog-leg lateral views), which
• 11 show the posteriorly and inferiorly displaced femoral head. Both hips should be imaged for comparison and
• 12 to assess for contralateral displacement. The gold standard treatment is immediate surgical screw fixation
• 13 at the current degree of slippage to avoid the risk of avascular necrosis (AVN).
• 14
• 15 (Choice A) AVN of the femoral head can be a complication of SCFE as the displaced femoral head can
• 16 disrupt the blood supply.
• 17
• 18 (Choice B) Developmental hip dysplasia is caused by abnormal development of the hip in utero. It is usually
• 19
detected on the newborn physical examination but can be diagnosed at a later age when a limp is noted. Leg
length discrepancy is present and radiographs show a poorly formed femoral head. making this diagnosis
20
21
unlikely.
• 22
(Choice C) Femoral neck stress fractures occur most commonly in runners or other athletes doing
• 23
extensive training. Patients typically present with a gradual increase in hip pain, especially with activity. They
• 24
also have pain with passive range of motion of the hip. especially internal and external rotation.

(Choice ID) Legg-Calve-Perthes disease is a syndrome of idiopathic AVN of the hip that most commonly
Fl.

• 25
27
affects boys age 5-7 years.
• 28
• 29 Educational objective:
. 30 Slipped capital femoral epiphysis is a common hip disorder seen in overweight adolescents. Urgent surgical
. 31 fixation is required to prevent avascular necrosis of the hip_
• 32
.
References:
• 34
. 35 1. Slipped capital femoral epiphysis update.
• 36 2. Changing incidence of slipped capital femoral epiphysis: a relationship with obesity?
.~
• 38
• 39
Copyright © UWorld Last updated: [1/6/2015]
• 40
• 41
• 42 0
Feedback End Block
1
2 Item: 6 of 44 Mark
3 Previous Next Lab Values Notes Calculator
4 Media Exhibit
5
6 Slipped uspitslfemoral epiphysis
7
8
9
• 10
• -1-1
• 12
• 13
• 14
• 15
• 16
• 17
• 19
• 19
20
21
• 22
• 23
• 24
• 25
• 26
• 27
• 28
• 29
• 30
• 31
• 32
• 33
• 34
• 35
• 36
• 37
• 39
• 39
• 40


41
42
'I Feedback
a
End Block
A
• 2 Item: 6 of 44 .111M Mark
▪ 3 Previous Next Lab Values Notes Calculator
- 4 141edia Exhibit
- 5
MRI occult hip frachffe
• 7
▪ 8
▪ 9
• 10
• -1 -1
• 12
• 13
• 14
• 15
• 16
• 17
• 19
• 19
20
21
• 22
• 23
• 24
• 25
• 26
• 27
• 28
• 29
• 30
• 31
• 32
• 33
• 34
• 35
• 36
• 37
• 39
• 39
• 40
• 41
• 42 End Block
Feedback
▪ 2 Item: 6 of 44 .111M Mark
11
.

▪ 3 Previous Next Lab Values Notes Calculator


- 4 141edia Exhibit A
- 5

• 7
▪ 8
▪ 9
• 10
• -1 -1
• 12
• 13
• 14
• 15
• 16
• 17
• 19
• 19
20
21
• 22
• 23
• 24
• 25
• 26
• 27
• 28
• 29
• 30
• 31
• 32
• 33
• 34
• 35
• 36
• 37 a
• 39
• 39
• 40
• 41
• 42 End Block
Feedback
1
2 Item: 6 of 44 .111M Mark
EF.

3 Previous Next Lab Values Notes Calculator


4 Media Exhibit

X-ray Legg-Calve-Perthes disease

▪ 8
▪ 9
• 10
• 11
• 12
• 13
• 14
• 15
• 16
• 17
• 19
• 19
20
21
• 22
• 23
• 24
• 25
• 26
• 27
• 28
• 29
• 30
• 31
• 32
• 33
• 34
• 35
• 36
• 37
• 39
• 39
• 40


41
42
'1 Feedback
a
End Block
1
2 Item: 6 of 44 .111 II Mark
EF.

3 Previous Next Lab Values Notes Calculator


4 Media Exhibit

X-ray Legg-Calve-Perthes disease

▪ 8
▪ 9
• 10
• 11
• 12
• 13
• 14
• 15
• 16
• 17
• 19
• 19
20
21
• 22
• 23
• 24
• 25
• 26
• 27
• 28
• 29
• 30
• 31
• 32
• 33
• 34
• 35
• 36
• 37 V
• 39
• 39
j
• 40
• 41
• 42
Item: 7 of 44 V-• Mark
Previous Next Lab Values. Notes Calculator.

11.

A 4-year-old boy is brought to the physician for a routine health maintenance visit He will be starting
preschool soon, and his parents want to make sure that he is in optimal health_ The patient is toilet trained for
day and night. He eats a well-balanced diet and takes no medications. The patient has a history of acute
9 otitis media that occurred twice during infancy. He lives with his parents in a suburban house built in 1981_
• 10 The patient's father has a history of high blood pressure and quit smoking cigarettes before the boy was born.
• 11 Examination shows an active, cooperative boy who speaks in full sentences clearly_ He can balance and hop
• 12 on one foot Which of the following is the most appropriate next step in this patient's primary care?
• 13
• 14 1
• 15
A. Meningococcal vaccination [14%]
• 16 B. Monocular visual acuity test [48%]
• 17
C. Rotavirus vaccination [16°A]
• 18
• 19
D. Serum lead level [20%]
20 E Serum lipid panel [113.6]
21
F. Ultrasound for inguinal hernias [0%]
• 22
• 23
• 24 Explanation:
• 25
• 26 Vision assessment should be performed at every well-child visit as untreated eye abnormalities during the first
• 27 few years of life can lead to permanent vision loss. During infancy, visual behavior is assessed by
• 28 observation fixation and tracking. The cover-uncover test can be performed in older infants and children to
• 29 assess for strabismus. Monocular visual acuity assessment by Snellen letters or numbers or the
• 30 tumbling E chart can begin at age 3. Visual acuity worse than 20/40 at age 3-5 or worse than 20/30 at age
• 31 >6 should prompt ophthalmologic evaluation for refractive errors. Abnormal red reflexes, misalignment,
• 32 pupillary asymmetry of mm, corneal asymmetry, ptosis or other lesions obstructing the visual axis, and
• 33 nystagmus are additional indications for referral.
• 34
• 35 (Choice A) The quadrivalent meningococcal vaccination should be administered at age 11-12 followed by a
• 36 booster dose at age 16 due to the risk of college outbreaks. Children with asplenia, HIV, and complement
• 37 deficiency may be vaccinated as early as age 2. This patient is at low risk for meningococcal disease and
• 38 does not require vaccination.
• 39
(Choice C) The rotavirus vaccine is recommended at age 2-8 months as infants are at greatest risk for V
• 40
• 41
• 42 End Block
Feedback
Item: 7 of 44 F Mark
Previous Next Lab Values. Notes Calculator.
Explanation:

Vision assessment should be performed at every well-child visit as untreated eye abnormalities during the first
few years of life can lead to permanent vision loss. During infancy, visual behavior is assessed by
observation fixation and tracking. The cover-uncover test can be performed in older infants and children to
9 assess for strabismus. Monocular visual acuity assessment by Snellen letters or numbers or the
• 10 tumbling E chart can begin at age 3. Visual acuity worse than 20/40 at age 3-5 or worse than 20/30 at age
• 11 >6 should prompt ophthalmologic evaluation for refractive errors. Abnormal red reflexes, misalignment,
• 12 pupillary asymmetry of mm, corneal asymmetry, ptosis or other lesions obstructing the visual axis, and
• 13 nystagmus are additional indications for referral.
• 14
• 15 (Choice A) The quadrivalent meningococcal vaccination should be administered at age 11-12 followed by a
• 16 booster dose at age 16 due to the risk of college outbreaks. Children with asplenia, HIV, and complement
• 17 deficiency may be vaccinated as early as age 2. This patient is at low risk for meningococcal disease and
• 18 does not require vaccination.
• 19
(Choice C) The rotavirus vaccine is recommended at age 2-8 months as infants are at greatest risk for
20
dehydration from severe gastroenteritis_ It is not administered past this age range.
21
• 22
(Choice 0) Early diagnosis and treatment of lead toxicity are important due to the risk for microcytic anemia
• 23
and permanent neurologic damage. Risk factors for environmental lead exposure include minority race or
• 24
ethnicity, low socioeconomic status, pre-1978 housing, occupational exposure from parents (eg, pottery), and
• 25 1
recent immigration. Screening is not recommended in asymptomatic children with no risk factors.
• 26
• 27 (Choice E) Early identification of lipid disorders may decrease the development of atherosclerotic vascular
• 28 disease_ Universal screening for dyslipidemia is recommended at age 9-11 and again at age 17-21 as lipid
• 29 levels are relatively stable just prior to and after puberty. Otherwise, screening is limited to patients at high
• 30 risk for cardiovascular disease (eg, history of obesity, diabetes mellitus, and/or tobacco exposure; family
• 31 history of premature coronary disease) and men age
• 32
• 33 (Choice F) Inguinal hernias can cause inguinal/scrotal swelling and pain and are usually evidenced on
• 34 physical examination. An ultrasound may be performed if the etiology of groin pathology is
• 35 unclear. Ultrasound examination of asymptomatic patients is not indicated_
• 36
• 37 Educational objective:
• 38 Vision development is critical during the first few years of life, and screening should occur at every well-child
• 39 examination. Infants should be evaluated by observing fixation and tracking. The cover-uncover test should
be Der-formed in older infants and children to assess for strabismus. Visual acuity testing should beain at age V
• 40
• 41
• 42 End Block
Feedback
Item: 7 of 44 F' Mark U.P
Previous Next Lab Values. Notes I Calculator.
1,,,, /III LIIx LI 14-.1-1 411 =I 111111, 4.1- 1,111 1,-L11 L-1.,y1111 114. 41 y 1 11 1 1,--,1,1 11--, 11.01,-,11.144,r11.111 LI I,- If 4111,1
A
nystagmus are additional indications for referral.

(Choice A) The quadrivalent meningococcal vaccination should be administered at age 11-12 followed by a
booster dose at age 16 due to the risk of college outbreaks. Children with asplenia, HIV, and complement
deficiency may be vaccinated as early as age 2. This patient is at low risk for meningococcal disease and
does not require vaccination.
• 10
• 11 (Choice C) The rotavirus vaccine is recommended at age 2-8 months as infants are at greatest risk for
• 12 dehydration from severe gastroenteritis_ It is not administered past this age range.
• 13
• 14 (Choice ID) Early diagnosis and treatment of lead toxicity are important due to the risk for microcytic anemia
• 15 and permanent neurologic damage. Risk factors for environmental lead exposure include minority race or
• 16 ethnicity, low socioeconomic status, pre-1978 housing, occupational exposure from parents (eg, pottery), and
• 17 recent immigration. Screening is not recommended in asymptomatic children with no risk factors.
• 18
• 19 (Choice E) Early identification of lipid disorders may decrease the development of atherosclerotic vascular
20 disease_ Universal screening for dyslipidemia is recommended at age 9-11 and again at age 17-21 as lipid
21 levels are relatively stable just prior to and after puberty. Otherwise, screening is limited to patients at high
• 22 risk for cardiovascular disease (eg, history of obesity, diabetes mellitus, and/or tobacco exposure; family
• 23 history of premature coronary disease) and men age
• 24
• 25 (Choice F) Inguinal hernias can cause inguinal/scrotal swelling and pain and are usually evidenced on
• 26 physical examination. An ultrasound may be performed if the etiology of groin pathology is
• 27 unclear. Ultrasound examination of asymptomatic patients is not indicated_
• 28
Educational objective:
• 29
Vision development is critical during the first few years of life, and screening should occur at every well-child
• 30
examination. Infants should be evaluated by observing fixation and tracking. The cover-uncover test should
• 31
be performed in older infants and children to assess for strabismus. Visual acuity testing should begin at age
• 32
3 with the tumbling E or Snellen chart
• 33
• 34
• 35 References:
• 36 1. Screening for visual impairment in children ages 1-5 years: update for the USPSTF.
• 37
• 38
• 39
Copyright © UWorld Last updated: [12/8/2014]
• 40
• 41
• 42 End Block
Feedback
Item: 7 of 44 ridlark
Previous Next Lab Values Notes Calculator
Media Exhibit
5
Disconjugate gaze

9
• 10
• -1 -1
• 12
• 13
• 14
• 15
• 16
• 17
• 19
• 19
20
21
• 22
• 23
• 24
• 25
• 26
• 27
• 28
• 29
• 30
• 31
• 32
• 33
• 34
• 35
• 36
• 37
• 39
• 39
• 40
• 41
• 42 End Block
Feedback
-1

2 Item: 7 of 44 'Mark
3 Previous Next Lab Values Notes Calculator
4- Media Exhibit
5
6 Red reflex and corneal Ii4Wtion
7
8
9 Red reflex & corneal light reflection
• 10
• 11
• 12
• 13
• 14
• 15
• 16
• 17
• 19
• 19
20
21
• 22
• 23
• 24
• 25
Normal eyes
• 26 Red reflexes & corneal light reflexes are equal
• 27
• 28
• 29
• 30
• 31
• 32
• 33
▪ 34
• 35
• 36
• 37 V
• 39
• 39
• 40
• 41
• 42 End Block
Feedback
-1

2 Item: 7 of 44 Al M 'Mark
3 Previous Next Lab Values Notes Calculator
4- Media Exhibit
5
6 Red reflex and corneal light reflection
7
8
9
• 10
• -1-1
• 12
• 13
• 14
• 15
Normal eyes
• 16 Red reflexes & corneal light reflexes are equal
• 17
• 19
• 19
20
21
re-
• 22
• 23
• 24
• 25
• 26
• 27
• 28
• 29
• 30
• 31
Absent reflex
• 32
• 33 White reflex on abnormal eye can result from opacities of the lens (eg,
• 34 cataract), or tumor (eg: retinoblastorna)
• 35
• 36
• 37 V
• 39
• 39
• 40
• 41
• 42 End Block
Feedback
-1

2 Item: I of 44 .11M Mark


3 Previous Next Lab Values Notes Calculator
4- Media Exhibit
5
6 Red reflex and corneal light reflection
7
8
9
• 10
• -1-1
• 12
• 13
Absent reflex
• 14
• 15 White reflex on abnormal eye can result from opacities of the lens (eg,
• 16 cataract), or tumor (eg, retinoblestonna)
• 17
• 19
• 19
20
21
• 22
• 23
• 24
• 25
• 26
• 27
• 28
• 29
• 30
• 31
• 32
Asymmetric reflections
• 33 In strabismus, the red reflex is more intense in the deviated eye_ The
• 34 corneal light reflexes are also asymmetric.
• 35
• 36 01.11$101U.WorIci, L _C
• 37
• 39
• 39
• 40
• 41
• 42 End Block
Feedback
1
2 Item: 8 of 44 V-• Mark
3 Previous Next Lab Values. Notes Calculator.
4-
5
6 A 9-year-old girl is seen in the office for unsteady gait and weakness in the lower limbs. On examination she is
7 found to have a wide-based gait with constant shifting of position to maintain her balance. There is decreased
8 vibratory and position sense in the lower extremities, and ankle jerks are absent bilaterally. Her feet are
9 deformed with high plantar arches_ MRI of the brain and spinal cord shows marked atrophy of the cervical
• 10 spinal cord and minimal cerebellar atrophy. Nerve conduction velocity results are within normal limits. EKG
• 11 shows T-wave inversions in the inferior and lateral chest leads. The mother says that her other child, a
• 12 3-year-old boy, is completely normal. He shares toys with his sister and plays well with other children at
• 13 playschool. He is able to copy squares and crosses but is unable to copy a triangle. Which of the following
• 14 statements would you advance while discussing the girl's condition with her mother?
• 15
• 16
• 17
A. There is nothing to worry about since it's a non-progressive condition [5%]
• 18 R. It is most likely a result of spontaneous mutation & the chances of your boy developing the same
• 19 condition are minimal [34%]
20 C. The EKG abnormalities are most likely due to myocardial ischemia [6%]
21
• 22
ID. The inability of your boy to copy a triangle worries me since it may be an early manifestation of the
• 23
same condition your daughter is suffering from [13%]
• 24 ▪ E. I advise you to go for prenatal counseling if you desire any future pregnancies [41%]
• 25
• 26
• 27 Explanation:
• 28
The physical findings and MRI features described in the question classically fit into the description of
• 29
Friedreich Ataxia (FA)_ It is an autosomal recessive condition characterized by an excessive number of
• 30
trinucleotide repeat sequences, resulting in an abnormality of a tocopherol transfer protein. The disorder is
• 31
progressive with poor prognosis (Choice A). Most patients are wheelchair bound by the age of 25, with death
• 32
occurring by 30-35 years of age_
• 33
• 34
FA is associated with necrosis and degeneration of cardiac muscle fibers leading to myocarditis, myocardial
• 35
fibrosis and cardiomyopathy. Cardiac arrhythmia and congestive heart failure contribute to a significant
• 36
number of deaths. T-wave inversion in this case is most likely due to myocarditis, and not myocardial
• 37
infarction (Choice Cy [Remember the differential for T-wave inversion: myocardial infarction, myocarditis, old
• 38
pericarditis, myocardial contusion and digoxin toxicity]_
• 39
• 40 o ......,r

• 41
• 42 End Block
Feedback
1
2 Item: 8 of 44 F' Mark
3 Previous Next Lab Values. Notes Calculator.
4- A. There is nothing to worry about since it's a non-progressive condition [5%]
5
B. It is most likely a result of spontaneous mutation & the chances of your boy developing the same
6
condition are minimal [34%]
7
8 C. The EKG abnormalities are most likely due to myocardial ischemia [6%]
9 ID. The inability of your boy to copy a triangle worries me since it may be an early manifestation of the
• 10 same condition your daughter is suffering from [13%]
• 11
E. I advise you to go for prenatal counseling if you desire any future pregnancies [41%]
• 12
• 13
• 14 Explanation:
• 15
• 16 The physical findings and MRI features described in the question classically fit into the description of
• 17 Friedreich Ataxia (FA)_ It is an autosomal recessive condition characterized by an excessive number of
• 18 trinucleotide repeat sequences, resulting in an abnormality of a tocopherol transfer protein. The disorder is
• 19 progressive with poor prognosis (Choice A). Most patients are wheelchair bound by the age of 25, with death
20 occurring by 30-35 years of age_
21
• 22 FA is associated with necrosis and degeneration of cardiac muscle fibers leading to myocarditis, myocardial
• 23 fibrosis and cardiomyopathy. Cardiac arrhythmia and congestive heart failure contribute to a significant
• 24 number of deaths. T-wave inversion in this case is most likely due to myocarditis, and not myocardial
• 25 infarction (Choice Cy [Remember the differential for T-wave inversion: myocardial infarction, myocarditis, old
• 26 pericarditis, myocardial contusion and digoxin toxicity].
• 27
• 28 The milestones of the 3-year-old boy are normal and ability to copy a triangle develops after the age of 5 years
• 29 (Choice 0). [3 years- copies a cross and circle, 4 years- copies a square and rectangle, 5 years- copies a
• 30 triangle, 6 years- copies a diamond]
• 31
In families with one affected child, the subsequent risk of another affected child is 25%, which is significant
• 32
(Choice BY Genetic counseling is recommended for prenatal diagnosis for parents with one affected child
• 33
(Choice Ey
• 34
• 35
Educational Objective:
• 36
Friedreich Ataxia is an autosomal recessive condition and genetic counseling is recommended for prenatal
• 37
diagnosis for parents with one affected child.
• 38
• 39
Copyright © UWorld Last updated: [9/30/2014]
• 40
• 41
• 42 End Block
Feedback
• 1
• 2 Item: 9 of 44 F' Mark
• 3 Previous Next Lab Values. Notes Calculator.
• 4-
• 5
• 6 A 5-year-old boy is brought to the emergency department with blurred vision, right eye pain, and headache_
• 7 His past medical history includes otitis media, dental caries, eczema, and allergic rhinitis. He is allergic to
insect venom. His temperature is 39.4' C (103° F), blood pressure is 100/70 mm Hg, pulse is 120/min, and
respirations are 20imin. Examination shows conjunctival injection, pain with extraocular movements,
• 10 periorbital edema, limited adduction, and proptosis of the right eye. The left eye is normal. Which of the
• 11 following is the most common predisposing factor for this child's condition?
• 12
• 13
• 14
▪ A. Bacterial sinusitis [55%]
• 15 Dacryocystitis [4%]
• 16
C. Dental abscess [7%]
• 17
• 18
D. Hordeolum [21.6]
• 19 E. Impetigo [1%]
20 F. Insect bite [11 %]
21
• 22
G. Orbital trauma [12%]
• 23 H. Otitis media [7%]
• 24
I. Strabismus surgery [156]
• 25
• 26
• 27 Explanation:
• 28
• 29 Orbital cellulitis is an infection of the orbital soft tissue posterior to the orbital septum. This patient has pain
• 30 with eye movements, proptosis, ophthalmoplegia, and diplopia, consistent with the diagnosis of orbital
• 31 cellulitis_ Bacterial sinusitis is the most common predisposing factor for orbital cellulitis due to the prevalence
• 32 of this disease as well as the proximity of the sinuses to the orbital space and the valveless orbital venous
• 33 system.
• 34
• 35
• 36
The valveless ophthalmic venous system
• 37
• 38
• 39 Lacrimal vein
• 40
• 41
• 42 End Block
Feedback
1
• 2 Item: 9 of 44 V- Mark
• 3 Previous Next Lab Values. Notes Calculator.
• 4-
• 5
Orbital cellulitis is an infection of the orbital soft tissue posterior to the orbital septum. This patient has pain
6
with eye movements, proptosis, ophthalmoplegia, and diplopia, consistent with the diagnosis of orbital
7
cellulitis_ Bacterial sinusitis is the most common predisposing factor for orbital cellulitis due to the prevalence
of this disease as well as the proximity of the sinuses to the orbital space and the valveless orbital venous
system.
• 10
• 11
The valveless ophthalmic venous system
• 12
• 13
• 14
• 15
Lacrimal vein
• 16
• 17
• 18
Superior ophthalmic vein
• 19
20
21
• 22
• 23
• 24
• 25
• 26 Cavernous sinus
• 27
• 28
• 29
• 30
• 31
• 32
• 33
• 34
• 35
• 36
• 37
• 38
• 39
• 40
se
• 41
• 42 End Block
Feedback
• 1
• 2. Item: 9 of 44 F'Mark
▪ 3
3 Previous Next Lab Values. Notes Calculator.
• 4
▪ 5
▪ 6 7
▪ 7

• 10
• 11
• 12
• 13
• 14
• 15
• 16
Inferior Posterior Anterior Nasofrontal
• 17
ophthalmic ethrnoidal ethmoidar vein
• 18
• 19 vein vein vein
USMLEWorkl, LLC
20
21
Dangerous complications of orbital cellulitis include blindness, subperiosteal abscesses, cavernous sinus
• 22
thrombosis, intracranial infection, and death. Contrast-enhanced computed tomography can be used to
• 23
identify abscesses requiring surgery_
• 24
• 25
(Choices B, C, 13, E, F, G, H, I) Less common risk factors for orbital cellulitis include dacryocystitis, dental
• 26
abscess, hordeolum, impetigo, insect bite, orbital trauma, otitis media, and ophthalmic surgery (eg,
• 27
strabismus repair).
• 28
• 29 Educational objective:
• 30 Bacterial sinusitis is the most common predisposing factor for orbital cellulitis. Symptoms of orbital cellulitis
• 31 include proptosis, ophthalmoplegia, and diplopia.
• 32
• 33
References:
• 34
• 35 1. Orbital complications of sinusitis in children.
• 36 2. Periorbital and orbital cellulitis.
• 37
• 38
• 39
Copyright @ UWorld Last updated: [10/7/2014]
• 40
• 41
• 42 End Block
Feedback
2 Item: 9 of 44 .111M Mark
-4(1 M
ora

3 Previous Next Lab Values Notes Calculator


4- 141edia Exhibit
5
6 cellulitis
7
8
9
• 10
• -1 -1
• 12
• 13
• 14
• 15
• 16
• 17
• 19
• 19
20
21
• 22
• 23
• 24
• 25
• 26
• 27
• 28
• 29
• 30
• 31
• 32
• 33
• 34
• 35
• 36
• 37
• 39
• 39
40
V

• 41
• 42 End Block
Feedback
2 Item: 9 of 44 .111M Mark
M
ora

• 3 Previous Next Lab Values Notes Calculator


4- 141edia Exhibit A
▪• 5
6 cellulitis
• 7
8
9
• 10
• -1 -1
• 12
• 13
• 14
• 15
• 16
• 17
• 19
• 19
20
21
• 22
• 23
• 24
• 25
• 26
• 27
• 28
• 29
• 30
• 31
• 32
• 33
• 34
• 35
• 36
• 37
• 39
• 39
• 40
• 41
• 42 End Block
4
V Feedback
1
2 Item: 10 of 44 F' Mark
3 Previous Next Lab Values. Notes Calculator.

4-
5
6 A 2-year-old child is brought to the office due to vomiting. He is recovering from an upper respiratory
7 infection, which was treated by his mother with over-the-counter aspirin_ an examination he is irritable,
8 lethargic, agitated and uncooperative. His height, weight and head circumference are at the 50th percentile.
9 The pupils are equal and have a sluggish reaction to light The sclera is non-icteric. The neck is supple and
without adenopathy. The abdomen is flat, with normal bowel sounds. The lab findings are as follows:
• 11
Serum bilirubin Normal
• 12
Serum AST and ALT Increased
• 13
Serum ammonia Increased
• 14
Prothrombin time Prolonged
• 15
• 16
• 17
A CT scan of the brain shows cerebral edema. CSF analysis is normal, except for increased pressure. What
• 18
is the most likely diagnosis?
• 19
20 A. Aseptic meningitis [1%]
21
• 22
B. Reye syndrome [94%]
• 23
• 24
• 25 C. Carnitine deficiency [1%]
• 26
• 27 ID. Sepsis [11%]
• 28
• 29
E Viral encephalitis [2%]
• 30
• 31
• 32
• 33 Explanation:
• 34
• 35 Reye syndrome is a rare illness seen exclusively in children less than 15 years old who were treated with
• 36 salicylates for a viral infection_ The common presentation is as mentioned above: vomiting, agitation, and
• 37 irrational behavior, progressing to lethargy, stupor, and restlessness_ Convulsions may occur. Characteristic
• 38 laboratory findings include hyperammonemia, normal or slightly elevated levels of bilirubin and alkaline
• 39 phosphatase, prolonged prothrombin time, hypoglycemia, and moderate to severe elevations in AST, ALT,
• 40 nell2ntA-e riehvrirnnen2se levels Rinnsv of the liver kidneys and hmin reve.2Is minrnvesimilAr ste2tnsis v
• 41
• 42 End Block
Feedback
1
2 Item: 10 of 44 VI/lark
3 Previous Next Lab Values. Notes Calculator.
U. ...Del.i L 1 70]
4-
5
6 E. Viral encephalitis [2%]
7
8
9
Explanation:
• 11
Reye syndrome is a rare illness seen exclusively in children less than 15 years old who were treated with
• 12
salicylates for a viral infection_ The common presentation is as mentioned above: vomiting, agitation, and
• 13
irrational behavior, progressing to lethargy, stupor, and restlessness_ Convulsions may occur. Characteristic
• 14
laboratory findings include hyperammonemia, normal or slightly elevated levels of bilirubin and alkaline
• 15
phosphatase, prolonged prothrombin time, hypoglycemia, and moderate to severe elevations in AST, ALT,
• 16
and lactate dehydrogenase levels. Biopsy of the liver, kidneys and brain reveals microvesicular steatosis.
• 17
Treatment is supportive.
• 18
• 19
(Choice A) Even though bacterial/viral meningitis can present with mental changes, the absence of neck
20
rigidity and the normal CSF study makes this diagnosis unlikely. Aseptic meningitis alone will not have
21
elevated liver enzymes_
• 22
• 23
(Choice C) Systemic carnitine deficiency (SCD) is an inherited defect in fatty acid oxidation. The clinical
• 24
picture of SLID is somewhat similar to that of Reye Syndrome (i.e., hypoglycemia, hyperammonemia,
• 25
hypoprothrombinemia, and acute episodes of encephalopathy); however, in the former, the acyl-carnitine
• 26
levels are elevated, whereas these are normal in the latter. In this case, the classic presentation following a
• 27
viral illness and aspirin ingestion makes Reye syndrome the most likely diagnosis.
• 28
• 29
(Choice 0) Sepsis in children can present with variable symptoms such as nonspecific mental changes,
• 30
irritability, and lethargy. Even though they can present with a normal body temperature, they are usually much
• 31
sicker than the above described patient, and signs of diminished perfusion such as delayed capillary refill,
• 32
weak peripheral pulses, and cool extremities are usually present Tachypnea and tachycardia are common
• 33
signs of sepsis.
• 34
• 35
Educational Objective:
• 36
Salicylates are contraindicated in young children with viral infections. Recognize the clinical presentation of
• 37
Reye syndrome.
• 38
• 39
Copyright © UWorld Last updated: [8/22/2014]
• 40 1
• 41
• 42 End Block
Feedback
• 1
• 2 Item: 11 of 44 F' Mark U.P
▪ 3 Previous Next . Lab Values, Notes Calculator.
• 4-
▪ 5
▪ 6 A 6-hour-old boy is in the newborn nursery with respiratory distress_ He was born at 39 weeks gestation by
▪ 7 uncomplicated vaginal delivery_ The pregnancy was complicated by pre-eclampsia during the 3rd trimester_
▪ 8 The mother had an elevated 1-hour glucose tolerance screening, but her 3-hour glucose tolerance test was
▪ 9 normal. The infant weighs 1.9 kg (4 lb 4 oz) (<3rd percentile). He has not been able to feed due to
• 10 tachypnea_ Temperature is 372 C (9 F), blood pressure is 60/34 mm Hg, pulse is 144/min, and respirations
are 68/min. Pulse oximetry shows 96% saturation on room air. On examination, the infant's skin is plethoric
• 12 and he has mild cyanosis of his lips and tongue. No cardiac murmur is heard. He has a normal Moro reflex
• 13 and normal tone. Examination is otherwise normal. Chest radiograph shows clear lung fields with a normal
• 14 cardiac silhouette. Initial laboratory results are as follows:
• 15
Complete blood count
Hemoglobin 22.8 g/dL
• 17
Hematocrit 69%
• 18
Platelets 2113,000/pL
• 19
Leukocytes 5,504 L
20
Neutrophils 56%
21
Lymphocytes 33%
• 22
Glucose 50
• 23
• 24
Which of the following is the most likely cause of this patient's respiratory distress?
• 25
• 26
• 27 A. Cyanotic heart disease [6%]
• 28 B. Dehydration [5%]
• 29
C. Hyperinsulinism [12%]
• 30
• 31 D. Polycythemia [53%]
• 32 E Transient tachypnea of the newborn [24%]
• 33
• 34
• 35 Explanation:
• 36
• 37
• 39 Neonatal polycythemia
• 39
V
• 40
• 41
• 42 End Block
Feedback
• -1
• 2 Item: 11 of 44 F'Mark
▪ 3 Previous Next Lab Values. Notes Calculator.

• 4-
▪ 5
▪ 6 Neonatal polycythemia
▪ 7
▪ 8
Definition Hematocrit >65% in term infants
▪ 9
• 10
Increased erythropoiesis from intrauterine hypoxia
• 12
• Maternal diabetes
• 13
• Maternal hypertension
• 14
• 15
• Smoking
Causes • Intrauterine growth restriction
• 17 Erythrocyte transfusion
• 19
• 19 • Delayed cord clamping
20 • Twin-twin transfusion
21
• 22
• 23
• Ruddy skin
• 24 • Hypoglycemia
• 25 Clinical • Respiratory distress
• 26 presentation • Cyanosis
• 27 • Apnea, irritability, Redness
• 28
• Abdominal distension
• 29
• 30
• 31 Treatment Partial exchange transfusion (remove blood, infuse normal saline)
• 32 OUSFALEViorldr LLC
• 33
• 34 Polycythemia is defined as a hematocrit >85% in term neonates. A common cause of neonatal polycythemia
• 35 is delayed clamping of the umbilical cord, resulting in excess transfer of placental blood. Other predisposing
• 36 conditions include in-utero hypoxia (eg, maternal hypertension, smoking) or poor placental gas exchange
• 37 (ie, maternal diabetes). In this patient, maternal pre-eclampsia has led to poor placental function, which in
• 39 turn has caused intrauterine growth restriction and neonatal polycythemia.
• 39
• 40 Mnst nenntps with nnlve_vthemia ars nther than arrrrearinn nifirivinIpthnrie Hnwever a the se
• 41
• 42 End Block
Feedback
• 1
• 2 Item: 11 of 44 F Mark U.P
▪ 3 Previous Next Lab Values, Notes Calculator.
• 4- Polycythemia is defined as a hematocrit >65% in term neonates. A common cause of neonatal polycythemia
▪ 5 is delayed clamping of the umbilical cord, resulting in excess transfer of placental blood. Other predisposing
▪ 6 conditions include in-utero hypoxia (eg, maternal hypertension, smoking) or poor placental gas exchange
▪ 7 (ie, maternal diabetes). In this patient, maternal pre-eclampsia has led to poor placental function, which in
▪ 8 turn has caused intrauterine growth restriction and neonatal polycythemia.
▪ 9
• 10 Most neonates with polycythemia are asymptomatic other than appearing ruddy/plethoric. However, as the
hematocrit rises, the viscosity of the blood increases and impairs blood flow to various organs. The most
• 12 common symptoms are lethargy, irritability, and jitteriness. Other potential manifestations are respiratory
• 13 distress, tachypnea, cyanosis, and poor feeding, as seen in this neonate. The increased red blood cell mass
• 14 can lead to hypoglycemia and hypocalcemia due to increased cellular uptake_
• 15
Asymptomatic neonates require only hydration by feeding or parenteral fluids. Symptomatic neonates
• 17 require partial exchange transfusion, in which blood is removed in exchange for normal saline to normalize the
• 18 hematocrit.
• 19
(Choice Al Cyanotic heart disease is unlikely in this patient The absence of a murmur or cardiomegaly is
20
reassuring, and the neonate has oxygen saturation of >95% on room air_ The cyanosis in this neonate is due
21
to the high red blood cell mass and hyperviscosity rather than a right-to-left shunt
• 22
• 23
(Choice B) Dehydration can cause elevated hematocrit levels. However. dehydration is very unusual in term
• 24
neonates in the first 2 days of life as they are born with excess extracellular water. Although this neonate has
• 25
not fed in 6 hours. it is unlikely that fluid loss could account for his symptoms.
• 26
• 27 (Choices C and El Respiratory distress syndrome is unlikely in this term neonate. Rarely, term neonates of
• 28 diabetic mothers may have surfactant deficiency, as high insulin levels block the maturation of sphingomyelin.
• 29 However, this mother did not have diabetes and her newborn has a normal chest x-ray. Transient tachypnea
• 30 of the newborn would be a more common cause of respiratory distress in the term neonate but should show
• 31 increased pulmonary vascular markings or fluid in the fissures on chest x-ray.
• 32
• 33 Educational objective:
• 34 Polycythemia is defined as a hematocrit >65% in term neonates. Risk factors include delayed cord clamping,
• 35 maternal hypertension, and maternal diabetes mellitus_ Symptomatic neonates generally have a combination
• 36 of respiratory distress, hypoglycemia. and neurologic manifestations_
• 37
• 39 References:
• 39
• 40 1. Restrictive management of neonatal polycythemia. se
• 41
• 42 End Block
Feedback
• 1
• 2 Item: 11 of 44 V-• Mark U.P
▪ 3 Previous Next Lab Values, Notes Calculator.
I 15...1 IULU TWILI I puiy.....yu IGI 111 c1 E11 u !auk. ULI 11 LI 1E11 I C7r.Jr.JGC71 II I I ....14.14.1).7 piuu IUI IL,. I IIJYTUVUI E1.3 LI IU
• 4-
▪ 5
hematocrit rises, the viscosity of the blood increases and impairs blood flow to various organs. The most
common symptoms are lethargy, irritability, and jitteriness. Other potential manifestations are respiratory
▪ 6
distress, tachypnea, cyanosis, and poor feeding, as seen in this neonate. The increased red blood cell mass
▪ 7
can lead to hypoglycemia and hypocalcemia due to increased cellular uptake_
▪ 8
▪ 9
Asymptomatic neonates require only hydration by feeding or parenteral fluids. Symptomatic neonates
• 10
require partial exchange transfusion, in which blood is removed in exchange for normal saline to normalize the
hematocrit.
• 12
• 13 (Choice A) Cyanotic heart disease is unlikely in this patient The absence of a murmur or cardiomegaly is
• 14 reassuring. and the neonate has oxygen saturation of >95% on room air_ The cyanosis in this neonate is due
• 15 to the high red blood cell mass and hyperviscosity rather than a right-to-left shunt.

• 17 (Choice B) Dehydration can cause elevated hematocrit levels. However, dehydration is very unusual in term
• 18 neonates in the first 2 days of life as they are born with excess extracellular water. Although this neonate has
• 19 not fed in 6 hours, it is unlikely that fluid loss could account for his symptoms.
20
21 (Choices C and E) Respiratory distress syndrome is unlikely in this term neonate. Rarely, term neonates of
• 22 diabetic mothers may have surfactant deficiency, as high insulin levels block the maturation of sphingomyelin.
• 23 However, this mother did not have diabetes and her newborn has a normal chest x-ray. Transient tachypnea
• 24 of the newborn would be a more common cause of respiratory distress in the term neonate but should show
• 25 increased pulmonary vascular markings or fluid in the fissures on chest x-ray.
• 26
• 27 Educational objective:
• 28 Polycythemia is defined as a hematocrit >65% in term neonates. Risk factors include delayed cord clamping, I
• 29 maternal hypertension, and maternal diabetes mellitus_ Symptomatic neonates generally have a combination
• 30 of respiratory distress, hypoglycemia, and neurologic manifestations_
• 31
• 32 References:
• 33
1. Restrictive management of neonatal polycythemia.
• 34
• 35 2. Partial exchange transfusion for polycythemia hyperviscosity syndrome.
• 36 3. Management of polycythemia in neonates.
• 37
• 39
• 39
Copyright © LIWorld Last updated: [9/5/2014]
• 40
• 41
• 42 End Block
Feedback
Item: 12 of 44 I'Mark
Previous Next . Lab Values. Notes Calculator.

A 7-year-old boy is brought to the physician because of fever, malaise, and pain in his right lower leg for the
last three days. His pain is 7110 in severity, non-radiating, and unrelieved by ibuprofen, heat, or rest He has
no history of trauma or fracture. There is no family history of sickle cell anemia. His temperature is 39.3'C
(103"F). Examination of his right leg shows bony point tenderness, erythema, and warmth over a portion of
the proximal tibia. Blood is drawn for culture and sensitivity studies_ Which of the following organisms is
most likely to be isolated from his blood?

A. Staphylococcus epodermidis [2%]


B. Staphylococcus aureus [86%]
C. Salmonella species [3%]
• 17
• 18 D. Klebsiella species [1%]
• 19 E. Pseudomonas aeruginosa [1%]
20 Group B streptococcus [.2%]
21
• 22
G. Escherichia cols [1%]
• 23 H. Streptococcus pyogenes [4%]
• 24
• 25
• 26 Explanation:
• 27
In both infants and children, Staphylococcus aureus is the most common organism responsible for
• 28
osteomyelitis. Other common organisms include group B streptococcus and Escherichia coli in infants, and
• 29
Streptococcus pyogenes in children.
• 30
• 31
(Choice A) Staphylococcus epidermidis is a frequent cause of osteomyelitis associated with prosthetic
• 32
devices_
• 33
• 34 (Choice C) Salmonella is a frequent cause of osteomyelitis in patients with sickle cell anemia.
• 35
• 36 (Choices D and E) Gram-negative rods such as Klebsiella and Pseudomonas are possible causes of
• 37 osteomyelitis when patients have a history of urinary tract infection (UTI) or urinary tract instrumentation_
• 38
• 39 Educational Objective:
• 40 Staphylococcus aureus is the most common cause of osteomyelitis in both infants and children. se
• 41
• 42 End Block
Feedback

• 2 Item: 12 of 44 V- Mark
• 3 Previous Next Lab Values. Notes Calculator.
• 4- H. -year -uiu uuy i ur uugr IL LU use pr ipiurdr LitU.clUt UI ItVtF, p-dIfl IFI FII F igr IL lUVitF ity IUF LFIt
• 5 last three days. His pain is 7110 in severity, non-radiating, and unrelieved by ibuprofen, heat, or rest He has
• 6 no history of trauma or fracture. There is no family history of sickle cell anemia His temperature is 39.3°C
• 7 (103°F). Examination of his right leg shows bony point tenderness, erythema, and warmth over a portion of
• 8 the proximal tibia Blood is drawn for culture and sensitivity studies_ Which of the following organisms is
• 9 most likely to be isolated from his blood?
• 10
- 11 A. Staphylococcus epidermidis [29C]
H. Staphylococcus aureus [86%]
- 13
- 14 C. Salmonella species [3%]
6 15 ID_ klebsiella species [1%]
E Pseudornonas aeruginosa
• 17
• 18 • Group 13 streptococcus [2%]
• 19 G. Escherichia colt [1%]
20
H. Streptococcus pyogenes [4%]
21
• 22
• 23 Explanation:
• 24
• 25 In both infants and children, Staphylococcus aureus is the most common organism responsible for
• 26 osteomyelitis. Other common organisms include group 13 streptococcus and Escherichia coli in infants. and
• 27 Streptococcus pyogenes in children.
• 28
• 29 (Choice A) Staphylococcus epidermidis is a frequent cause of osteomyelitis associated with prosthetic
• 30 devices_
• 31
• 32 (Choice C) Salmonella is a frequent cause of osteomyelitis in patients with sickle cell anemia
• 33
(Choices D and El Gram-negative rods such as Klebsiella and Pseudomonas are possible causes of
• 34
osteomyelitis when patients have a history of urinary tract infection (UTI) or urinary tract instrumentation.
• 35
• 36
Educational Objective:
• 37
Staphylococcus aureus is the most common cause of osteomyelitis in both infants and children_
• 39
• 39
Copyright @ UWorld Last updated: [8/22/2014]
• 40
• 41
• 42 End Block
Feedback
• 1
• 2 Item: 13 of 44 F' Mark
▪ 3 Previous Next Lab Values. Notes Calculator.
• 4-
▪ 5
▪ 6 A 16-year-old girl comes to the emergency department with painless, heavy vaginal bleeding for 3 days. Her
▪ 7 menses have been irregular since menarche at age 14, and skip up to 4 months at a time_ Her last menstrual
▪ 8 period was almost 4 months ago_ The patient has a steady boyfriend and uses condoms for contraception.
▪ 9 She has no other bleeding problems. Her temperature is 37 C (99 1), blood pressure is 110/60 mm Hg, pulse
• 10 is 90/min, and respirations are 16/min_ On speculum examination, she has a moderate amount of blood in
• 11 the vaginal vault and active bleeding through the cervical os. Bimanual examination shows a uterus of normal
• 12 size and shape with no adnexal masses_ Pregnancy test is negative. Coagulation studies are within normal
13 limits_ Her hemoglobin is 9.8 g/dL. Which of the following is the most appropriate next step in management of 1
• 14 this patient?
• 15

A. Emergency dilation and curettage [7]
• 17
• 18 ▪ B. High-dose estrogen therapy [34%]
• 19 C. High-dose gonadotropin-releasing hormone agonists [16%]
20
D. Packed red blood cell transfusion [8%]
21
• 22 E Sonohysterogram [3496]
• 23
• 24
Explanation:
• 25
• 26 Medical treatment options for acute abnormal uterine
• 27 bleeding
• 28
• 29
• High-dose intravenous or oral estrogen
• 30
• High-dose combined oral contraceptive pills
• 31
• High-dose progestin pills
• 32
• Tranexamic acid
• 33
• 34
• 35
This patient's acute abnormal uterine bleeding (AUB) is most likely due to ovulation dysfunction and
• 36
requires clinical intervention to prevent further bleeding_ In adolescents, the most common reason for acute
• 37
• 39
AUB is an immature hypothalamic-pituitary-ovarian axis resulting in some anovulatory cycles. Without
ovulation, these adolescents have persistent endometrial proliferation, followed by heavy menses in cycles
• 39
when ovulation does occur. Adolescents with heavy menstrual bleeding should also be evaluated for a
• 40
• 41
• 42 End Block
Feedback
• 1
• 2 Item: 13 of 44 V-Mark
▪ 3 Previous Next Lab Values. Notes Calculator.
• 4
Explanation:
▪ 5
▪ 6 7 Medical treatment options for acute abnormal uterine
▪ 7 bleeding
▪ 8
▪ 9
• High-dose intravenous or oral estrogen
• 10
• High-dose combined oral contraceptive pills
• 11
• High-dose progestin pills
• 12
• Tranexamic acid
13
• 14
• 15
This patient's acute abnormal uterine bleeding (AUB) is most likely due to ovulation dysfunction and

requires clinical intervention to prevent further bleeding_ In adolescents, the most common reason for acute
• 17
• 18
AUG is an immature hypothalamic-pituitary-ovarian axis resulting in some anovulatory cycles. Without
ovulation, these adolescents have persistent endometrial proliferation, followed by heavy menses in cycles
• 19
when ovulation does occur. Adolescents with heavy menstrual bleeding should also be evaluated for a
20
disorder of hemostasis if clinically appropriate. Sexually active adolescents should be tested for pregnancy_
21
• 22
Acute abnormal uterine bleeding in a hemodynamically stable patient may be managed medically. First-line
• 23
treatment is hormonal medication in formulations that work quickly, including estrogen (conjugated equine
• 24
estrogen) intravenously or orally, combined estrogen/progesterone pills taken in higher doses than for
• 25
contraception, or higher-dose progesterone pills_ Estrogen works rapidly by promoting regrowth of
• 26
endometrium over the denuded epithelial surface that results from prolonged heavy bleeding. Progesterone
• 27
may be given to patients who have a contraindication to estrogen or to treat acute AUB related to
• 28
anovulation. It stabilizes the disorganized endometrial lining and prevents further growth but is less effective
• 29
once the epithelial surface has been denuded_ Tranexamic acid, an antifibrinolytic, is used in patients with
• 30
contraindications to both estrogen and progestin.
• 31
• 32 (Choices A and D) Emergency dilation and curettage is rarely needed, but would be appropriate if medical
• 33 management failed after 24-36 hours. Packed red blood cell transfusion would be indicated if this patient
• 34 were hemodynamically unstable secondary to vaginal bleeding. However, these steps are not indicated given
• 35 her hemoglobin of >9 g/dL and the absence of symptoms secondary to anemia. Iron supplementation would
• 36 be enough to correct her anemia if bleeding is controlled.
• 37
• 39 (Choice C) High-dose gonadotropin-releasing hormone agonists may be used to suppress symptoms from
• 39 endometriosis and uterine fibroids. However, high-dose estrogen therapy is the appropriate treatment for
• 40 qrtiwp AI IP
• 41
• 42 End Block
Feedback
• 1
• 2 Item: 13 of 44 'Mark
▪ 3 Previous Next Lab Values. Notes Calculator.
• 4
▪ 5 Acute abnormal uterine bleeding in a hemodynamically stable patient may be managed medically. First-line
▪ 6 treatment is hormonal medication in formulations that work quickly, including estrogen (conjugated equine
▪ 7 estrogen) intravenously or orally, combined estrogen/progesterone pills taken in higher doses than for
▪ 8 contraception, or higher-dose progesterone pills_ Estrogen works rapidly by promoting regrowth of
▪ 9 endometrium over the denuded epithelial surface that results from prolonged heavy bleeding. Progesterone
• 10 may be given to patients who have a contraindication to estrogen or to treat acute AUB related to
• 11 anovulation. It stabilizes the disorganized endometrial lining and prevents further growth but is less effective
• 12 once the epithelial surface has been denuded_ Tranexamic acid, an antifibrinolytic, is used in patients with
13 contraindications to both estrogen and progestin.
• 14
• 15 (Choices A and D) Emergency dilation and curettage is rarely needed, but would be appropriate if medical
• management failed after 24-36 hours. Packed red blood cell transfusion would be indicated if this patient
• 17 were hemodynamically unstable secondary to vaginal bleeding. However, these steps are not indicated given
• 18 her hemoglobin of >9 WI:IL and the absence of symptoms secondary to anemia. Iron supplementation would
• 19 be enough to correct her anemia if bleeding is controlled.
20
21 (Choice C) High-dose gonadotropin-releasing hormone agonists may be used to suppress symptoms from
• 22 endometriosis and uterine fibroids. However, high-dose estrogen therapy is the appropriate treatment for
• 23 active AUB.
• 24
• 25 (Choice E) Sonohysterogram is appropriate in evaluating the uterine cavity for fibroids and polyps_ A 16-year-
• 26 old with this patient's history is unlikely to have these findings, and a sonohysterogram would not be part of the
• 27 initial evaluation.
• 28
Educational objective:
• 29
Acute abnormal uterine bleeding in adolescents is usually a result of anovulatory cycles from an immature
• 30
hypothalamic-pituitary-ovarian axis. Evaluation for pregnancy and bleeding disorders is generally
• 31
advised. High-dose estrogen is the first-line treatment for patients with moderate-to-severe bleeding.
• 32
• 33
• 34 References:
• 35 1. Abnormal uterine bleeding and dysfunctional uterine bleeding in pediatric and
• 36 adolescent gynecology.
• 37
• 39
• 39
Copyright © LIWorld Last updated: [11/12/2014]
• 40
• 41
• 42 End Block
Feedback
• 1
• 2 Item: 14 of 44 F' Mark -4(1

▪ 3 Previous Next Lab Values. Notes Calculator.


• 4-
▪ 5
▪ 6 A 2-hour-old boy has developed worsening cyanosis over the past few minutes. The infant was delivered
▪ 7 vaginally at term after an uncomplicated pregnancy. Birth weight was 4.1 kg (9 lb}_ Apgar scores were 7 and
▪ 8 8 at 1 and 5 minutes, respectively. Oxygen is administered by hood but does not relieve the cyanosis.
▪ 9 Further examination shows tachypnea, subcostal retractions, a normal first heart sound, a single and loud
• 10 second heart sound, and no murmur_ Which of the following is the most likely cause of the infant's cyanosis?
• 11
• 12
A. Atrial septal defect [2%]
• 13
14 Coarctation of the aorta [3%]
• 15 C. IDextrocardia [0%]
• 16
ID. Hypoplastic left heart syndrome [14%]
• 17
• 18 E Patent ductus arteriosus [3'36]
• 19 • Tetralogy of Fallot [18%]
20
G. Transposition of the great vessels [56%]
21
• 22 H. Ventricular septal defect [4%]
• 23
• 24
Explanation:
• 25
• 26
• 27
• 28
Congenital cyanotic heart disease in newborns
• 29
• 30
Condition Examination X-ray findings
• 31
• 32
• 33 Transposition of the • Single S2 "Egg-on-a-string" heart
• 34
great vessels • Murmur may be absent (narrow rnediastinum)
• 35
• 36
• 37 • Harsh pulmonic
• 39
stenosis murmur "Boot-shaped" heart
• 39 Tetralogy of Fallot (right ventricular
• 40 • Ventricular septa! hxy n rr rth
• 41
• 42 End Block
Feedback
Item: 14 of 44 -461 M=E--
F'Mark U.P
Previous , Next Lab Values, Notes Calculator.

Congenital cyanotic heart disease in newborns


• 7
• 8
• 0 Condition Examination X-ray findings
• 10
!
• 11
Transposition of the • Single 52 "Egg-on-a-string" heart
• 12
• 13 great vessels • Murmur may be absent (narrow mediastinurn)

• 15
• 16 • Harsh pulmonic
ss
tenosi murmur "Boot-shaped" heart
• 17
Tetralogy of Fallot (right ventricular
• 18
• Ventricular septa! hypertrophy)
• 19
defect murmur
20
_ I

21
• 22 • Single Si
• Minimal pulmonary
Tricuspid atresia • Ventricular septal
• 24 blood flow
defect murmur
.•
.
FIM
• 25
27
Tricuspid regurgitation Increased pulmonary
• 28 Truncus arteriosus
murmur blood flow, edema
• 20
• 30
• Total anomalous • Severe cyanosis
• 32 pulmonary venous Pulmonary edema
• 33 return with obstruction • Respiratory distress
• 34
.71

COUWorld
• 36
• D-transposition of the great vessels (TGV) is the most common congenital cyanotic heart disease.
• 38 Abnormal rotation of the great vessels during cardiac development results in an aorta arising from the right
• 39 ventricle and the pulmonary artery from the left ventricle ("arterial switch"). As a result, deoxygenated blood
• 40 rnrninn frnm tko h rirloc to tko rinht atril Im and uantrir lo anrJ is r harle to tko hetrivr tkrnl ink tho annFa
• 41
• 42
Feedback, End Block .
• 1
• 2 Item: 14 of 44 F'Mark
▪ 3 Previous Next . Lab Values. Notes Calculator.
V cI I11-1 I .1.-rU ICU .aU
• 4- blood flow
5 defect murmur
▪ 6 7
▪ 7
Tricuspid regurgitation Increased pulmonary
▪ 8 Truncus arteriosus
murmur blood flow, edema
▪ 9
• 10
• 11 Total anomalous • Severe cyanosis
• 12 pulmonary venous Pulmonary edema
• 13
return with obstruction • Respiratory distress
• 14
• 15 ©UWodd
• 16
• 17 ID-transposition of the great vessels (TGV) is the most common congenital cyanotic heart disease.
• 18 Abnormal rotation of the great vessels during cardiac development results in an aorta arising from the right
• 19 ventricle and the pulmonary artery from the left ventricle ("arterial switch"). As a result, deoxygenated blood
20 coming from the body goes to the right atrium and ventricle and is cycled back to the body through the aorta.
21 Oxygenated blood from the lungs is returned to the lungs by the left side of the heart through the pulmonary
• 22 artery. Characteristic findings include cyanosis within 24 hours of life, an absent S2 sound (single S2) on
• 23 auscultation, and a narrow mediastinum ("egg on a string") on x-ray.
• 24
• 25 After delivery, mixing of deoxygenated and oxygenated blood is necessary for survival and typically occurs
• 26 through a patent ductus arteriosus or ventricular septal defect (both of which cause murmurs) or through a
• 27 patent foramen ovale (usually no murmur). This infant most likely is mixing through a patent foramen ovale as
• 28 there is no murmur on examination. If there is inadequate mixing, the infant will rapidly develop severe
• 29 cyanosis and pulmonary edema and die unless a shunt is created (ie, emergency atrial septostomy).
• 30 Therefore, if TGV is suspected, prostaglandins should be initiated to keep the ductus arteriosus patent to
• 31 optimize inter-circulatory mixing, and echocardiography should be obtained.
• 32
• 33 (Choices A, C, E, and H) Atrial septal defect, dextrocardia, patent ductus arteriosus. and ventricular septal
• 34 defect are acvanotic congenital heart diseases_
• 35
(Choices B and 13) Coarctation of the aorta and hypoplastic left heart syndrome generally present with shock
• 36
due to impaired systemic perfusion. There may be some cyanosis depending on the degree of obstruction,
• 37
but more often the infants are pale or mottled. Both conditions feature a significant murmur.
• 39
• 39
(Choice F) Tetralogy of Fallot typically has a significant systolic murmur from the pulmonary stenosis as well V
• 40 • .1

• 41
• 42 End Block
Feedback
• 1
• 2 Item: 14 of 44 F' Mark
▪ 3 Previous Next Lab Values. Notes Calculator.
- r -
• 4-
Abnormal rotation of the great vessels during cardiac development results in an aorta arising from the right
▪ 5
ventricle and the pulmonary artery from the left ventricle ("arterial switch"). As a result, deoxygenated blood
▪ 6
coming from the body goes to the right atrium and ventricle and is cycled back to the body through the aorta.
▪ 7
Oxygenated blood from the lungs is returned to the lungs by the left side of the heart through the pulmonary
▪ 8
artery. Characteristic findings include cyanosis within 24 hours of life, an absent S2 sound (single S2) on
▪ 9
auscultation, and a narrow mediastinum ("egg on a string") on x-ray.
• 10
• 11 After delivery, mixing of deoxygenated and oxygenated blood is necessary for survival and typically occurs
• 12 through a patent ductus arteriosus or ventricular septal defect (both of which cause murmurs) or through a
• 13 patent foramen ovale (usually no murmur). This infant most likely is mixing through a patent foramen ovale as
• 14 there is no murmur on examination. If there is inadequate mixing, the infant will rapidly develop severe
• 15 cyanosis and pulmonary edema and die unless a shunt is created (ie, emergency atrial septostomy).
• 16 Therefore, if TGV is suspected, prostaglandins should be initiated to keep the ductus arteriosus patent to
• 17 optimize inter-circulatory mixing, and echocardiography should be obtained.
• 18
• 19 (Choices A, C, E, and H) Atrial septal defect, dextrocardia, patent ductus arteriosus. and ventricular septal
20 defect are acyanotic congenital heart diseases_
21
• 22 (Choices B and D) Coarctation of the aorta and hypoplastic left heart syndrome generally present with shock
• 23 due to impaired systemic perfusion. There may be some cyanosis depending on the degree of obstruction,
• 24 but more often the infants are pale or mottled. Roth conditions feature a significant murmur.
• 25
• 26 (Choice F) Tetralogy of Fallot typically has a significant systolic murmur from the pulmonary stenosis as well
as a ventricular septal defect murmur. The presentation can range from birth to a few years depending on the I
• 27
• 28 degree of right ventricular outflow obstruction and pulmonary blood flow.
• 29
Educational objective:
• 30
Transposition of the great vessels is the most common congenital cyanotic heart defect It generally presents
• 31
in the first few hours of life with cyanosis and a single loud second heart sound. Prostaglandins should be
• 32
initiated to optimize intra-circulatory mixing.
• 33
• 34
• 35 References:
• 36 1. Transposition of the great arteries.
• 37
• 39
• 39
Copyright © LIWorld Last updated: [1/8/2015]
• 40
• 41
• 42 End Block
Feedback
• 1 a
• 2 Item: 14 of 44 .11M Mark
▪ 3 Previous Next Lab Values Notes Calculator
• 4- Media Exhibit X
▪ 5
▪ 6 Transposition of the great arteries
▪ 7
▪ 8
Normal Heart Transposition of Great Vessels
▪ 9
• 10
• -1 -1
• 12
Transposed aorta
• 13
• 14
• 15
• 16
• 17
Transposed
• 19 pulmonary
• 19
artery
20
21
• 22
• 23
• 24
• 25
• 26
• 27
• 28
• 29
• 30
USPAL EWorIcir L LC
• 31
• 32
• 33
• 34
• 35
• 36
• 37
• 39
• 39
J
• 40


41
42
Feedback
a
End Block
• 1
• 2 Item: 15 of 44 F' Mark
▪ 3 Previous Next Lab Values. Notes Calculator.
• 4-
▪ 5
▪ 6 A 3-week-old boy in the neonatal intensive care unit has increased gastric residual volume and vomiting_ He
▪ 7 was born at 28 weeks gestation by vaginal delivery due to cervical incompetence_ Birth weight was 900 g (2
▪ 8 lb). He initially required mechanical ventilation but extubated to nasal cannula oxygen last week. He has been
▪ 9 advancing on continuous nasogastric formula feeds and gaining an average of 30 g each day. His
• 10 temperature is 35.6 C (96 F), blood pressure is 85/45 mm Hg, pulse is 14 /min, and respirations are 34/min.
• 11 Pulse oximetry is 97% on 2 L oxygen. Examination shows a lethargic neonate with abdominal distension.
• 12 Laboratory studies show leukocytosis and metabolic acidosis. Imaging would most likely show which of the
• 13 following?
• 14
I 15
• A. Abdominal x-ray with air in the bowel wall and portal veins [55%]
• 16
• 17 B. Abdominal x-ray with air in the stomach and duodenum but no distal air [14%]
• 18 C. Abdominal ultrasound with thickened pylorus muscle [17%]
• 19
ID. Contrast enema with narrow rectosigmoid and dilation of the rest of the colon [8%]
20
21 E. Head ultrasound with hemorrhage into the lateral ventricles [6%]
• 22
• 23
Explanation:
• 24
• 25
• 26
• 27
• 28
• 29
• 30
• 31
• 32
• 33
• 34
• 35
• 36
• 37
• 39
• 39
• 40
se
• 41
• 42 End Block
Feedback
• 1
• 2 Item: 15 of 44 r. Mark
▪ 3 Previous Next Lab Values. Notes Calculator.
• 4-
▪ 5
▪ 6
▪ 7
▪ 8
▪ 9
• 10
• 11
• 12 1
• 13
• 14
* 15
• 16
• 17
• 18
• 19
20
21
• 22
• 23
• 24
• 25
• 26
• 27
• 28
• 29
• 30
• 31
• 32
• 33
• 34
• 35
This infant's prematurity and very low birth weight put him in danger of life-threatening necrotizing
• 36
enterocolitis (NEC), the most common gastrointestinal emergency in the neonatal intensive care unit Clinical
• 37
findings depend on the location and extent of intestinal damage. Initial signs may be nonspecific, such as the
• 39
hypothermia and lethargy seen in this patient This patient also has characteristic gastrointestinal symptoms
• 39
(eg, vomiting, residual milk in the stomach, abdominal distension}_ so
• 40
• 41
• 42 End Block
Feedback
• 1
• 2 Item: 15 of 44 V-Mark
▪ 3 Previous Next Lab Values. Notes Calculator.
• 4- A

▪ 5
This infant's prematurity and very low birth weight put him in danger of life-threatening necrotizing
▪ 6
enterocolitis (NEC), the most common gastrointestinal emergency in the neonatal intensive care unit Clinical
▪ 7
findings depend on the location and extent of intestinal damage. Initial signs may be nonspecific, such as the
▪ 8
hypothermia and lethargy seen in this patient This patient also has characteristic gastrointestinal symptoms
▪ 9
(eg, vomiting, residual milk in the stomach, abdominal distension}_
• 10
• 11
The pathogenesis involves a combination of gut immaturity and exposure to bacteria from enteral feeds,
• 12
leading to a cascade of inflammation and damage to the bowel wall. The premature intestinal mucosa is
• 13
thought to have increased permeability and bacterial penetration. In addition, immature local host defenses
• 14
and decreased motility allow for bacterial overgrowth. If possible, premature infants should receive breast
I 15
milk instead of formula. Multiple studies have demonstrated decreased rates of NEC in breastfed
• 16
premature infants as breast milk may counteract some of the problems associated with gut immaturity.
• 17
However, the optimal timing of enteral feeding initiation and rate of advancement remains unclear, and
• 18
premature infants, especially <32 weeks gestation, remain vulnerable to NEC and its complications.
• 19
20 In a normal abdominal x-ray, air is visible only in the lumen of the bowel. However, in NEC, the air is visible in
21 the bowel wall ("double-line" or "train-track" appearance), leading to the hallmark finding of pneumatosis
• 22 intestinalis (red arrows above) on x-ray_ Linear, branching areas of lucency over the liver are also abnormal
• 23 and represent portal venous air (yellow arrows above). This results from gas produced by bacteria in the
• 24 portal veins or by the transmigration of gas from the bowel wall to mesenteric veins and into the portal vein.
• 25 Severe intestinal necrosis can cause perforation and pneumoperitoneum. Laboratory findings of
• 26 leukocytosis and metabolic acidosis reflect inflammation and intestinal ischemia, respectively_
• 27
• 28 (Choice B) Duodenal atresia typically presents as bilious vomiting shortly after initiation of feeds. The
• 29 abdomen is not distended because air cannot pass the duodenum. The classic radiologic finding is the
• 30 "double-bubble" sign (dilated stomach and duodenum), making this diagnosis unlikely.
• 31
• 32 (Choice C) The typical presentation of pyloric stenosis is post-prandial nonbilious emesis in an otherwise
• 33 healthy 3-6-week-old infant There is no abdominal distension due to obstruction in the proximal gut
• 34
• 35 (Choice ID) Almost all patients with Hirschsprung disease present with abdominal distension, feeding
• 36 intolerance. and failure to pass meconium within 48 hours of birth. Patients with milder disease may be
• 37 diagnosed later in life, but they typically have a history of chronic constipation and poor weight gain.
• 39
• 39 (Choice E) Premature infants are at risk for intraventricular hemorrhage. Although severe hemorrhage can
• 40 cause obstructive hydrocephalus and symptoms of increased intracranial pressure (eg, vomiting), it is so
• 41
• 42 End Block
Feedback
• 1
• 2 Item: 15 of 44 F' Mark
3 Previous Next Lab Values. Notes Calculator.
• 4-
intestinalis (red arrows above) on x-ray_ Linear, branching areas of lucency over the liver are also abnormal
5
and represent portal venous air (yellow arrows above). This results from gas produced by bacteria in the
6
portal veins or by the transmigration of gas from the bowel wall to mesenteric veins and into the portal vein.
7
Severe intestinal necrosis can cause perforation and pneumoperitoneum. Laboratory findings of
8
leukocytosis and metabolic acidosis reflect inflammation and intestinal ischemia, respectively_
9
• 10 (Choice B) Duodenal atresia typically presents as bilious vomiting shortly after initiation of feeds. The
• 11 abdomen is not distended because air cannot pass the duodenum. The classic radiologic finding is the
• 12 "double-bubble" sign (dilated stomach and duodenum), making this diagnosis unlikely_
• 13
• 14 (Choice C) The typical presentation of pyloric stenosis is post-prandial nonbilious emesis in an otherwise
I 15 healthy 3-6-week-old infant. There is no abdominal distension due to obstruction in the proximal gut.
• 16
• 17 (Choice 0) Almost all patients with Hirschsprung disease present with abdominal distension, feeding
• 18 intolerance, and failure to pass meconium within 48 hours of birth. Patients with milder disease may be
• 19 diagnosed later in life, but they typically have a history of chronic constipation and poor weight gain.
20
21 (Choice E) Premature infants are at risk for intraventricular hemorrhage. Although severe hemorrhage can
• 22 cause obstructive hydrocephalus and symptoms of increased intracranial pressure (eg, vomiting), it is
• 23 typically preceded by other abnormalities such as expanding head circumference, hypotonia, irregular
• 24 breathing, apnea, and/or seizures. This patient's abdominal distension and increased gastric residuals also
• 25 point to a gastrointestinal rather than neurologic problem.
• 26
• 27 Educational objective:
• 28 Increased gastric residual volume, vomiting, and abdominal distension in a preterm neonate are highly I
• 29 suspicious for necrotizing enterocolitis. Hallmark findings on x-ray include pneumatosis intestinalis
• 30 (intramural air) and portal venous air.
• 31
• 32 References:
• 33
1. Feeding practices and necrotizing enterocolitis.
• 34
• 35 2. A clinical perspective of necrotizing enterocolitis: past, present, and future.
• 36 3. Necrotizing enterocolitis.
• 37
• 39
• 39
Copyright © LIWorld Last updated: [1 G/14/2014]
• 40
• 41
• 42 End Block
Feedback
• -1
• 2 Item: 15 of 44 .111M Mark
▪ 3 Previous Next Lab Values Notes Calculator
• 4- Media Exhibit
▪ 5
▪ 6 Necrotiing enterocoitis
▪ 7
▪ 8
▪ 9
• 10
• -1 -1
• 12
• 13
• 14
I 15
• 16
• 17
• 19
• 19
20
21
• 22
• 23
• 24
• 25
• 26
• 27
• 28
• 29
• 30
• 31
• 32
• 33
• 34
• 35
• 36
• 37 v
• 39
• 39
• 40
• 41
• 42 End Block
Feedback
• -1
Item: 15 of 44 ja71 111
.
• 2 .11M Mark
▪ 3 Previous Next Lab Values Notes Calculator
• 4- Media Exhibit
▪ 5
▪ 6 Necrotiing enterocoitis
▪ 7
I. !
▪ 8
▪ 9
• 10
• -1 -1
• 12
• 13
• 14
I 15
• 16 Free air
• 17
• 19
under diaphgram
• 19

Nasogastric
• 22
tube
• 23
• 24
• 25
• 26
• 27
• 28
• 29
• 30
• 31
• 32
• 33
• 34
• 35
• 36
sq.-ad isini.m..1%•
• 37 v
• 30
• 39
• 40
• 41
• 42
Item: 15 of 44 71
ja
111
.
• 2 .11M Mark
▪ 3 Previous Next Lab Values Notes Calculator
• 4- Media Exhibit
▪ 5
▪ 6 Duodenal atreAa
▪ 7
▪ 8
• 9
• 10 Stomach
• 11
• 12
• 13
• 14
I 15
• 16
• 17
• 19
• 19 Duodenum
20
21
• 22 Nasogastric


23
24
tube
• 25
• 26
• 27
• 28
• 29
• 30 Absent di
• 31
• 32
integ al gas
• 33
• 34
• 35
• 36
• 37
• 39
• 39
• 40
• 41
• 42 End Block.
Feedback
• 1
• 2 Item: 16 of 44 11 V- Mark
▪ 3 Previous Next Lab Values. Notes Calculator.
• 4-
▪ 5
▪ 6 A 2-year-old boy is brought to the emergency department after developing coughing and shortness of breath a
▪ 7 few minutes after eating peanuts. He has a history of eczema treated with topical hydrocortisone_ His father
▪ 8 has a history of asthma. His temperature is 37.2° C (99° F), blood pressure is 100/70 mm Hg, pulse is
▪ 9 128/min, and respirations are 30imin_ Pulse oximetry is 91% on room air_ Diffuse wheezing and retractions
• 10 are present His voice is hoarse and he complains of itchiness_ Skin examination is shown in the photograph
• 11 below.
• 12
• 13
• 14
• 15
I 16
• 17
• 18
• 19
20
21
• 22
• 23
• 24
• 25
• 26
• 27
• 28
• 29
• 30
• 31
• 32
• 33
• 34
• 35
• 36
• 37
• 39
• 39
• 40
so
• 41
• 42 End Block
Feedback
• 1
• 2 Item: 16 of 44 Ali- Mark
▪ 3 Previous Next Lab Values. Notes Calculator.
• 4-
▪ 5
▪ 6
▪ 7
▪ 8
▪ 9
• 10
• 11 Which of the following is the best initial treatment for this patient?
• 12
• 13 A. Endotracheal intubation [10%]
• 14
B. Intramuscular epinephrine [85%]
• 15
I 16 C. Intravenous methylprednisolone [2%]
• 17 0. Inhaled albuterol [19.6]
• 18
E Oral diphenhydramine [19q
• 19
20 • Rigid bronchoscopy [0%]
21 G. Topical hydrocortisone [CM]
• 22
• 23
• 24 Explanation:
• 25
• 26 Anaphylaxis is a severe allergic reaction that is rapid in onset and can cause death. The clinical course is
• 27 unpredictable and potentially fatal, and prompt diagnosis and treatment are essential.
• 28
Children with one or more first-degree relatives with an allergic condition (eg, eczema, asthma, food allergy)
• 29
are at high risk for developing allergic disease_ Food allergies are caused by abnormal immunologic
• 30
• 31
responses to food proteins that swiftly induce inflammation to an organ system. Anaphylaxis occurs when the
patient has allergic symptoms affecting > 1 organ system or if the patient collapses after exposure to the food
• 32
protein. Symptoms can involve the oropharynx, skin, gastrointestinal tract, lungs, and cardiovascular
• 33
system. This patient suddenly developed respiratory distress, hoarseness, and urticaria after peanut
• 34
ingestion and needs immediate treatment with intramuscular epinephrine_
• 35
• 36
Epinephrine is the only medication proven to prevent and reverse progression of anaphylaxis to anaphylactic
• 37
shock and death_ Its p, agonist effect causes bronchodilation and decreases the systemic release of
• 39
inflammatory mediators. Its c4i agonist effect causes vasoconstriction, which raises blood pressure and
• 39
• 40 decreases upper-airway edema. Caregivers should be counseled on the importance of carrying an se
• 41
• 42 End Block
Feedback
• 1
• 2 Item: 16 of 44 V-Mark
▪ 3 Previous Next Lab Values. Notes Calculator.
• 4- Explanation: A

▪ 5
▪ 6 Anaphylaxis is a severe allergic reaction that is rapid in onset and can cause death. The clinical course is
▪ 7 unpredictable and potentially fatal, and prompt diagnosis and treatment are essential.
▪ 8
Children with one or more first-degree relatives with an allergic condition (eg, eczema, asthma, food allergy)
▪ 9
are at high risk for developing allergic disease_ Food allergies are caused by abnormal immunologic
• 10
• 11
responses to food proteins that swiftly induce inflammation to an organ system. Anaphylaxis occurs when the
patient has allergic symptoms affecting > 1 organ system or if the patient collapses after exposure to the food
• 12
protein. Symptoms can involve the oropharynx, skin, gastrointestinal tract, lungs, and cardiovascular
• 13
system. This patient suddenly developed respiratory distress, hoarseness, and urticaria after peanut
• 14
ingestion and needs immediate treatment with intramuscular epinephrine_
• 15
I 16
Epinephrine is the only medication proven to prevent and reverse progression of anaphylaxis to anaphylactic
• 17
shock and death_ Its 1 2 agonist effect causes bronchodilation and decreases the systemic release of
• 18
inflammatory mediators. Its c4i agonist effect causes vasoconstriction, which raises blood pressure and
• 19
20 decreases upper-airway edema. Caregivers should be counseled on the importance of carrying an
21 epinephrine autoinjector at all times and avoiding specific foods_
• 22
(Choice A) Patients with signs of respiratory decline (eg, worsening or severe hypoxia, stridor) and/or altered
• 23
• 24
mental status require emergent intubation. Delays to epinephrine administration increase the risk of
• 25
respiratory decline, and patients should be monitored closely for decompensation.
• 26
(Choices C and G) Glucocorticoids need several hours to take effect and do not provide the immediate
• 27
action required in this emergency situation. Systemic glucocorticoids (eg, oral or intravenous) can be helpful
• 28
in preventing prolonged or biphasic anaphylactic reactions_ Topical steroids have no significant role in this
• 29
systemic disease.
• 30
• 31
(Choice ID) The p2 agonist effect from albuterol can help relieve lower-airway obstruction (eg,
• 32
• 33 wheezing). However, albuterol does not relieve other dangerous symptoms of anaphylaxis such as upper-
• 34 airway edema (eg, stridor, hoarseness) or hypotension. Priority must be given to epinephrine as delays in its
• 35 administration increase the risk of mortality.
• 36
(Choice E) H, antihistamines relieve pruritus and hives_ Antihistamines do not relieve airway obstruction or
• 37
• 39 hypotension. Furthermore, oral medications should not be given to patients in acute respiratory distress due
• 39 to the risk of aspiration.
• 40
so
• 41
• 42 End Block
Feedback
• 1
• 2 Item: 16 of 44 V-Mark
▪ 3 Previous Next Lab Values. Notes Calculator.
• 4
(Choice A) Patients with signs of respiratory decline (eg, worsening or severe hypoxia, stridor) and/or altered
▪ 5
mental status require emergent intubation_ Delays to epinephrine administration increase the risk of
▪ 6
respiratory decline, and patients should be monitored closely for decompensation.
▪ 7
▪ 8
(Choices C and G) Glucocorticoids need several hours to take effect and do not provide the immediate
▪ 9
action required in this emergency situation. Systemic glucocorticoids (eg, oral or intravenous) can be helpful
• 10
in preventing prolonged or biphasic anaphylactic reactions_ Topical steroids have no significant role in this
• 11
systemic disease.
• 12
• 13
(Choice 0) The 132 agonist effect from albuterol can help relieve lower-airway obstruction (eg,
• 14
wheezing). However, albuterol does not relieve other dangerous symptoms of anaphylaxis such as upper-
• 15
airway edema (eg, stridor, hoarseness) or hypotension. Priority must be given to epinephrine as delays in its
I 16
administration increase the risk of mortality_
• 17
• 18
(Choice El H, antihistamines relieve pruritus and hives_ Antihistamines do not relieve airway obstruction or
• 19
hypotension. Furthermore, oral medications should not be given to patients in acute respiratory distress due
20
to the risk of aspiration.
21
• 22
(Choice F) The differential diagnosis of wheezing and coughing after food ingestion should include foreign
• 23
body aspiration, especially in children age < 3 years_ If aspiration is suspected, rigid bronchoscopy is the
• 24
procedure of choice for identifying and removing the object Foreign body aspiration is less likely in this patient
• 25
given the bilateral distribution of wheezes, presence of hives on examination, and absence of choking.
• 26
• 27 Educational objective:
• 28 Anaphylaxis is a severe allergic reaction characterized by sudden collapse or symptoms involving > 1 organ
• 29 system after exposure to an offending antigen. Intramuscular epinephrine is the first-line treatment and should
• 30 be given immediately to reduce mortality.
• 31
• 32
References:
• 33
• 34 1. Self-injectable epinephrine for first-aid management of anaphylaxis.
• 35 2. 2012 Update: World Allergy Organization Guidelines for the assessment and management
• 36 of anaphylaxis.
• 37
• 39
• 39
Copyright @ UWorld Last updated: [10/5/2014]
• 40
• 41
• 42 End Block
Feedback
• 1
• 2 Item: 17 of 44 VI/lark
▪ 3 Previous Next Lab Values. Notes Calculator.
• 4- 1.
▪ 5
▪ 6 A 6-year-old boy is brought to the pediatrician with leg pain_ Over the last few months, he has complained of
▪ 7 pain in both legs that occurs only at night The pain is worst in his thighs although he occasionally has pain in
▪ 8 his calves as well. The episodes last a few hours each and improve with massage and over-the-counter
▪ 9 medications. The pain has remained unchanged over the past few months. The patient is able to walk, run,
• 10 and play at school without any complaints, and his mother has noticed no change in his activity level. He has
• 11 no fever, chills, or weight loss and there is no history of recent trauma. On examination, the boy has full range
• 12 of motion of the hips and knees bilaterally. No swelling or tenderness to palpation is noted and he has a
• 13 normal gait His mother is very worried because his 9-year-old cousin died of leukemia 6 months earlier.
• 14 Which of the following is the most appropriate next step in management of this patient?
• 15
• 16
A. Blood cultures [11.6]
• 18 R. Bone scan [19C]
• 19
C. Complete blood count [14%]
20
21
D. Observation and reassurance [11 %]
• 22 E Plain radiographs [11 %]
• 23
F. Psychiatric evaluation [3%]
• 24
• 25
• 26 Explanation:
• 27
• 28 Growing pains are a common musculoskeletal complaint in children, occurring in approximately 1G%-30% of
• 29 children age 2-12 years. The etiology of growing pains is unknown, but they are unrelated to growth, despite
• 30 their name. The diagnosis of growing pains can be made clinically (Table) in the absence of systemic
• 31 symptoms and abnormal examination findings. Laboratory studies and radiographs are not necessary.
• 32


33
34
Growing pains
• 35
• 36 • Occurs primarily at night & resolves by morning
• 37
Clinical • Affects lower extremities (eg, thighs, calves),
• 39
• 39 features usually bilateral
• 40 11.1 pew% r -k% lir.1".21. I nvominotinn 2 emenfikoil-io

• 41
• 42
• 1
• 2 Item: 17 of 44 F'Mark
▪ 3 Previous Next . Lab Values. Notes Calculator.
• 4-
5 Growing pains
▪ 6 7 —
▪ 7
▪ 8 • Occurs primarily at night & resolves by morning
▪ 9
Clinical • Affects lower extremities (eg, thighs, calves),
• 10
features usually bilateral
• 11
• 12 • Normal physical examination & activity
• 13
• 14
• 15 • Parental education & reassurance
Treatment
• 16 • Massage, stretching exercises, heat & analgesics

• 18 OUVVorld
• 19
20 Treatment of growing pains consists of parental education and reassurance along with massage,
21 muscle-stretching exercises, and administration of over-the-counter analgesics_ Children with growing pains
• 22 should be followed closely to monitor for pain that increases in frequency or intensity, which may warrant
• 23 further evaluation.
• 24
• 25 (Choice A) Blood cultures are indicated in children with osteomyelitis or septic arthritis, which are bone and
• 26 joint infections that present with fever, localized pain. and decreased range of motion_ None of these are
• 27 present in this patient
• 28
• 29 (Choice B) Bone scans are used in the diagnosis of metastatic bone disease or osteomyelitis, both of which
• 30 typically present with unilateral pain and systemic symptoms such as fever or weight loss_ Bone scans are
• 31 not indicated in children suspected of having growing pains.
• 32
• 33 (Choice C) A complete blood count can be used to diagnose leukemia, which can be a cause of limb pain in
• 34 children. Leukemia classically presents with fever, weight loss, pallor, and other systemic symptoms that are
• 35 not present in this patient.
• 36
(Choice E) X-rays can be used to diagnose both benign (eg, osteoid osteoma) and malignant (eg,
• 37
• 39
osteosarcoma) bone tumors. Although osteoid osteomas frequently present with limb pain that is worse at
night and responds to treatment with nonsteroidal anti-inflammatory drugs, they are most common in the
• 39
second decade of life and the pain is often unilateral_ Limp and point tenderness may be present on V
• 40
• 41
• 42 End Block
Feedback
• 1
• 2 Item: 17 of 44 V- Mark
-4Z1

▪ 3 Previous Next Lab Values. Notes Calculator.


11. .6011LPIL•W FL) 1_714..14..11...1 4.UILU1 GJ E11 G 1111...114...CILLI II I 4,11114,11U! I TYILI I 1.,ILUI...1111),U1ILI. UI JG 1JLI4. Ul LI II 10E1114,11 Ul G UUI IG C711U
• 4-
▪ 5
joint infections that present with fever, localized pain, and decreased range of motion. None of these are
present in this patient
▪ 6
▪ 7
(Choice B) Bone scans are used in the diagnosis of metastatic bone disease or osteomyelitis, both of which
▪ 8
typically present with unilateral pain and systemic symptoms such as fever or weight loss_ Bone scans are
▪ 9
not indicated in children suspected of having growing pains.
• 10
• 11 (Choice C) A complete blood count can be used to diagnose leukemia, which can be a cause of limb pain in
• 12 children_ Leukemia classically presents with fever, weight loss, pallor, and other systemic symptoms that are
• 13 not present in this patient
• 14
• 15 (Choice E) X-rays can be used to diagnose both benign (eg, osteoid osteoma) and malignant (eg,
• 16 osteosarcoma) bone tumors. Although osteoid osteomas frequently present with limb pain that is worse at
night and responds to treatment with nonsteroidal anti-inflammatory drugs, they are most common in the
• 18 second decade of life and the pain is often unilateral_ Limp and point tenderness may be present on
• 19 examination. X-rays are indicated in children with systemic symptoms, unilateral limb pain, limp, limitation of
20 activities, or abnormal physical examination findings. This patient has none of these.
21
• 22 (Choice F) The etiology of growing pains is unknown; although psychogenic illness has been mentioned as a
• 23 potential cause, it has never been formally studied. Children with growing pains often have a lower pain
• 24 threshold and more depressive symptoms when compared to other children, but psychiatric evaluation is not
• 25 required for these patients.
• 26
• 27 Educational objective:
• 28 Growing pains are bilateral, lower-extremity pains that occur at night in children age 2-12 years_ Children with
• 29 growing pains have no systemic symptoms, normal activity levels, and normal physical examinations.
• 30 Treatment consists of observation, parental reassurance, massage, and over-the-counter pain medications.
• 31
• 32 References:
• 33
1. Growing pains: a study of 30 cases and a review of the literature.
• 34
• 35 2. Decreased pain threshold in children with growing pains.
• 36 3. Are growing pains a myth?
• 37
• 39
• 39
Copyright © LIWorld Last updated: [1D/21/2014]
• 40
• 41
• 42 End Block
Feedback
• 1
• 2 Item: 17 of 44 V-• Mark FF. 11
.
3 Previous Next Lab Values Notes Calculator

• 4- Media Exhibit X
5
• 6 Osteoid osteoma
• 7
• 8
9
• 10
• 11
• 12
• 13
• 14
• 15
• 16
• 17
• 19
• 19
20
21
• 22
• 23
• 24
• 25
• 26
• 27 .„ •
• 28
• 29
• 30
• 31
• 32
• 33
Aollf
• 34
• 35
• 36
V
• 37
• 39
• 39
• 40


41
42
v Feedback
a
End Block
• -1
• 2 Item: 17 of 44 V-Mark
11
.

▪ 3 Previous Next Lab Values Notes Calculator


• 4- 141edia Exhibit
▪ 5
▪ 6 Osteosarcoma
▪ 7
▪ 8
▪ 9
• 10
• 11
• 12
• 13
• 14
• 15
• 16

• 19
• 19
20
21
• 22
• 23
• 24
• 25
• 26
• 27
• 28
• 29
• 30
• 31
• 32
• 33
• 34
• 35
• 36
• 37
• 39
• 39
• 40
• 41
• 42
• 1
• 2 Item: 18 of 44 F' Mark -4(1

▪ 3 Previous Next Lab Values. Notes Calculator.


• 4-
▪ 5
▪ 6 A 16-year-old girl is brought to the emergency department by her 18-year-old boyfriend due to nausea,
▪ 7 vomiting, and severe pain in her lower right quadrant She also complains of some anorexia_ The patient has
▪ 8 had these symptoms for almost 12 hours. Her last menstrual period was 2 weeks ago, and she has not been
▪ 9 sexually active in the past 2 weeks. Her temperature is 38.1' C (1C[16° F), blood pressure is 110/713 mm Hg,
• 10 pulse is 88/min, and respirations are 12/min. Examination shows diffuse discomfort in her abdomen that is
• 11 increased in the right lower quadrant without guarding or rebound tenderness_ Pregnancy test, workup for
• 12 sexually transmitted diseases, and pelvic examination are negative. Abdominal and pelvic computed
• 13 tomography scans show nonspecific inflammation in the area of the appendix but are not definitive for
• 14 appendicitis. The patient's condition remains stable with improved pain control. However, it is recommended
• 15 that she be admitted to the hospital for further observation, with consideration for possible exploratory
• 16 laparotomy in the next 24 hours if she worsens_ This is discussed with the patient and her boyfriend. The
• 17 physician inquires about obtaining consent for further medical care. The patient says that she lives with her
I 18 parents but does not want them called as "they would find out I've been spending time with my boyfriend, and
• 19 they don't approve of him." Who is the appropriate person to sign the consent form for admission and further
20 medical care?
21
• 22
• 23
A. Parent of the adult boyfriend [0%]
• 24 R. Patient [14%]
• 25
C. Patient's adult boyfriend [2%]
• 26
• 27
► ID. Patient's parent [8.213C]
• 28 E Physician in the emergency department [2%]
• 29
• 30
• 31 Explanation:
• 32
This patient is considered to be an unemancipated minor and her condition is stable (as opposed to an
• 33
▪ 34 emergency). It is therefore necessary for a parent to provide consent.
• 35
• 36 Circumstances in which minors
• 37
do not require consent
• 38
• 39
• 40 I . Ernimrnanrw r--5ina
• 41
• 42 End Block
Feedback
• 1
• 2 Item: 18 of 44 F'Mark
▪ 3 Previous Next . Lab Values. Notes Calculator.
• 4-
5
Circumstances in which minors
▪ 6
▪ 7 do not require consent
▪ 8 . .
▪ 9
• Emergency care
• 10
• 11 Medical • Sexually transmitted infections
• 12 circumstances • Substance abuse (most states)
• 13
• 14 • Prenatal care (most states)
• 15
• 16
• 17 • Homeless
I 18 • Parent
• 19
20 Emancipated • Married
21 minor • Military
• 22
• 23
• Financially independent
• 24 • High school graduate
• 25
• 26 uworld.corn
• 27
• 28 Minors in all states may be given emergency care without consent if obtaining consent would delay treatment,
• 29 but the parents or legal guardians must be notified as soon as possible. Laws may vary by state regarding
• 30 minors in some situations. Many states also allow minors to consent without parental notification for
• 31 contraceptive services, sexually transmitted disease care, prenatal care, mental health services, and
• 32 substance abuse. Minors who are parents may consent for treatment involving their own minor children.
• 33
• 34 This patient may have the computed tomography scan without consent as it is part of the emergency care
• 35 required to make a diagnosis and determine the need for surgery. However, she does not need emergency
• 36 surgery and will be admitted and observed, with the possibility of surgery in the future. Her parents or legal
• 37 guardians should be notified and are required to give consent for further medical care.
• 38
• 39 (Choices A and C) Although this girl's boyfriend and his mother are adults, they do not have the legal
authority to provide consent for her medical care. V
• 40
• 41
• 42 End Block
Feedback
• 1
• 2 Item: 18 of 44 F'Mark
▪ 3 Previous Next . Lab Values. Notes Calculator.
• 4- • noineless
5 • Parent
▪ 6
▪ 7 Emancipated • Married
▪ 8 minor • Military
▪ 9
• 10 • Financially independent
• 11 • High school graduate
• 12
• 13 uworld.corn
• 14
• 15 Minors in all states may be given emergency care without consent if obtaining consent would delay treatment,
• 16 but the parents or legal guardians must be notified as soon as possible. Laws may vary by state regarding
• 17 minors in some situations. Many states also allow minors to consent without parental notification for
I 18 contraceptive services, sexually transmitted disease care, prenatal care, mental health services, and
• 19 substance abuse. Minors who are parents may consent for treatment involving their own minor children.
20
21 This patient may have the computed tomography scan without consent as it is part of the emergency care
• 22 required to make a diagnosis and determine the need for surgery. However, she does not need emergency
• 23 surgery and will be admitted and observed, with the possibility of surgery in the future. Her parents or legal
• 24 guardians should be notified and are required to give consent for further medical care. 141
• 25
• 26 (Choices A and C) Although this girl's boyfriend and his mother are adults, they do not have the legal
• 27 authority to provide consent for her medical care.
• 28
• 29 (Choice B) Because this girl is an unemancipated minor, she does not have the authority to consent to
• 30 medical treatment unless it is regarding a protected issue such as contraception, pregnancy, or substance
• 31 abuse.
• 32
(Choice E) Physicians do not have the legal authority to provide consent for an unemancipated minor_
• 33
• 34
Educational objective:
• 35
Unemancipated minors normally cannot consent to their own medical treatment Parents or legal guardians
• 36
must provide consent on the minor's behalf before the physician can proceed, although there are exceptions
• 37
in emergencies and other situations.
• 38
• 39
Copyright © UWorld Last updated: [9/22/2014]
• 40
• 41
• 42 End Block
Feedback
1
2 Item: 19 of 44 V-• Mark -4(1
U
.P
3 Previous Next Lab Values, Notes Calculator.
4-
5
6 A mother brings her 3-year-old son to the pediatrician because he has had 7 days of fever and a painful
7 swollen lymph node in his right groin. This is the boy's sixth episode of lymph node swelling; previous
8 episodes resolved after drainage and prolonged antibiotic therapy. The child also has a past medical history
9 significant for pneumonia at 1 months of age that required chest tube placement for drainage. His maternal
10 uncle died in childhood from recurrent infections_ On examination, the boy is at the 5th percentile for both
11 height and weight His temperature is 385°C (1013°F) There is an enlarged, tender lymph node in the right
12 inguinal area with erythema and warmth of the overlying skin. There are several healed incisions in the
13 inguinal regions bilaterally, as well as around the neck from old drainage sites. Laboratory analyses reveal the
14 following:
15
Hematocrit 35%
16
Platelet count 350,000imm3
17
18
Leukocyte count 17000/mm3
Segmented neutrophils 65%
Bands 10%
Lymphocytes 25%
• 22
• 23 Gram stain of fluid aspirated from the affected lymph node reveals numerous bacteria-filled segmented
• 24 neutrophils. Cultures grow S. aureus. What is the most likely mechanism underlying these findings?
• 25
• 26
A_ Adenosine deaminase deficiency [7%]
• 27
• 28
B. Complement consumption [2%]
• 29 C. Defective opsonization [8%]
• 30
ID. Destruction of CD4+ lymphocytes [1%]
• 31
• 32
E Developmental arrest in B lymphocyte maturation [12°•0]
• 33 E Dysmorphogenesis of the 3rd and 4th pharyngeal pouches [2%]
• 34
0 G. Impaired oxidative metabolism within phagocytes [61%]
• 35
• 36
• 37 Explanation:
• 38
• 39 This patient most likely has chronic granulomatous disease (CGD), a congenital immunodeficiency
• 40 syndrome. In normal individuals. pathogens are destroyed within phagocytes secondary to oxidative damage. se


41
42
Feedback
a
End Block
• 1
• 2 Item: 19 of 44 V- Mark
▪ 3 Previous Next Lab Values. Notes Calculator.
• 4
This patient most likely has chronic granulomatous disease (CGD), a congenital immunodeficiency
5
syndrome. In normal individuals, pathogens are destroyed within phagocytes secondary to oxidative damage.
▪ 6
In CGD, a mutation causes loss or inactivation of the NADPH oxidase responsible for oxidation_ Patients
▪ 7
experience recurrent infections starting early in life, and are especially susceptible to catalase-expressing
▪ 8
organisms like Staphylococcus aureus_ Neutrophilic functions such as chemotaxis, phagocytosis and
▪ 9
degranulation are intact in these patients; only intracellular killing is deficient Thus neutrophils filled with
• 10
bacteria are a classic finding on Gram stain.
• 11
• 12
(Choice A) Adenosine deaminase deficiency is an autosomal recessive cause of severe combined
• 13
immunodeficiency (SCID). While bacterial infections do occur in patients with SLID, Prieurnocystis joroveci,
• 14
candida, parainfluenza, and herpesvirus infections are most common. Lymphopenia is a classic finding, in
• 15
contrast to the patient above who has an absolute lymphocyte count of 4250/mm3_
• 16
- 17 (Choice B) Complement overconsumption occurs when large quantities of antibody-antigen complexes are
- 18 present, as in vasculitis, glomerulonephritis, and certain connective tissue diseases. The classic example is
systemic lupus erythematosus. This patient's presentation is not consistent with any of these diseases.

(Choice C) C3 is the major opsonin of the complement system. C3 deficiency predisposes to recurrent
• 22 infections with encapsulated bacteria, not S. aureus, beginning shortly after birth.
• 23
• 24 (Choice ID) CD4+ lymphocyte destruction is the hallmark finding in HIV/AIDS. While patients with AIDS may
• 25 have recurrent bacterial infections, they are more likely to suffer from viral, fungal, mycobacterial, and
• 26 protozoal infections. Most patients with AIDS are lymphopenic.
• 27
• 28
(Choice E) 9-cell maturational arrest is the best-described cause of agammaglobulinemia, the most
• 29
common primary immunodeficiency. These patients have poor 9-cell maturation and therefore poor antibody
• 30
production. Affected individuals are at high risk for infections with encapsulated bacteria because
• 31
encapsulated bacteria require antibody binding for effective opsonization. Sinusitis, bronchitis, otitis media
• 32
and pneumonia are common infections in these patients.
• 33
(Choice F) DiGeorge syndrome is marked by dysmorphogenesis of the third and fourth pharyngeal pouches.
• 34
Affected patients develop hypocalcemia from parathyroid hypoplasia and defective T-cell function from thymic
• 35
hypoplasia. Fungal and viral infections are common.
• 36
• 37
Educational objective:
• 38
Chronic granulomatous disease is an inherited immunodeficiency disorder marked by an inability to oxidize
• 39
pathogens within phagocytes_ Patients experience recurrent infections starting early in life. Pneumonia and
• 40 _1
• 41
• 42 End Block
Feedback
1
• 2 Item: 19 of 44 V-Mark
-4Z1

3 Previous Next Lab Values. Notes Calculator.


II I V 4J I-1, L4 I I ILI LLi LI LI I I L. Li Li 4-..J I4.1JJ 4-1 I III L4 L. LIY L4 LI 4-1 I I LiI LI Ii. I 510-41-11 I I LA G .J L. I I IJI L.. Ii. I 4-1 I LA L4 LI LI I I_ I L4 LI L.! I L.J
• 4-
experience recurrent infections starting early in life, and are especially susceptible to catalase-expressing
5
organisms like Staphylococcus aureus_ Neutrophilic functions such as chemotaxis, phagocytosis and
6
degranulation are intact in these patients; only intracellular killing is deficient Thus neutrophils filled with
7
bacteria are a classic finding on Gram stain.
8
9
(Choice A) Adenosine deaminase deficiency is an autosomal recessive cause of severe combined
• 10
immunodeficiency (SCID). While bacterial infections do occur in patients with SLID, Pneurnocystis firoveci,
• 11
candida, parainfluenza, and herpesvirus infections are most common_ Lymphopenia is a classic finding, in
• 12
contrast to the patient above who has an absolute lymphocyte count of 4250imm3_
• 13
• 14 (Choice B) Complement overconsumption occurs when large quantities of antibody-antigen complexes are
• 15 present. as in vasculitis, glomerulonephritis, and certain connective tissue diseases. The classic example is
• 16 systemic lupus erythematosus. This patient's presentation is not consistent with any of these diseases.
- 17
- 18 (Choice C) C3 is the major opsonin of the complement system. C3 deficiency predisposes to recurrent
infections with encapsulated bacteria, not S. aureus, beginning shortly after birth.

(Choice ID) CD4+ lymphocyte destruction is the hallmark finding in HIV/AIDS. While patients with AIDS may
• 22 have recurrent bacterial infections, they are more likely to suffer from viral, fungal, mycobacterial, and
• 23 protozoal infections. Most patients with AIDS are lymphopenic.
• 24
• 25 (Choice El B-cell maturational arrest is the best-described cause of agammaglobulinemia, the most
• 26 common primary immunodeficiency. These patients have poor B-cell maturation and therefore poor antibody
• 27 production. Affected individuals are at high risk for infections with encapsulated bacteria because
• 28 encapsulated bacteria require antibody binding for effective opsonization. Sinusitis, bronchitis, otitis media
• 29 and pneumonia are common infections in these patients.
• 30
(Choice F) DiGeorge syndrome is marked by dysmorphogenesis of the third and fourth pharyngeal pouches_
• 31
Affected patients develop hypocalcemia from parathyroid hypoplasia and defective T-cell function from thymic
• 32
hypoplasia. Fungal and viral infections are common.
• 33
. 34
Educational objective:
• 35
Chronic granulomatous disease is an inherited immunodeficiency disorder marked by an inability to oxidize
• 36
pathogens within phagocytes_ Patients experience recurrent infections starting early in life. Pneumonia and
• 37
suppurative adenitis are especially common infections in these patients.
• 38
• 39
Copyright © UWorld Last updated: [8/22/2014]
• 40
• 41
• 42 End Block
Feedback
• 1
• 2 Item: 20 of 44 r Mark .4111°.
▪ 3 Previous Next Lab Values. Notes Calculator.
• 4
▪ 5
The following vignette applies to the next 2 items.
▪ 6
▪ 7
▪ 8
A 4-day-old full-term boy is brought to the physician for his first office visit after an uncomplicated vaginal
▪ 9
delivery and newborn nursery course. The mother's blood type is A positive, and prenatal laboratory results
• 10
and ultrasound were normal_ Birth weight was 3.4 kg (7 lb 8 oz) and length was 48.5 cm (18 in). He has been
• 11
exclusively breastfed since birth and nurses for 10 minutes on each breast every 4 hours. The infant passed
• 12
several dark-brown, sticky meconium stools during the first 2 days of life but has had only smears of dark
• 13
yellow stool today. He has 3 wet diapers each day_ For the past day, his diapers have appeared as shown
• 14
below.
• 15
• 16
• 17
• 18
• 19

21
• 22
• 23
• 24
• 25
• 26
• 27
• 28
• 29
• 30
• 31
• 32
• 33
• 34
• 35
• 36
• 37
• 38
• 39 The neonate's current weight is 3.15 kg (6 lb 15 oz). He has scleral icterus and jaundice of the face, chest,
• 40 and abdomen. The rest of his physical examination is normal. Laboratory results are as follows:
• 41
• 42 End Block
Feedback
• 1
• 2 Item: 20 of 44 RI Mark
▪ 3 Previous Next Lab Values. Notes Calculator.
• 4
▪ 5
▪ 6
▪ 7
▪ 8
▪ 9 The neonate's current weight is 3.15 kg (6 lb 15 oz). He has scleral icterus and jaundice of the face, chest,
• 10 and abdomen. The rest of his physical examination is normal. Laboratory results are as follows:
• -1 -1 Total bilirubin 15 mg/dL
• 12 Direct bilirubin 0.9 mgldL
• 13
• 14 Item 1 of 2
• 15
• 16 Which of the following is the most likely cause of this infant's hyperbilirubinemia?
• 17
• 18
A. Alloimmune hemolytic disease [8%]
• 19
B. Biliary atresia [59C]
C. Breast milk jaundice [37%]
• 22
V D. Breastfeeding failure jaundice [42%]
• 23
• 24 E. Dubin-Johnson syndrome [5%]
• 25 F. Galactosemia [3%]
• 26
• 27
• 28 Explanation:
• 29
• 30
• 31
Breastfeeding failure jaundice vs breast milk jaundice
I
• 32
• 33
Diagnosis Timing Pathophysiology Clinical features
I
• 34
• 35 Lactation failure resulting in:
• 36 • Suboptimal
Breastfeeding •• Decreased bilirubin breasffceding
• 37 First week
failure elimination
• 33 of life • Signs of
• 39
jaundice • Increased enterohepatic dehydration se
• 40 rirri 11Qtirin
• 41
• 42 End Block
Feedback
• 1
• 2
▪ 3 Previous Next Lab Values. Notes Calculator.
• 4-
▪ 5 Breastfeeding failure jaundice vs breast milk jaundice
▪ 6
▪ 7 Diagnosis Timing Pathophysiology Clinical features
▪ 8
▪ 9
Lactation failure resulting in:
• 10 • Suboptimal
• 11 Breastfeeding •• Decreased bilirubin breastfeeding
First week
• 12 failure elimination
• 13
of life • Signs of
jaundice • Increased enterohepatic
• 14 dehydration
• 15 circulation
• 16
• 17 Starts at
• 18 High levels. of 3-glucuronidase • Adequate
age 3 -5
• 19 Breast milk in breast milk deconjugate breastfeeding
days;
jaundice intestinal bilirubin & increase • Normal
peaks at 2 enterohepatic circulation examination
• 22 weeks
• 23
• 24 OU5MLEWorkl,L1C
• 25
• 26 Breastfeeding failure jaundice is exaggerated unconjugated hyperbilirubinemia in the first week of life that is
• 27 caused by lactation failure_ It can be caused by both maternal (eg, inadequate milk supply, cracked/clogged
• 28 nipples, engorgement, infrequent feeding) and infant (eg, poor latch, ineffective suck, falling asleep)
• 29 factors. Normal infants pass dark, sticky meconium during the first 2 days of life, after which they should
• 30 transition to yellowish or green stool if ingesting adequate milk_ Inadequate stooling results in decreased
• 31 bilirubin elimination and increased enterohepatic circulation of bilirubin.
• 32
In addition to jaundice, infants with breastfeeding failure are often dehydrated_ During the first week of life, the
• 33
normal number of wet diapers a day should equal the infant's age in days (eg, a 4-day-old infant should have
• 34
L4 wet diapers a day). This infant has lost 7% of his birth weight, has slightly decreased urine output. and has
• 35
"brick-red" urate crystals in his diapers, all of which are signs of mild dehydration_ He also has not passed
• 36
yellow-seedy stools_ These clinical findings and the 4-hour feeding intervals make breastfeeding failure
• 37
jaundice the most likely diagnosis.
• 38
• 39
(Choice A) Alloimmune hemolytic disease is caused by hemolysis of fetal red blood cells due to ARO or Rh se
• 40
• 41
• 42 End Block
Feedback
• 1
• 2 Item: 20 of 44 F' Mark
▪ 3 Previous Next Lab Values. Notes Calculator.
• 4
▪ 5 (Choice A) Alloimmune hemolytic disease is caused by hemolysis of fetal red blood cells due to ABO or Rh
▪ 6 incompatibility ABO hemolytic disease affects almost exclusively infants with blood types A or B who are
▪ 7 born to mothers with type 0 blood. Routine testing for infant blood type is unnecessary as ABO and Rh
▪ 8 incompatibility is unlikely to occur with maternal A+ blood type.
▪ 9
• 10 (Choice B) Biliary atresia presents with conjugated hyperbilirubinemia (direct bilirubin >20% of total bilirubin)
• 11 in the first 2 months of life. Other findings include clay-colored stools, dark urine, and an enlarged liver, none
• 12 of which are present in this infant
• 13
• 14 (Choice C) Breast milk jaundice can be distinguished from breastfeeding failure jaundice if the infant has no
• 15 signs of dehydration or feeding problems, making this diagnosis unlikely.
• 16
• 17 (Choice El Dubin-Johnson syndrome is a rare autosomal recessive condition characterized by conjugated
• 18 hyperbilirubinemia, which this infant does not have. Affected individuals are usually asymptomatic except for
• 19 mild scleral icterus.

(Choice F) Galactosemia is a rare cause of conjugated hyperbilirubinemia due to galactose-1-phosphate


uridyl transferase deficiency_ In addition to jaundice, neonates develop vomiting and growth failure due to
• 22
inability to digest galactose in breast milk or cow's milk-based formulas.
• 23
• 24
Educational objective:
• 25
Breastfeeding failure jaundice manifests as unconjugated hyperbilirubinemia and dehydration in the first week
• 26
of life_ Inadequate stooling results in suboptimal bilirubin elimination and increased enterohepatic circulation of
• 27
bilirubin.
• 28
• 29
References:
• 30
• 31 1. Breastfeeding and jaundice
• 32
2. Neonatal jaundice and human milk.
• 33
• 34 3. Hyperbilirubinemia in the newborn infant > or =35 weeks' gestation: an update with
• 35 clarifications.
• 36 4. Management of hyperbilirubinemia in the newborn infant 35 or more weeks of gestation.
• 37
• 38
• 39
Copyright © UWorld Last updated: [11/6/2014]
• 40
• 41
• 42 End Block
Feedback
• -1
• 2 Item: 20 of 44 .111M 'Mark
▪ 3 Previous Next Lab Values Notes Calculator
• 4- 141edia Exhibit
▪ 5
▪ 6 Urate crystals secondary to dehydration
▪ 7
▪ 8
▪ 9
• 10
• -1 -1
• 12
• 13
• 14
• 15
• 16
• 17
• 18
• 19

21
• 22
• 23
• 24
• 25
• 26
• 27
• 28
• 29
• 30
• 31
• 32
• 33
• 34
• 35
• 36
• 37
• 38
• 39
• 40


41
42
Feedback
a
End Block
• 1
• 2 Item: 21 of 44 V-Mark
▪ 3 Previous . Next Lab Values. Notes Calculator.
• 4-
▪ 5
▪ 6 Item 2 of 2
▪ 7
▪ 8 Which of the following is the best next step in management of this patient?
▪ 9
• 10
A. Administer Rho(D) immune globulin to the infant's mother [1%]
• 11
• 12 B. Ereastfeed 15 minutes per side every 2-3 hours [64%]
• 13 C. Continue current feeding regimen, follow-up in 1 week [13%]
• 14
D. Phototherapy and exchange transfusion [11 %]
• 15
E Switch to a cow's milk-based formula [6°4]
• 16 •
- 17 • Switch to soy formula [5%]
- 18
- 19
Explanation:
Bilirubin metabolism
• 22
• 23
• 24 • Hemoglobin
• 25
• •
• catabolism
• 26
• 27 4
• 28 Biliru bin
• 29
• 30
• 31
Unconjugatecl
• 32
bilirubin + albumin
• 33
• 34
• 35
• 36
• 37
Hepatic artery
• 38
• 39 Conjugation by hepatic uridine
• 40 dipho-phooluconurate
• 41
• 42 End Block
Feedback
• 1
• 2 Item: 21 of 44 F'Mark .4111°.
▪ 3 Previous Next Lab Values. Notes Calculator.

• 4-
▪ 5 Bilirubin metabolism
▪ 6
▪ 7
Hemoglobin
▪ 8 •• •
• catabolism
▪ 9
• 10
• -1 -1
• 12 Bilirubin
• 13
Jr
• 14
• 15 Unconjugated
bilirubin + albumin
• 16 •
- 17
- 18
- 19
Hepatic artery

• 22 Conjugation by hepatic uridine


• 23 diphosphogluconurate
• 24
glucuronosyltransferase
• 25
• 26
• 27
• 28
• 29
• 30
• 31 Portal vein
• 32
• 33
• 34 Conjugated
• 35 bilirubin
• 36
• 37
Enterahepatic
• 38
• 39
recycling of
• 40 un conjugated se
• 41
• 42 End Block
Feedback
• 1
• 2 Item: 21 of 44
▪ 3 Previous Next Lab Values. Notes Calculator.
• 4-
▪ 5 Unconjugated
▪ 6 bilirubin + albumin
▪ 7
▪ 8
▪ 9
• 10 Hepatic artery
• -1 -1
• 12 Conjugation by hepatic uridine
• 13 diphosphogluconurate
• 14
g Li curonosylt ra nsferase
• 15
• 16 •
- 17
- 18
- 19

Portal vein
• 22
• 23
• 24 Conjugated
• 25
bilirubin
• 26
• 27
Enterohepatic
• 28
recycling of
• 29
• 30 nconjugated
• 31 bilirubin
• 32
• 33
• 34
• 35
• 36
• 37
Deconjugation Reduction
• 38
by intestinal 0- by bacterial
• 39
g lucuronidase dehydrogenase se
• 40
• 41
• 42 End Block
Feedback
• 1
• 2 Item: 21 of 44 1- Mark
▪ 3 Previous Next Lab Values. Notes Calculator.

• 4-
▪ 5
▪ 6
▪ 7
Deconjugation Reduction
▪ 8
by intestinal 13- by bacterial
▪ 9
glucuronldase dehydrogenase
• 10
• 11
• 12
• 13
• 14
• 15
• 16 •
- 17
- 18 Fecal urobilinogen
CI Li SJALEWeild. LLC
- 19

All newborns have mild unconjugated hyperbilirubinemia due to high hemoglobin turnover and immature
• 22 hepatic uridine diphosphogluconurate glucuronosyltransferase activity. Bilirubin is excreted primarily through
• 23 the stool, and inadequate feeding can result in bilirubin accumulation.
• 24
The best treatment for breastfeeding failure jaundice in otherwise healthy full-term newborns is to increase the
• 25
frequency and duration of feeds to stimulate milk production, maintain adequate hydration, and promote
• 26
bilirubin excretion. Neonates should breastfeed —8-12 times a day (every 2-3 hours) for MG-20 minutes per
• 27
breast during the first month of life_ When bilirubin is eliminated as fecal urobilinogen, less bilirubin is
• 28
reabsorbed and recycled in the enterohepatic circulation, thereby decreasing serum bilirubin levels_
• 29
• 30
This infant should be closely monitored to ensure that he is being fed adequately and that his bilirubin level is
• 31
decreasing. If the bilirubin continues to rise despite efforts to optimize lactation, formula supplementation may
• 32
be necessary (Choice Ey However, discontinuing breastfeeding and switching to formula will further
• 33
decrease the mother's milk supply and prevent the infant from receiving the benefits of breastfeeding, which
• 34
include improved infant immunity and mother-infant bonding.
• 35
• 36 (Choice A) All Rh-negative mothers should receive Rho(D) immune globulin during pregnancy to prevent Rh-
• 37 antibody hemolytic disease of the newborn. This mother is Rh-positive and does not require Rho(D) immune
• 38 globulin.
• 39
• 40 frhnirtp Rerai ISP this infa nt is milrilv riphvrirtpri fppriinn frpni ienry shniilri he inr rpspri anri he shniilri
se
• 41
• 42 End Block
Feedback
• 1
• 2 Item: 21 of 44 F' Mark
▪ 3 Previous Next Lab Values. Notes Calculator.
• 4-
The best treatment for breastfeeding failure jaundice in otherwise healthy full-term newborns is to increase the
▪ 5
frequency and duration of feeds to stimulate milk production, maintain adequate hydration, and promote
▪ 6
bilirubin excretion. Neonates should breastfeed —8-12 times a day (every 2-3 hours) for 10-21:1 minutes per
▪ 7
breast during the first month of life_ When bilirubin is eliminated as fecal urobilinogen, less bilirubin is
▪ 8
reabsorbed and recycled in the enterohepatic circulation, thereby decreasing serum bilirubin levels_
▪ 9
• 10
This infant should be closely monitored to ensure that he is being fed adequately and that his bilirubin level is
• 11
decreasing. If the bilirubin continues to rise despite efforts to optimize lactation, formula supplementation may
• 12 be necessary (Choice E). However, discontinuing breastfeeding and switching to formula will further
• 13
decrease the mother's milk supply and prevent the infant from receiving the benefits of breastfeeding, which
• 14
include improved infant immunity and mother-infant bonding.
• 15
• 16 • (Choice A) All Rh-negative mothers should receive Rho(D) immune globulin during pregnancy to prevent Rh-
- 17 antibody hemolytic disease of the newborn. This mother is Rh-positive and does not require Rho(D) immune
- 18 globulin.
- 19
(Choice C) Because this infant is mildly dehydrated, feeding frequency should be increased and he should
be reevaluated within 2 days. If the unconjugated bilirubin levels continue to rise, the infant is at increased risk
• 22 of kernicterus and devastating neurologic consequences; waiting a week is too long.
• 23
• 24 (Choice ID) The threshold for phototherapy in a full-term, healthy 4-day-old infant is a total bilirubin level X20
• 25 mg/I:IL based on the American Academy of Pediatrics bilirubin nomogram. Exchange transfusion is reserved
• 26 for infants with levels mg/dL or those with bilirubin-induced neurologic dysfunction_
• 27
• 28 (Choice F) Galactosemia is an absolute contraindication to breast milk or cow's milk-based formula. and soy
• 29 formula should be fed to these patients. Soy formula is inappropriate for breastfeeding failure jaundice.
• 30
Educational objective:
• 31
Treatment of breastfeeding failure jaundice consists of optimizing lactation and increasing breastfeeding
• 32
frequency. If the mother's milk supply is inadequate, supplementation with a cow's milk-based formula can be
• 33
considered, but breastfeeding should not be discontinued.
• 34
• 35
• 36 References:
• 37 1. Breastfeeding and jaundice
• 38
• 39 2. Neonatal jaundice and human milk.
• 40 Hvnprhiliruhinpmi2 in the nawhnrn infant nr =35 wppkc' ripct2tirin• n linr12tp with
• 41
• 42 End Block
Feedback
• 1
• 2 Item: 21 of 44 V-Mark
▪ 3 Previous Next Lab Values. Notes Calculator.
• 4- This infant should be closely monitored to ensure that he is being fed adequately and that his bilirubin level is
▪ 5 decreasing. If the bilirubin continues to rise despite efforts to optimize lactation, formula supplementation may
▪ 6 be necessary (Choice Ey However, discontinuing breastfeeding and switching to formula will further
▪ 7 decrease the mother's milk supply and prevent the infant from receiving the benefits of breastfeeding, which
▪ 8 include improved infant immunity and mother-infant bonding.
▪ 9
• 10 (Choice A) All Rh-negative mothers should receive Rho(D) immune globulin during pregnancy to prevent Rh-
• 11 antibody hemolytic disease of the newborn. This mother is Rh-positive and does not require Rho(D) immune
• 12 globulin.
• 13
• 14 (Choice C) Because this infant is mildly dehydrated, feeding frequency should be increased and he should
• 15 be reevaluated within 2 days. If the unconjugated bilirubin levels continue to rise, the infant is at increased risk
of kernicterus and devastating neurologic consequences; waiting a week is too long.
• 16 •
- 17
- 18 (Choice 13) The threshold for phototherapy in a full-term, healthy 4-day-old infant is a total bilirubin level X20
- 19 mg/dL based on the American Academy of Pediatrics bilirubin nomogram. Exchange transfusion is reserved
for infants with levels X25 mg/dL or those with bilirubin-induced neurologic dysfunction.

(Choice F) Galactosemia is an absolute contraindication to breast milk or cow's milk-based formula, and soy
• 22
formula should be fed to these patients. Soy formula is inappropriate for breastfeeding failure jaundice_
• 23
• 24
Educational objective:
• 25
Treatment of breastfeeding failure jaundice consists of optimizing lactation and increasing breastfeeding
• 26
frequency. If the mother's milk supply is inadequate, supplementation with a cow's milk-based formula can be
• 27
considered, but breastfeeding should not be discontinued.
• 28
• 29 1
References:
• 30
• 31 1. Breasffeeding and jaundice
• 32
2. Neonatal jaundice and human milk.
• 33
• 34 3. Hyperbilirubinemia in the newborn infant > or =35 weeks' gestation: an update with
• 35 clarifications.
• 36 4. Management of hyperbilirubinemia in the newborn infant 35 or more weeks of gestation.
• 37
• 38
• 39
Copyright © UWorld Last updated: [11/6/2014]
• 40
• 41
• 42 End Block
Feedback
1
2 Item: 22 of 44 F' Mark U
.P
3 Previous Next Lab Values, Notes Calculator.
4-
5
6 A 5-year-old boy is brought to the physician for evaluation of left hip pain after his parents saw him limping this
7 morning. He spent time on the playground yesterday but his parents do not think he was injured then_ The
8 boy complains that the pain in his hip worsens when he moves or walks_ For the past 3 days, he has had a
9 runny nose and congestion. Review of systems is otherwise negative. His grandmother has rheumatoid
10 arthritis treated with methotrexate. His temperature is 37_2° C (99° F), blood pressure is 100/65 mm Hg,
11 pulse is 92/min, and respirations are 18/min. Physical examination shows a well-appearing child with clear
12 rhinorrhea and intermittent dry cough. Lungs are clear to auscultation bilaterally_ His left hip is slightly
13 abducted and externally rotated with mildly decreased range of motion. He is able to stand and bear
14 weight The remainder of his examination is normal_ X-rays of both hips are normal. His laboratory results
15 are as follows:
16
Complete blood count
17
Hemoglobin 12.5 WI:IL
18
Platelets 281,000/mm3
19
Leukocyte count 8,500/mm3
Neutrophils 30%
Eosinophils 1%
Lymphocytes 64%
• 23
Monocytes 5%
• 24
Erythrocyte sedimentation rate 3G mm/h
• 25
C-reactive protein 9 mg/L (N: s 8 mg/L)
• 26
• 27
Which of the following is the best next step in management of this patient?
• 28
• 29
• 30 A. Ibuprofen_ rest, and follow-up in 1 week [53%]
• 31 B. Intravenous antibiotics [3%]
• 32
C. Magnetic resonance imaging of the left hip [21%]
• 33
• 34 ID. Serum antinuclear antibodies [5%]
• 35 E Synovial fluid aspiration of the left hip [18%]
• 36
• 37
• 38 Explanation:
• 39
• 40 Transient synovitis is the most common cause of hip pain in children, typically occurring in boys age 3-10 V



41
42
Feedback
a
End Block
• 1
• 2 Item: 22 of 44 V- Mark
▪ 3 Previous Next Lab Values. Notes Calculator.
• 4-
5
Transient synovitis is the most common cause of hip pain in children, typically occurring in boys age 3-10
▪ 6
years. The cause is unknown but usually follows a viral infection or mild trauma. Synovial inflammation leads
▪ to pain, decreased range of motion, and limping. On examination, the affected hip is typically flexed, slightly
▪ 8
7
abducted, and externally rotated. This position maximizes the joint space, thereby providing some pain relief_
▪ 9
Because characteristics of transient synovitis overlap with septic arthritis, laboratory studies should be sent to
• 10
assess for severity of inflammation. In contrast to septic arthritis, children with transient synovitis rarely have
• 11
fever or significant laboratory abnormalities_
• 12
• 13
• 14 Features of septic arthritis of the hip
• 15
• 16
• 17 • Fever 138.5°C (101°F)
• 18 • Inability to bear weight
• 19
• White blood cell count >12r000imm3
• Erythrocyte sedimentation rate >40 mm/h
• L-reactive protein >260 mgidL (20 rn9/1)
• 23
• 24 1.15,64LEViorld. LLC
• 25
• 26
Patients who are ill-appearing, febrile, or have >3 or 4 findings shown in the table should undergo immediate
• 27
arthrocentesis (Choice E) and intravenous antibiotics (Choice 6). This child is well-appearing, able to bear
• 28
weight, and has no concerning laboratory findings. making invasive arthrocentesis and intravenous antibiotics
• 29
unnecessary.
• 30
• 31 Plain radiographs should be done to exclude bony lesions, fractures, and Legg-Calve-Perthes disease (LCP).
• 32 Anteroposterior and frog-leg lateral views of both hips should be obtained to compare affected and normal
• 33 sides for subtle changes. Additional workup is unnecessary unless symptoms persist or worsen. Treatment
• 34 consists of rest and nonsteroidal anti-inflammatory medications (NSAIDs). NSAIDs (eg, ibuprofen) have
• 35 both analgesic and anti-inflammatory properties and are recommended over other pain relievers (eg,
• 36 acetaminophen, opioids). The exception is aspirin, which should be avoided due to the risk of Reye
• 37 syndrome. Children usually recover within 1-4 weeks and have no complications.
• 38
• 39 (Choice C) If symptoms persist or worsen, a diagnosis of LP should be reconsidered as initial radiographs
• 40 mar ann P ar nrirrn1 in rimiQnnnrci imaninn ran riatort P so
• 41
42 I End Block
Feedback
• 1
• 2 Item: 22 of 44 A V-Mark
▪ 3 Previous Next Lab Values. Notes Calculator.
• 4- 0 LISHLEWDriii, LLC
▪ 5
▪ 6
Patients who are ill-appearing, febrile, or have >3 or 4 findings shown in the table should undergo immediate
▪ 7
arthrocentesis (Choice E) and intravenous antibiotics (Choice B). This child is well-appearing, able to bear
▪ 8
weight, and has no concerning laboratory findings, making invasive arthrocentesis and intravenous antibiotics
▪ 9
unnecessary.
• 10
• 11 Plain radiographs should be done to exclude bony lesions, fractures, and Legg-Calve-Perthes disease (LCP).
• 12 Anteroposterior and frog-leg lateral views of both hips should be obtained to compare affected and normal
• 13 sides for subtle changes. Additional workup is unnecessary unless symptoms persist or worsen. Treatment
• 14 consists of rest and nonsteroidal anti-inflammatory medications (N AIDS}_ NSAIDs (eg, ibuprofen) have
• 15 both analgesic and anti-inflammatory properties and are recommended over other pain relievers (eg,
• 16 acetaminophen, opioids). The exception is aspirin, which should be avoided due to the risk of Reye
• 17 syndrome. Children usually recover within 1-4 weeks and have no complications.
• 18
• 19 (Choice C) If symptoms persist or worsen, a diagnosis of LCP should be reconsidered as initial radiographs
may appear normal in early disease. Magnetic resonance imaging can detect early LCP as well as marrow
changes suggestive of osteomyelitis. This scan is very expensive, often requires sedation in young children,
and is not warranted for this patient at this time_
• 23
6 24 (Choice ID) Pauciarticular-onset juvenile idiopathic arthritis or pauciarthritis is the most common subgroup of
6 25 juvenile idiopathic arthritis. Serum antinuclear antibodies are usually the only laboratory abnormality_ Although
• 26 pauciarthritis can present with a morning limp, it typically occurs in female toddlers and rarely involves the hip,
• 27 making this diagnosis unlikely.
• 28
• 29 Educational objective:
• 30 Transient synovitis is the most common cause of hip pain in children and is treated with rest and ibuprofen.
• 31 There are usually no laboratory abnormalities or fever. Bilateral hip x-rays should be obtained to assess for
• 32 Legg-Calve-Perthes disease.
• 33
• 34 References:
• 35
1. Factors distinguishing septic arthritis from transient synovitis of the hip in children. A
• 36
prospective study.
• 37
• 38 2. A clinical practice guideline for treatment of septic arthritis in children: efficacy in
• 39 improving process of care and effect on outcome of septic arthritis of the hip.
• 40 1•••••••01"1..; a* alai.. ...a. awFm ml I;..,..- ;101i FILL. Rol mob a.
• 41
• 42 End Block
Feedback
• 1
• 2 Item: 22 of 44 F' Mark
▪ 3 Previous Next Lab Values. Notes Calculator.
• 4
▪ 5 Plain radiographs should be done to exclude bony lesions, fractures, and Legg-Calve-Perthes disease (LCP).
▪ 6 Anteroposterior and frog-leg lateral views of both hips should be obtained to compare affected and normal
▪ 7 sides for subtle changes. Additional workup is unnecessary unless symptoms persist or worsen. Treatment
▪ 8 consists of rest and nonsteroidal anti-inflammatory medications (N AIDS}_ NSAIDs (eg, ibuprofen) have
▪ 9 both analgesic and anti-inflammatory properties and are recommended over other pain relievers (eg,
• 10 acetaminophen, opioids). The exception is aspirin, which should be avoided due to the risk of Reye
• 11 syndrome. Children usually recover within 1-4 weeks and have no complications.
• 12
• 13 (Choice C) If symptoms persist or worsen, a diagnosis of LCP should be reconsidered as initial radiographs
• 14 may appear normal in early disease. Magnetic resonance imaging can detect early LCP as well as marrow
• 15 changes suggestive of osteomyelitis. This scan is very expensive, often requires sedation in young children,
• 16 and is not warranted for this patient at this time.
• 17
(Choice ID) Pauciarticular-onset juvenile idiopathic arthritis or pauciarthritis is the most common subgroup of
• 18
juvenile idiopathic arthritis. Serum antinuclear antibodies are usually the only laboratory abnormality_ Although
• 19
pauciarthritis can present with a morning limp, it typically occurs in female toddlers and rarely involves the hip,
making this diagnosis unlikely.

Educational objective:
• 23
Transient synovitis is the most common cause of hip pain in children and is treated with rest and ibuprofen.
• 24
There are usually no laboratory abnormalities or fever. Bilateral hip x-rays should be obtained to assess for
• 25
Legg-Calve-Perthes disease.
• 26
• 27
References:
• 28
• 29 1. Factors distinguishing septic arthritis from transient synovitis of the hip in children. A
• 30 prospective study.
• 31
2. A clinical practice guideline for treatment of septic arthritis in children: efficacy in
• 32
improving process of care and effect on outcome of septic arthritis of the hip.
• 33
• 34 3. Transient synovitis as a cause of painful limps in children.
• 35 4. Validation of a clinical prediction rule for the differentiation between septic arthritis and
• 36 transient synovitis of the hip in children.
• 37
• 38
• 39
Copyright © UWorld Last updated: [1/12/2015]
• 40
• 41
• 42 End Block
Feedback
• -1
• 2 Item: 22 of 44 .111M Mark
11
.

▪ 3 Previous Next Lab Values Notes Calculator


• 4- Media Exhibit
▪ 5
▪ 6 X-ray Legg-Calve-Perthes disease
▪ 7
▪ 8
▪ 9
• 10
• -1 -1
• 12
• 13
• 14
• 15
• 16
• 17
• 19
• 19

6 23
6 24
6 25
• 26
• 27
• 28
• 29
• 30
• 31
• 32
• 33
• 34
• 35
• 36
• 37 V
• 39
• 39
• 40


41
42
Feedback
a
End Block
• -1
• 2 Item: 22 of 44 .111 II Mark
11
.
▪ 3 Previous Next Lab Values Notes Calculator
• 4- Media Exhibit A
▪ 5
▪ 6 X-ray Legg-Calve-Perthes disease
▪ 7
▪ 8
▪ 9
• 10
• -1-1
• 12
• 13
• 14
• 15
• 16
• 17
• 19
• 19

623
6 24
6 25
• 26
• 27
• 28
• 29
• 30
• 31
• 32
• 33
• 34
• 35
• 36
• 37 V
• 39
• 39
• 40
• 41
• 42
• 1
• 2 Item: 22 of 44 IM V-Mark
• 3 6..11 Previous Next Lab Values Notes Calculator
4 Media Exhibit
• 5
▪ 6 Osteomyelitis MRI
▪ 7
▪ 8
▪ 9
• 10
• 11
• 12
• 13
• 14
• 15
• 16
• 17
• 19
• 19
• 20
• 21

• 23
6 24
6 25
• 26
• 27
• 28
• 29
• 30
• 31
• 32
• 33
• 34
• 35
• 36
• 37
• 39
• 39
• 40 dl
• 4-1
• 42 End Block
Feedback
• -1
• 2 FF. 11
.
▪ 3 Previous Next Lab Values Notes Calculator
• 4- Media Exhibit
▪ 5
▪ 6 Osteomyelitis MRI
▪ 7
▪ 8
▪ 9
• 10
• -1 -1
• 12
• 13
• 14
• 15
• 16
• 17
• 19
• 19
• 20
• 21

6 23
6 24
6 25
• 26
• 27
• 28
• 29
• 30
• 31
• 32
• 33
• 34
• 35
• 36
• 37
• 38
• 39
• 40
• 41
• 42
• 1
• 2 Item: 23 of 44 V-Mark
▪ 3 Previous Next Lab Values. Notes Calculator.
• 4-
▪ 5
▪ 6 A 7-year-old boy is brought to the clinic due to malaise and headache for the past 2 weeks. Three weeks ago,
▪ 7 he went on a hiking trip with his family during which his mother found a tick attached to his thigh. The mother
▪ 8 removed the tick with tweezers, and a red ''bump" remained at the site where the tick was attached. One
▪ 9 week later, a red ring developed around the "bump" and has since grown larger. The rash is slightly itchy but
• 10 not painful_ The patient has a history of eczema treated intermittently with topical hydrocortisone. Last year,
• 11 he had tines pedis that resolved with terbinafine. Examination shows a nontender rash, approximately 8 cm
• 12 in diameter, on his right thigh. The remainder of his examination is normal. What is the most appropriate next
• 13 step in management of this patient?
• 14
• 15
• 16
A. Intravenous ceftriaxone [€%]
• 17 • B. Oral amoxicillin [35%]
• 18
C. Oral doxycycline [57%]
• 19
20
D. Oral itraconazole [1%]
21 E Topical hydrocortisone [1%]
▪ 22
23
• 24 Explanation:
• 25
This patient has erythema chronicum migrans, the classic rash that is present in most patients with early
• 26
localized Lyme disease. The rash is not particularly painful but may occasionally burn or itch. Many patients
• 27
also experience nonspecific constitutional symptoms such as headache, malaise, fatigue, and fever. The
• 28
causative agent is the spirochete Borrefia burgdorferi, which is spread to humans by the bite of an infected
• 29
&odes tick. Diagnosis of early localized Lyme disease is based solely on the presence of the trademark rash
• 30
in the context of recent travel to Lyme-endemic areas.
• 31
• 32
Clinical trials have shown that oral doxycycline, amoxicillin, and cefuroxime have equivalent efficacy for
• 33
treating early localized Lyme disease. Doxycycline is often used because it is effective in treating potential
• 34
coexisting Anaplasrna phagocytoplrikun infection, which is also transmitted by the ixodes tick_ However,
• 35
doxycycline is contraindicated in children age <8 years and pregnant women. It can slow bone growth in
• 36
exposed fetuses and cause enamel hypoplasia and permanent teeth stains during tooth development in young
• 37
children_ Therefore, oral amoxicillin or cefuroxime is the treatment of choice for children age <8 years and
• 39
pregnant women.
• 39
• 40 f 1r+ 6. An AN. AN. A% .1.1-11-1 Pa, i • NA- L-1. la-La-1 le
v
• 41
• 42 End Block
Feedback
-1

2 Item: 23 of 44 V-Mark
EF. 11
.

3 Previous Next Lab Values. Notes Calculator


4- 141edia Exhibit
ii
▪ 5
Media 1 of 1

▪ 8
▪ 9
• 10
• -1 -1
• 12
• 13
• 14
• 15
• 16
• 17
• 19
• 19
20
21
• 22
23
• 24
• 25
• 26
• 27
• 28
• 29
• 30
• 31
• 32
• 33
• 34
• 35
• 36
• 37
• 39
• 39
• 40


41
42
Feedback
a
End Block
-1

2 Item: 23 of 44 V-Mark
EF. 11
.
3 Previous Next Lab Values. Notes Calculator
4- 141edia Exhibit Eic II
▪ 5
▪ 6 Media 1 of 1
▪ 7
▪ 8
▪ 9
• 10
• 11
• 12
• 13
• 14
• 15
• 16
• 17
• 19
• 19
20
21
• 22
23
• 24
• 25
• 26
• 27
• 28
• 29
• 30
• 31
• 32
• 33
• 34
• 35
• 36
V
• 37
• 39
• 39
• 40


41
42
Feedback
a
End Block
• 1
• 2 Item: 23 of 44 V-• Mark
▪ 3 Previous Next Lab Values. Notes Calculator.
• 4
Explanation:
▪ 5
▪ 6 This patient has erythema chronicum migrans, the classic rash that is present in most patients with early
▪ 7 localized Lyme disease. The rash is not particularly painful but may occasionally burn or itch. Many patients
▪ 8 also experience nonspecific constitutional symptoms such as headache, malaise, fatigue, and fever. The
▪ 9 causative agent is the spirochete Borrefia burgdorferi, which is spread to humans by the bite of an infected
• 10 Ixodes tick. Diagnosis of early localized Lyme disease is based solely on the presence of the trademark rash
• 11 in the context of recent travel to Lyme-endemic areas.
• 12
• 13 Clinical trials have shown that oral doxycycline, amoxicillin, and cefuroxime have equivalent efficacy for
• 14 treating early localized Lyme disease. Doxycycline is often used because it is effective in treating potential
• 15 coexisting Anaplasrna phagocytophilurn infection, which is also transmitted by the Ixodes tick_ However,
• 16 doxycycline is contraindicated in children age <8 years and pregnant women. It can slow bone growth in
• 17 exposed fetuses and cause enamel hypoplasia and permanent teeth stains during tooth development in young
• 18 children. Therefore, oral amoxicillin or cefuroxime is the treatment of choice for children age <8 years and
• 19 pregnant women.
20
21 (Choice A) Intravenous ceftriaxone is reserved for Lyme meningitis and heart block. manifestations of early
• 22 disseminated Lyme disease. This patient has an otherwise normal examination and does not warrant
23 hospitalization for intravenous antibiotics_
• 24
• 25 (Choices D and E) Topical corticosteroids are used for nummular dermatitis while topical antifungals are
• 26 the first-line treatment for Linea corporis. Although these are also annular rashes. they are typically scaly and
• 27 pruritic and are not associated with tick bites.
• 28
Educational objective:
• 29
Erythema chronicum migrans is pathognomonic for early localized Lyme disease. Amoxicillin is the treatment
• 30
of choice in children age <8 years.
• 31
• 32
33 References:
34 1. The clinical assessment, treatment, and prevention of Lyme disease, human granulocytic
• 35 anaplasmosis, and babesiosis: clinical practice guidelines by the Infectious Diseases
• 36 Society of America.
• 37
• 38
• 39
Copyright © UWorld Last updated: [9/2/2014]
• 40
• 41
• 42 End Block
Feedback
• -1
• 2 Item: 23 of 44 V- Mark
EF. 11
.
▪ 3 Previous Next Lab Values Notes Calculator
• 4- Media Exhibit
▪ 5
▪ 6 Nummular dermatitis
▪ 7
▪ 8
▪ 9
• 10
• -1-1
• 12
• 13
• 14
• 15
• 16
• 17
• 19
• 19
20
21

• 24
• 25
• 26
• 27
• 23
• 29
• 30
• 31
• 32
• 33
• 34
• 35
• 36
• 37
• 39
• 39
• 40 ii
• 41
• 42 End Block
Feedback
• 1 A
• Item: 23 of 44 P 1'2 ilark 117
.fr
4ii
Previous Next Lab Values, Notes Calculator
▪ 3
• 4 Media Exhibit
▪ 5
• 6 Tinea corporis
▪ 7
8
▪ 9
• 10
• 11
• 12
• 13
• 14
• 15
• 16
• 17
• 18
• 19
20
21

• 23
24
25
26
27
28
29
30
31
32
33
34
• 35
• 36
• 37
• 38
• 39
• 40 dl
• 41
F
• 42 Feedback End Block
_ A '7
• 1
• 2 Item: 24 of 44 V-Mark
▪ 3 Previous Next Lab Values. Notes Calculator.
• 4-
6
▪ 6 A 34-year-old woman who describes herself as a "holistic healer" brings her two children aged four years and
▪ 7 three years to the pediatrician for routine physical examinations_ This is a first visit as they have recently
▪ 8 moved from another state. The medical records for the children indicate that no immunizations have been
▪ 9 given_ When the mother is questioned about this, she proudly replies, ''My children are wonderfully healthy on
• 10 their own and have no need for these artificial vaccines." The principles and benefits of immunization are
• 11 discussed at length, as well as the risks inherent in not being immunized. The physician recommends that the
• 12 children be given all age-appropriate vaccinations today. The mother remains convinced that immunizations
• 13 cause more harm than good, and she steadfastly refuses to allow her children to be vaccinated. What is the
• 14 most appropriate next step?
• 15
• 15
A. Obtain a court order for immunization of children [5%]
• 17
• 18 B. Proceed with immunizations today [3%]
• 19 C. Inform mother that she will be reported to the local health department [4%]
20
D. Document in the medical chart that the risks and benefits of vaccination have been explained [80%]
21
• 22 E Request to speak with her husband [7%]
• 23
• 24
Explanation:
• 25
• 25
In the medical community, the consensus is that all children should be immunized against major diseases
• 27
unless a specific contraindication (eg, allergy to vaccine components) exists. It is estimated that over 90% of
• 28
children in the United States have received all major childhood vaccinations by the time they enter school, and
• 29
public health personnel continue in their efforts to increase this number. Fortunately, however, the risk to
• 30
those who have not been vaccinated remains minimal. This is due in part to the presence of ''herd immunity,"
• 31
whereby the disease resistance of the majority confers protection upon the disease susceptible minority.
• 32
Therefore, the initial step in this case would be to explain the potential risks and benefits of vaccination to the
• 33
mother. If she persists in refusing to have her children vaccinated, her wishes should be respected.
• 34
Documentation of such a discussion in the medical chart is imperative.
• 35
• 35
(Choice A) There is no need to obtain a court order in this instance, as there is a very low risk that the child
• 37 will develop one of the diseases he is not immunized against_
• 38
• 39 (Choice B) Immunizing the children without consent from the mother would be appropriate only if they were in
• 40 imminent runner
• 41
42 I End Block

Feedback
• 1
• 2 Item: 24 of 44 V-Mark
▪ 3 Previous Next Lab Values. Notes Calculator.
• 4
▪ 5
B. Proceed with immunizations today [3%]
▪ 6 • C. Inform mother that she will be reported to the local health department [4%]
▪ 7
• D. Document in the medical chart that the risks and benefits of vaccination have been explained [80%]
▪ 8
▪ 9
E Request to speak with her husband [1%]
• 10
• 11
Explanation:
• 12
• 13 In the medical community, the consensus is that all children should be immunized against major diseases
• 14 unless a specific contraindication (eg, allergy to vaccine components) exists. It is estimated that over 90% of
• 15 children in the United States have received all major childhood vaccinations by the time they enter school, and
• 16 public health personnel continue in their efforts to increase this number. Fortunately, however, the risk to
• 17 those who have not been vaccinated remains minimal. This is due in part to the presence of "herd immunity,'
• 18 whereby the disease resistance of the majority confers protection upon the disease susceptible minority.
• 19 Therefore, the initial step in this case would be to explain the potential risks and benefits of vaccination to the
20 mother. If she persists in refusing to have her children vaccinated, her wishes should be respected.
21 Documentation of such a discussion in the medical chart is imperative.
• 22
• 23 (Choice A) There is no need to obtain a court order in this instance, as there is a very low risk that the child
• 24 will develop one of the diseases he is not immunized against
• 25
• 26 (Choice B) Immunizing the children without consent from the mother would be appropriate only if they were in
• 27 imminent danger.
• 28
• 29 (Choice C) Threatening to report the mother to the local health authorities is inappropriate. While health
• 30 departments are interested in recording the incidence of specific conditions (eg, measles), they do not track
• 31 or enforce immunization against such diseases.
• 32
(Choice E) Insisting upon speaking with the father is inappropriate. The mother's wishes should be respected
33
and documented in the medical chart
34
• 35
Educational Objective:
• 36
If no significant harm is likely to result from withholding therapy, parental wishes regarding the medical care of
• 37
a child should be honored and the discussion documented in the chart.
• 38
• 39
Copyright © UWorld Last updated: [10/14/2014]
• 40
• 41
• 42 End Block
Feedback
1
2 Item: 25 of 44 V-Mark
3 Previous Next Lab Values, Notes Calculator.
4-
5
6 An 18-year-old woman comes to the pediatrician with her 3-day-old daughter for her first newborn visit and
7 has questions about breastfeeding. She has been breastfeeding exclusively, but says that it is painful and
8 believes that she is not producing enough milk. She would like to stop breastfeeding and switch to formula.
9 Which of the following statements regarding breastfeeding is correct?
10
11
A. Breastfeeding is associated with increased risk of obesity in the infant [0%]
12
13 B. Breastfeeding reduces the risk of endometrial cancer in the mother [14%]
14 ▪ C. Ereastfeeding reduces the risk of otitis media in the infant [15%]
15
ID. Nutritional supplementation is not needed with exclusive breastfeeding [6°,6]
16
17 E There is no need for contraception with exclusive breastfeeding [5%]
18
19
Explanation:
20
21
• 22
Breastfeeding benefits & contraindications
• 23
• 24

Benefits Contraindications
• 26
• 27
• 28
• More rapid uterine involution & • Active untreated tuberculosis (mothers
• 29
• 30 decreased postpartum bleeding may breastfeed after 2 weeks of anti-
• 31 • Faster return to prepartum weight tuberculin therapy)
• 32 • Improved child spacing • Maternal HIV infection (in developed
• 33
• Improved maternal-infant bonding countries where formula is readily
• 34
• 35 • Reduced risk of breast & ovarian available)
• 36 Materna cancer • Herpetic breast lesions
• 37
• Varicella infection <5 days prior to or
• 38
• 39 within 2 days of delivery
• 40 Cnarifir rrin+..nrrinl nridarlit-=1-if-kric vl


41
42
Feedback
a
End Block .
2 Item: 25 of 44 F'Mark
3 Previous Next Lab Values, Notes Calculator.
4-
5 Brea stfeeding benefits & contraindications
6
7
8 Benefits Contraindications
9
10
-1 -1 • More rapid uterine involution & • Active untreated tuberculosis (mothers
12
decreased postpartum bleeding may breastfeed after 2 weeks of anti-
13
14 • Faster return to prepartum weight tuberculin therapy)
15 • In child spacing • Maternal HIV infection (in developed
16
• Improved maternal-infant bonding coil ntrieswhere formula is readily
17
19
• Reduced risk of breast & ovarian available)
19 Maternal cancer • Herpetic breast lesions
20 • Varicella infection <5 days prior to or
21
• 22
within 2 days of delivery
• 23 • Specific maternal medications
• 24 • Chemotherapy or ongoing radiation
therapy
• 26
• 27 • Active abuse of street drugs or alcohol
• 28
• 29
• 30 • Improved immunity • Galactosemia
• 31 • Improved gastrointestinal function
• 32
Prevention of infectious diseases:
• 33
• 34 ▪ Otitis media
• 35 • Gastroenteritis
infant
• 36
▪ Respiratory illnesses
• 37
• 39
• Urinary tract infections
• 39 • Decreased risk of childhood
• 40 I 0
v


41
42
Feedback
a
End Block
1
Item: 25
3 Previous Lab Values. Notes Calculator.

• Chemotherapy or ongoing radiation


therapy
6
7 • Active abuse of street drugs or alcohol
8
• 9
• 10 - Improved immunity • Galactosemia
• 11 • Improved gastrointestinal function
• 12
- Prevention of infectious diseases:
• 13
• 14 • Otitis media
• 15 • Gastroenteritis
• 16 Infant
• Respiratory illnesses
• 17
• 18
• Urinary tract infections
• 19 • Decreased risk of childhood
20 cancer, type I diabetes mellitus &
21
necrotizing enterocolitis
• 22
• 23
USPALEWorld. LLC
• 24

Breast milk is the ideal form of nutrition for the first 6 months of life. and exclusive breastfeeding is strongly
• 26
recommended. Breastfeeding has a myriad of advantages both for the mother and infant: however. there are
• 27
circumstances in which breastfeeding is contraindicated.
• 28
• 29 Of the answer choices. the only correct statement is that breastfed infants have lower rates of otitis media (as
• 30 well as gastrointestinal. urinary tract. and respiratory infections). Other infant benefits include improved
• 31 overall immunity and reduced risk of chronic diseases such as type I diabetes mellitus and cancer. For these
• 32 reasons. mothers should be encouraged to exclusively breastfeed their infants until age 6 months. This
• 33 mother should be counseled on this information and techniques to make breastfeeding more comfortable. A
• 34 visit with a lactation consultant may be helpful for her.
• ,71

• 36 (Choice A) Preliminary data suggest that breastfeeding reduces rather than increases the risk of childhood
• obesity.
• 38
• 39 (Choice B) Although breastfeeding reduces the risk of both ovarian and breast cancer, it does not affect the
• 40 risk of develooina endometrial cancer_ v
• 41
• 42 111
Feedback, End Block,
• 1
• 2 Item: 25 of 44 V-Mark
▪ 3 Previous Next Lab Values. Notes Calculator.
• 4-
▪ 5
Breast milk is the ideal form of nutrition for the first 6 months of life, and exclusive breastfeeding is strongly
▪ 6
recommended. Breastfeeding has a myriad of advantages both for the mother and infant; however, there are
▪ 7
circumstances in which breastfeeding is contraindicated.
▪ 8
▪ 9 Of the answer choices, the only correct statement is that breastfed infants have lower rates of otitis media (as
• 10 well as gastrointestinal, urinary tract, and respiratory infections). Other infant benefits include improved
• 11 overall immunity and reduced risk of chronic diseases such as type I diabetes mellitus and cancer. For these
• 12 reasons, mothers should be encouraged to exclusively breastfeed their infants until age 6 months. This
• 13 mother should be counseled on this information and techniques to make breastfeeding more comfortable. A
• 14 visit with a lactation consultant may be helpful for her.
• 15
• 16 (Choice A) Preliminary data suggest that breastfeeding reduces rather than increases the risk of childhood
• 17 obesity.
• 18
• 19 (Choice B) Although breastfeeding reduces the risk of both ovarian and breast cancer, it does not affect the
20 risk of developing endometrial cancer.
21
• 22 (Choice 0) Although formula supplementation is not required, exclusively breastfed infants do require
• 23 supplementation with vitamin D until solid foods are introduced into the diet at age 6 months.
• 24
(Choice E) Breastfeeding does increase the duration of postpartum anovulation. However, breastfeeding
26
alone is not a reliable form of contraception, and breastfeeding mothers who wish to avoid pregnancy should

be counseled to use another type of contraception.
• 27
• 28
Educational objective:
• 29
Breastfed infants have a decreased risk of developing otitis media; respiratory, gastrointestinal, and urinary
• 30
tract infections; and necrotizing enterocolitis. Breastfed infants also have lower rates of type I diabetes
• 31
mellitus and childhood cancer. The only absolute infant contraindication to breastfeeding is galactosemia.
• 32
• 33
References:
• 34
• 35 1. Breastfeeding and the use of human milk.
• 36
2. Management of mastitis in breastfeeding women.
• 37
• 39 3. Breast milk jaundice: natural history, familial incidence and late neurodevelopmental
• 39 outcome of the infant.
• 40
• 41
• 42 End Block
Feedback
• 1
• 2 Item: 25 of 44 F' Mark
3 Previous Next Lab Values. Notes Calculator.
rpne-...-e.
iCi iUCU. I-J I CO3LI CCU!! iy 1103 a I I lyl IOU [J I ono ver,
VON, ON 1.-1,1.-1, Tor rin

• 4- CULli OT OUVOI ILOyC3 UULI


oin LI IC moaner II 1101 IL, Howe LI ICI C are

5 circumstances in which breastfeeding is contraindicated.


6
Of the answer choices, the only correct statement is that breastfed infants have lower rates of otitis media (as
7
well as gastrointestinal, urinary tract, and respiratory infections). Other infant benefits include improved
8
overall immunity and reduced risk of chronic diseases such as type I diabetes mellitus and cancer. For these
9
reasons, mothers should be encouraged to exclusively breastfeed their infants until age 6 months. This
• 10
• 11
mother should be counseled on this information and techniques to make breastfeeding more comfortable. A
visit with a lactation consultant may be helpful for her.
• 12
• 13
(Choice A) Preliminary data suggest that breastfeeding reduces rather than increases the risk of childhood
• 14
obesity.
• 15
• 16 (Choice B) Although breastfeeding reduces the risk of both ovarian and breast cancer, it does not affect the
• 17 risk of developing endometrial cancer.
• 18
• 19 (Choice 13) Although formula supplementation is not required, exclusively breastfed infants do require
20 supplementation with vitamin ID until solid foods are introduced into the diet at age 6 months.
21
• 22 (Choice El Breastfeeding does increase the duration of postpartum anovulation. However, breastfeeding
• 23 alone is not a reliable form of contraception, and breastfeeding mothers who wish to avoid pregnancy should
• 24 be counseled to use another type of contraception.

• 26 Educational objective:
Il
• 27 Breastfed infants have a decreased risk of developing otitis media; respiratory, gastrointestinal, and urinary
• 28 tract infections; and necrotizing enterocolitis. Breastfed infants also have lower rates of type I diabetes
• 29 mellitus and childhood cancer. The only absolute infant contraindication to breastfeeding is galactosemia.
• 30
• 31 References:
• 32
1. Breastfeeding and the use of human milk.
• 33
• 34 2. Management of mastitis in breastfeeding women.
• 35 3. Breast milk jaundice: natural history, familial incidence and late neurodevelopmental
• 36 outcome of the infant.
• 37
• 39
• 39
Copyright © LIWorld Last updated: [1/6/2015]
• 40 Ii
• 41
• 42 End Block
Feedback
• 1
• 2 Item: 26 of 44 F' Mark
▪ 3 Previous Next Lab Values. Notes Calculator.
• 4-
5
▪ 6 A 12-year-old boy is brought to the physician because of right groin pain, knee pain, and limping. He has had
▪ 7 these symptoms for the past 2 weeks_ He is at the 90th percentile for weight and 613th percentile for height
▪ 8 He is afebrile, and his other vital signs are within normal limits. Examination shows that the range of motion of
▪ 9 the right knee joint is within normal limits but hip movements are restricted and the right foot points outward.
• 10 There is external rotation of the right thigh on flexion of the hip. After confirming the diagnosis, which of the
• 11 following is the most appropriate management?
• 12
• 13
A. Aspiration and microscopic examination of the hip joint synovial fluid [3%]
• 14
• 15 B. Closed reduction of the hip joint [14%]
• 15 C. Conservative management with rest and analgesics [13%]
• 17
ID. Immediate osteotomy of the femoral neck [5%]
• 18
• 19 • E Surgical pinning of the femoral head [65%]
20
21
Explanation:
• 22
• 23
• 24 Normal Slipped capital femoral epiphysis
• 25
Stable Unstable
• 27
• 28
• 29
• 30
• 31
• 32
• 33
• 34
• 35
• 36
• 37
• 33
• 39
• 40
• 41
• 42 End Block
Feedback
• 1
• 2 Item: 26 of 44 V-. Mark
▪ 3 Previous Next . Lab Values. Notes Calculator.
• 4-
5
Normal Slipped capital femoral epiphysis
▪ 6
▪ 7
Stable Unstable
▪ 8
▪ 9
• 10
• 11
• 12
• 13
• 14
• 15
• 15
• 17
• 18
• 19
20
21
• 22
• 23
• 24
• 25
USPALEWorli.LLC ei 2011

• 27
This patient has a slipped capital femoral epiphysis (SCFE), which is characterized by displacement of the
• 28
femoral head on the femoral neck due to disruption of the proximal femoral growth plate. It is commonly seen
• 29
in obese adolescent boys. The physis (Le., physical junction between the femoral head and neck) weakens
• 30
during early adolescence because it is rapidly expanding and primarily composed of cartilage, which does not
• 31
possess the strength of bone. When exposed to excessive shear stress, which is magnified by obesity, the
• 32
physis fractures and the femoral head slips posteriorly and medially relative to the femoral neck.
• 33
• 34
Patients typically present with hip or knee pain of insidious onset that causes limping_ Acute presentations
• 35
can occur. Diagnosis requires a high degree of clinical suspicion because knee pain (referred pain), not hip
▪ 35
pain, is a common presenting complaint with this condition. Physical examination shows loss of abduction
• 37
and internal rotation of the hip as well as external rotation of the thigh while the hip is being flexed. A frog-leg,
▪ 38
lateral-view x-ray of the hip is the diagnostic imaging technique of choice.
• 39
• 40 Qr6rC 11,1 kr,

• 41
• 42 End Block
Feedback
• 1
• 2 Item: 26 of 44 V-Mark
▪ 3 Previous Next Lab Values. Notes Calculator.
• 4 In ULJEE izluuluutriL uuy. I FIE prig i priyNudi juriuuuri UELWEEFI one IEFFIUrcll FlEdU 'dFlU FIEUK) WE'GIKEFI

▪ 5 during early adolescence because it is rapidly expanding and primarily composed of cartilage, which does not
▪ 6 possess the strength of bone_ When exposed to excessive shear stress, which is magnified by obesity, the
▪ 7 physic fractures and the femoral head slips posteriorly and medially relative to the femoral neck.
▪ 8
Patients typically present with hip or knee pain of insidious onset that causes limping_ Acute presentations
▪ 9
can occur. Diagnosis requires a high degree of clinical suspicion because knee pain (referred pain), not hip
• 10
pain, is a common presenting complaint with this condition. Physical examination shows loss of abduction
• 11
and internal rotation of the hip as well as external rotation of the thigh while the hip is being flexed. A frog-leg,
• 12
lateral-view x-ray of the hip is the diagnostic imaging technique of choice.
• 13
• 14
Patients with SCFE should be promptly treated with surgical pinning of the slipped epiphysis where it lies (i.e.,
• 15
in situ) in order to lessen the risks of avascular necrosis of the femoral head and chondrolysis.
• 15
• 17 (Choice A) Joint aspiration and microscopic analysis are useful in the diagnosis of a septic joint or
• 18 crystal-induced arthropathy.
• 19
20 (Choice B) Closed reduction is not advised due to the risk of further damage to the tenuous blood supply of
21 the femoral head, which can lead to avascular necrosis.
• 22
• 23 (Choice C) Conservative management with rest and analgesics is indicated in the treatment of a tendinous
• 24 or ligamentous strain.
• 25
(Choice ID) Corrective osteotomies can cause avascular necrosis and might not correct the exact anatomic
• 27 deformity. They are usually undertaken later in treatment if a patient experiences persistent pain and limited
• 28 range of motion after initial repair and attempted rehabilitation.
• 29
Educational objective:
• 30
Slipped capital femoral epiphysis typically occurs in obese, early-adolescent boys. It should be promptly
• 31
treated with surgical pinning of the slipped epiphysis where it lies (i.e., in situ) in order to lessen the risks of
• 32
avascular necrosis of the femoral head and chondrolysis_
• 33
• 34
• 35 References:
• 36 1. Slipped capital femoral epiphysis: diagnosis and management.
• 37
• 33
• 39
Copyright © LIWorld Last updated: [8/1/2014] 1
• 40
• 41
• 42 End Block
Feedback
• 1
• 2 Item: 27 of 44 .11V-• Mark
▪ 3 Previous Next Lab Values. Notes Calculator.
• 4-
▪ 5
▪ 6 A 4-month-old boy is brought to the physician for routine evaluation. He was born at 36 weeks gestation by
▪ 7 cesarean section due to shoulder dystocia. His birth weight was 4 kg (8 lb 13 oz). He spent 3 weeks in the
▪ 8 neonatal intensive care unit for management of hypoglycemia and poor feeding secondary to macroglossia.
▪ 9 The patient has had no significant issues since discharge home. He is currently at the 99th percentile for
• 10 weight, length, and head circumference. Physical examination shows an enlarged tongue and a reducible
• 11 umbilical hernia. His right upper and lower extremities are significantly larger in circumference than the left
• 12 extremities. The remainder of his examination is normal. In addition to routine vaccinations, which of the
• 13 following is the best next step in management of this patient?
• 14
• 15
A. Abdominal ultrasound [30%]
• 16
• 17 B. Brain magnetic resonance imaging [10%]
• 18 C. Referral for herniorrhaphy [2%]
• 19
D. Serum glucose level [14%]
20
21 E. Thyroid-stimulating hormone level [40%]
• 22 • Urine homovanillic and vanilmandelic acid [4°4]
• 23
• 24
• 25 Explanation:
• 26

• 28
Beckwith-Wiedemann syndrome
• 29
• 30
• 31 • Deregulation of imprinted gene
Pathogenesis
• 32 expression in chromosome 11 p15
• 33
• 34
• 35 • Fetal macrosomia, rapid growth
• 36
• 37 until late childhood
Physical
• 39 • Omphalocele or umbilical hernia
• 39 examination
• 40 • (Vlacroglossia se
• 41
• 42 End Block
Feedback
• 1
• 2 I I'Mark
▪ 3 Previous Next Lab Values. Notes Calculator.
• 4-
▪ 5
▪ 6 Beckwith-Wiedemann syndrome
▪ 7
▪ 8
▪ 9 • Deregulation of imprinted gene
• 10 Pathogenesis
• 11
expression in chromosome 11 p15
• 12
• 13
• 14
• Fetal macrosomia, rapid growth
• 15 until late childhood
• 16 Physical
• 17
• Omphalocele or umbilical hernia
examination
• 18 • lvlacroglossia
• 19
• Hernihyperplasia
20
21
• 22
• 23
• Wilms tumor
Complications
• 24 • Hepatoblastoma
• 25
• 26
• Serum alpha fetoprotein
• 28 S urveillance
• 29
• Abdominalfrenal ultrasound
• 30
USM LEWor Id, LLC
• 31
• 32
• 33 Beckwith-Wiedemann syndrome (RWS) is an overgrowth disorder characterized by a predisposition to
• 34 neoplasms. Most patients have a sporadic or inherited alteration of chromosome 11p15, which includes
• 35 genes that encode insulin-like growth factor 2, a growth-promoting hormone similar to insulin. At birth, classic
• 36 physical findings include macrosomia, macroglossia, hemihyperplasia, and medial abdominal wall defects
• 37 (umbilical hernia, omphalocele). Some infants also have visceromegaly.
• 39
Newborns must be monitored closely for hypoglycemia. Fetal hyperinsulinemia can result in profound
• 39
hypoglycemia at birth (similar to infants of diabetic mothers). This problem is usually transient, and older
• 40
• 41
• 42 End Block
Feedback
• 1
• 2 Item: 27 of 44 Mark
▪ 3 Previous Next Lab Values. Notes Calculator.

• 4- • Hepatoblastoma
5
▪ 6
▪ 7 • Serum alpha fetoprotein
▪ 8 Surveillance
▪ 9
• Abdominal/renal ultrasound
• 10
USIA LEWor Id, LLC
• 11
• 12
• 13 Beckwith-Wiedemann syndrome (BWS) is an overgrowth disorder characterized by a predisposition to
• 14 neoplasms. Most patients have a sporadic or inherited alteration of chromosome 11p15, which includes
• 15 genes that encode insulin-like growth factor 2, a growth-promoting hormone similar to insulin. At birth, classic
• 16 physical findings include macrosomia, macroglossia, hemihyperplasia, and medial abdominal wall defects
• 17 (umbilical hernia omphalocele). Some infants also have visceromegaly.
• 18
Newborns must be monitored closely for hypoglycemia. Fetal hyperinsulinemia can result in profound
• 19
hypoglycemia at birth (similar to infants of diabetic mothers). This problem is usually transient, and older
20
asymptomatic patients usually do not require ongoing glucose monitoring (Choice I3). Patients with BWS are
21
at increased risk of a wide variety of malignant and benign neoplasms; the 2 most common that require
• 22
vigilant surveillance are Wilms tumor and hepatoblastoma. Current recommendations include screening
• 23
• 24
abdominal ultrasound and alpha-fetoprotein levels every 3 months from birth to age 4 years, abdominal
• 25
ultrasound every 3 months from age 4-8 years, and then renal ultrasound from age 8 years through
adolescence_
• 26

Patients with isolated hemihyperplasia are also at increased risk for Wilms tumor and
• 28
hepatoblastoma. These patients should undergo frequent screening as in BWS.
• 29
• 30 (Choice B) Brain magnetic resonance imaging should be performed in patients with stigmata of
• 31 neurocutaneous syndromes (eg, neurofibromatosis, tuberous sclerosis, von Hippel-Lindau syndrome).
• 32 Although patients with BWS are at increased risk for various abdominal neoplasms, brain lesions are not
• 33 typical features of this disease_
• 34
• 35 (Choice C) Many umbilical hernias close in the first few years of life. In contrast to inguinal hernias, umbilical
• 36 hernias are seldom symptomatic and are at much lower risk of incarcerationistrangulation. Therefore, repair
• 37 is rarely recommended until after age 3 years.
• 39
• 39 (Choice E) Congenital hypothyroidism also can present with macroglossia and umbilical hernia. However,
V
• 40 hvnnnlve_emia anr1 hpn-iihvnernIsiA are not features of enhnphitl hynnthyrnielism anr1 this
• 41
• 42 End Block
Feedback
• 1
• 2 Item: 27 of 44 V-Mark
▪ 3 Previous Next . Lab Values. Notes Calculator.
• 4
▪ 5 (Choice B) Brain magnetic resonance imaging should be performed in patients with stigmata of
▪ 6 neurocutaneous syndromes (eg, neurofibromatosis, tuberous sclerosis. von Hippel-Lindau syndrome)_
▪ 7 Although patients with BWS are at increased risk for various abdominal neoplasms. brain lesions are not
▪ 8 typical features of this disease_
▪ 9
• 10 (Choice C) Many umbilical hernias close in the first few years of life. In contrast to inguinal hernias, umbilical
• 11 hernias are seldom symptomatic and are at much lower risk of incarcerationistrangulation. Therefore, repair
• 12 is rarely recommended until after age 3 years_
• 13
• 14 (Choice E) Congenital hypothyroidism also can present with macroglossia and umbilical hernia. However,
• 15 macrosomia, hypoglycemia, and hemihyperplasia are not features of congenital hypothyroidism, and this
• 16 endocrinopathy is not associated with BNB.
• 17
(Choice F) Patients with RWS are also at increased risk of neuroblastoma_ However, routine screening with
• 18
urinary homovanillic and vanilmandelic acid assays is not recommended due to the low incidence of this
• 19
associated tumor.
20
21
Educational objective:
• 22
Beckwith-Wiedemann syndrome is characterized by macrosomia, macroglossia, umbilical
• 23
hernialomphalocele, hemihyperplasia, and hypoglycemia_ Children must be closely monitored for
• 24
development of Wilms tumor or hepatoblastoma.
• 25
• 26
References:
• 28 1. Experience with hemihyperplasia and Beckwith-Wiedemann syndrome surveillance
• 29 protocol.
• 30
2. Tumour surveillance in Beckwith-Wiedemann syndrome and hemihyperplasia: a critical
• 31
review of the evidence and suggested guidelines for local practice.
• 32
• 33 3. Serum alpha-fetoprotein screening for hepatoblastoma in children with
• 34 Beckwith-Wiedemann syndrome or isolated hemihyperplasia.
• 35 4. Beckwith-Wiedemann syndrome: historical, clinicopathological, and etiopathogenetic
• 36 perspectives.
• 37
• 39
• 39
Copyright © LIWorld Last updated: [10112/2014]
• 40
• 41
• 42 End Block
Feedback
1
• 2 Item: 28 of 44 F' Mark
3 Previous Next Lab Values, Notes Calculator.
• 4-
5
6 A 6-week-old boy is brought to the physician by his parents for evaluation of inconsolable crying. The boy has
7 cried "almost nonstop" for up to 4 hours every evening over the past 3 weeks_ The parents say that pacifiers
8 do not calm him, and they feel overwhelmed and frustrated. Otherwise, he is happy and alert during the rest
9 of the day. The infant was born full term by uncomplicated spontaneous vaginal delivery and is growing well.
• 10 He breastfeeds every 2 hours and has 2-3 soft stools daily. The child's physical examination is
• 11 unremarkable. The parents are concerned that "something is wrong" with their child. Which of the following
• 12 is the most appropriate response?
• 13
• 14
A. "A lactose-free formula will help reduce the pain." [3%]
• 15
• 16 E will order a skeletal survey to evaluate for trauma_" [113C]
• 17 . • C. "I will order an abdominal ultrasound to rule out intussusception_" [4%]
• 18
1 • ID. "I will prescribe ranitidine for gastroesophageal reflux." [3%]
• 19
20 td E. 'Let's review some techniques for soothing your baby.' [89%]
21
• 22
Explanation:
• 23
• 24
• 25
Differential diagnosis for a crying infant
- 25
- 27
Diagnosis Clinical features

30
31 Crying that occurs in an otherwise healthy infant for >3 hours daily
Colic
32 (usually evening), times a week & for a duration of >3 weeks
33
34
• Arching of the back during or after feeding (Sandifer syndrome)
• 35 Gastroesophageal
• Frequent spit-ups or vomiting
• 36 reflux disease
• 37
• Poor weight gain
• 38
• 39 Corneal abrasion Positive fluorescein examination V
• 40


41
42
Feedback
a
End Block
• 1
• 2 Item: 28 of 44 r.Mark
▪ 3 Previous Next Lab Values. Notes Calculator.
• 4
▪ 5
▪ 6
Differential diagnosis for a crying infant
▪ 7
▪ 8 Diagnosis Clinical features
▪ 9
• 10
• 11
Crying that occurs in an otherwise healthy infant for ?3 hours daily
Colic
• 12 (usually evening), :- 3 times a week & for a duration of 3 weeks
• 13
• 14
• Arching of the back during or after feeding (Sandifer syndrome)
• 15 Gastroesophageal
• Frequent spit-ups or vomiting
• 15 reflux disease
• 17
• Poor weight gain
• 18
• 19 Corneal abrasion Positive fluorescein examination
20
21
• 22 Presence of hair that is accidentally tied or wrapped around an
Hair tourniquet
• 23 extremity or digit
• 24
• 25
Milk protein
- 25 Blood-streaked, mucousy, loose stools or severe constipation
allergy
- 27

• 29 Normal infant • Intermittent crying that resolves with usual consoling methods
• 30 crying • Duration <2 hours a day
• 31
• 32 OUWorld
• 33
• 34 Crying is the primary mode of communication during infancy; the normal duration is highly variable but is
• 35 generally 1-2 hours per day. Infantile colic is defined as excessive crying for hours a day, ?3 days a
• 36 week, over a period of weeks in an otherwise healthy infant Colic usually presents in the first few weeks
• 37 of life and resolves spontaneously by age 4 months. The crying generally occurs at the same time of the day,
• 39 typically early in the evening. The cause of colic is unknown but may be due to overstimulation of the infant
• 39 and parental unfamiliarity with alternate soothing methods. Parents often report difficulty and frustration with
• 40 consoling the infant. Calming techniques include using an infant swim swaddling, minimizing environmental v
• 41
• 42 End Block
Feedback
1
• 2 : 28 of 44 r VMark
3 Previous . Next Lab Values. Notes Calculator.

• 4-
5
Milk protein
6 Blood-streaked, mucousy, loose stools or severe constipation
allergy
7 . ,
8
9 Normal infant • Intermittent crying that resolves with usual consoling methods
• 10 crying • Duration <2 hours a day
• 11
_ .... .
• 12 a
• 13
• 14 Crying is the primary mode of communication during infancy; the normal duration is highly variable but is
• 15 generally 1-2 hours per day. Infantile colic is defined as excessive crying for al hours a day, al days a
• 16 week, over a period of weeks in an otherwise healthy infant Colic usually presents in the first few weeks
• 17 of life and resolves spontaneously by age 4 months. The crying generally occurs at the same time of the day,
• 18 typically early in the evening. The cause of colic is unknown but may be due to overstimulation of the infant
• 19 and parental unfamiliarity with alternate soothing methods. Parents often report difficulty and frustration with
20 consoling the infant Calming techniques include using an infant swing, swaddling, minimizing environmental
21 stimuli (eg, quiet dark room), and holding and rocking the baby. Feeding patterns should also be reviewed to
• 22 assess if there is overfeeding, underfeeding, or an inadequate burping technique.
• 23
• 24 (Choice A) There is insufficient evidence to replace cows' milk-based formula or breast milk with hydrolysate
• 25 formula, low-lactose formula, or soy-based formula. Breastfeeding mothers should generally be encouraged
- 25 to continue breastfeeding due to the immunologic benefits. A trial of a hypoallergenic maternal diet (eg, no
- 27 milk, eggs, nuts, wheat) can be considered if the infant does not improve after trying various soothing
techniques.

(Choice B) Colic is associated with increased risk of nonaccidental trauma and post-partum
30
depression. Parental feelings of exhaustion, frustration, guilt, and helplessness are normal and should be
31
acknowledged. Parents should be informed that colic is common and reassured that their child is
32
healthy. Breaks should be encouraged and may include having a friend or relative babysit if the parents feel
33
ovenAihelmed. This child's history and examination is normal and not concerning for child abuse. Further
34
workup (eg, skeletal survey) is not warranted.
• 35
• 36
(Choice C) Intussusception typically occurs at age 3-36 months and presents with recurrent episodes of
• 37
severe abdominal pain. Poor appetite and vomiting are common. This patient's age, crying pattern, and
• 38
normal appetite and growth make a gastrointestinal pathology unlikely_
• 39
V
• 40 ro,fli INF 11,11,111r Or% ,Inii+1,0,1 I Ir.. ,no-.1

• 41
• 42 End Block
Feedback
1
• 2 Item: 28 of 44 V-Mark
-4Z1

3 Previous Next Lab Values. Notes Calculator.


• 4-
consoling the infant Calming techniques include using an infant swing, swaddling, minimizing environmental
5
stimuli (eg, quiet dark room), and holding and rocking the baby Feeding patterns should also be reviewed to
6
assess if there is overfeeding, underfeeding, or an inadequate burping technique.
▪ 7
▪ 8 (Choice A) There is insufficient evidence to replace cows' milk-based formula or breast milk with hydrolysate
▪ 9 formula, low-lactose formula, or soy-based formula. Breastfeeding mothers should generally be encouraged
• 10 to continue breastfeeding due to the immunologic benefits. A trial of a hypoallergenic maternal diet (eg, no
• 11 milk, eggs, nuts, wheat) can be considered if the infant does not improve after trying various soothing
• 12 techniques.
• 13
• 14 (Choice B) Colic is associated with increased risk of nonaccidental trauma and post-partum
• 15 depression. Parental feelings of exhaustion, frustration, guilt, and helplessness are normal and should be
• 16 acknowledged. Parents should be informed that colic is common and reassured that their child is
• 17 healthy. Breaks should be encouraged and may include having a friend or relative babysit if the parents feel
• 18 overwhelmed. This child's history and examination is normal and not concerning for child abuse. Further
• 19 workup (eg, skeletal survey) is not warranted.
20
21 (Choice C) Intussusception typically occurs at age 3-36 months and presents with recurrent episodes of
• 22 severe abdominal pain. Poor appetite and vomiting are common. This patient's age, crying pattern, and
• 23 normal appetite and growth make a gastrointestinal pathology unlikely_
• 24
• 25 (Choice 0) Gastroesophageal reflux disease usually presents with excessive spitting up and crying after
- 25 feedings. Medication is generally reserved for those infants who are not gaining weight appropriately. This
- 27 patient has normal weight gain and the crying pattern does not correlate with feeds, making antacid therapy
inappropriate.
• 29
Educational objective:
• 30
Colic is common and begins in the first few weeks of life with excessive crying for 1
.3 hours a day (usually
• 31
evenings), al days a week. for 13 weeks in an otherwise healthy infant. Soothing and feeding techniques
• 32
should be reviewed and parents should be emotionally supported and reassured.
• 33
• 34
• 35 References:
• 36 1. Colic in infants.
• 37
• 38
• 39
Copyright © UWorld Last updated: [11/3/2014] lit
• 40
• 41
• 42 End Block
Feedback
1
• 2 Item: 29 of 44 V-Mark
3 Previous Next Lab Values. Notes Calculator.
• 4-
5
6 A 17-year-old boy is brought to the emergency department by his father after the boy began threatening him at
7 home. Over the last several months, the father reports, the boy has been increasingly abusive_ He was
8 recently involved in a fist fight at school. The boy states that there is nothing wrong. He is otherwise healthy.
9 He denies alcohol use, but does admit to occasional marijuana use. On examination he has acne on his
• 10 forehead and back and his hairline is receding. There is palpable tissue underneath his nipples bilaterally.
• 11 Heart and lung exams are normal_ What substance is this boy most likely abusing?
• 12
• 13
• 14
▪ A. Anabolic steroids [91%]
• 15 B. Cocaine [11C]
• 16
C. Heroin [11C]
• 17
• 18
D. Methamphetamine [11.6]
• 19 E. Phencyclidine [5%]
20 • Spray paint [1%]
21
• 22
• 23 Explanation:
• 24
• 25 This patient has increased aggression, acne, accelerated male pattern baldness, and gynecomastia
• 26 concerning for anabolic steroid use. Anabolic steroids are popular among young people and athletes looking
• 27 to improve their physical appearance and performance. Unfortunately, anabolic steroids (e.g. testosterone,
• 28 stanozolol, nadrolone) are associated with a significant number of side effects_ They impair endogenous
testicular function and decrease testicular size and sperm count Hepatic dysfunction may occur and patients
can develop erythrocytosis due to androgen-stimulated erythropoiesis. HDL levels decrease, and high doses
31 may even cause cardiac dysfunction_ Psychiatric disorders, like increased aggression and major mood
32 disorders, are more common in patients taking anabolic steroids. Gynecomastia can develop because
33 testosterone is converted to estrogen and will drive breast enlargement Skin changes include acne and
34 premature or accelerated male pattern baldness. Women may have other signs of virilization like hair growth
• 35 and clitoral enlargement
• 36
• 37 (Choice B) Acute cocaine intoxication is associated with euphoria but can be complicated by aggression,
• 38 psychiatric changes, and multiorgan dysfunction. Chronic use is associated with cognitive impairment and
• 39 mood disorders. It is not associated with acne or gynecomastia.
• 40
v
• 41
• 42 End Block
Feedback
1
• 2 Item: 29 of 44 V-Mark
▪ 3 Previous Next Lab Values. Notes Calculator.
• 4-
concerning for anabolic steroid use. Anabolic steroids are popular among young people and athletes looking
▪ 5
to improve their physical appearance and performance. Unfortunately, anabolic steroids (e_g_ testosterone,
▪ 6
stanozolol, nadrolone) are associated with a significant number of side effects_ They impair endogenous
7
testicular function and decrease testicular size and sperm count. Hepatic dysfunction may occur and patients
8
can develop erythrocytosis due to androgen-stimulated erythropoiesis. HDL levels decrease, and high doses
9
may even cause cardiac dysfunction_ Psychiatric disorders, like increased aggression and major mood
• 10
disorders, are more common in patients taking anabolic steroids. Gynecomastia can develop because
• 11
testosterone is converted to estrogen and will drive breast enlargement Skin changes include acne and
• 12
premature or accelerated male pattern baldness. Women may have other signs of virilization like hair growth
• 13
and clitoral enlargement.
• 14
• 15
(Choice B) Acute cocaine intoxication is associated with euphoria but can be complicated by aggression,
• 16
psychiatric changes, and multiorgan dysfunction. Chronic use is associated with cognitive impairment and
• 17
mood disorders. It is not associated with acne or gynecomastia.
• 18
• 19
(Choice C) Heroin intoxication causes symptoms ranging from mild euphoria to stupor and coma. It is
20
associated with respiratory depression, pupillary constriction, and decreased bowel function. Withdrawal
21
might be associated with agitation but not acne or gynecomastia_
• 22
• 23
(Choice 0) Methamphetamine is a CNS stimulant associated with a number of complications. including
• 24
hypertension, tachycardia, severe agitation or aggressiveness, and psychosis. Patients might have acne or
• 25
facial sores due to picking. Male pattern baldness may be seen; however, gynecomastia is absent.,
• 26
• 27
(Choice E) Phenylcyclidine (PCP) causes brief dissociative psychotic episodes. Patients taking PCP are
• 28
notoriously aggressive and unaware of pain, but they do not develop acne, baldness, or gynecomastia.

(Choice F) Spray paint and other volatile inhalants are used to produce short-lived psychotic experiences.
31
There are numerous possible side effects of use, including hypoxia, brain damage. and bone marrow
32
suppression. Acne, aggression, and gynecomastia are not strongly associated with inhalant abuse.
33
34
Educational objective:
• 35
Anabolic steroids are used to improve physique and athletic performance but they are associated with
• 36
numerous adverse effects, including acne, baldness, gynecomastia, hepatic dysfunction, altered lipid profiles,
• 37
virilization, testicular failure, and mood and behavior changes.
• 38
• 39
Copyright @ UWorld Last updated: [8/22/2014]
• 40
• 41
• 42 End Block
Feedback
• 1
• 2 Item: 30 of 44 F' Mark
▪ 3 Previous Next Lab Values. Notes Calculator.
• 4-
▪ 5
▪ 6 A 5-month-old full-term boy is brought to the physician for fever. He was doing well until this morning, when
▪ 7 he felt warm to his mother. He has had a "runny nose" and intermittent coughing but otherwise has been
▪ 8 breastfeeding well. His 3-year-old brother had an upper respiratory tract infection last week. The infant takes
▪ 9 vitamin ID, and his vaccinations are up to date_ His temperature is 38. 9 C 1 G2 1), blood pressure is 90160
• 10 mm Hg, pulse is 12Gimin, and respirations are 32/min. Pulse oximetry shows an oxygen saturation of 96%.
• 11 Examination shows crusted rhinorrhea at the nares. The patient's chest radiograph is shown below. iJ
• 12
• 13
• 14
• 15
• 16
• 17
• 18
• 19
20
21
• 22
• 23
• 24
• 25
• 26
- 27
- 28
• 29

• 31
• 32
• 33
• 34
• 35
• 36
• 37
Which of the following is marked on this patient's chest radiograph?
• 38
• 39
• 40 A. Hilar lymphadenopathy 13%1 se
• 41
• 42 End Block
Feedback
• 1
• 2 Item: 30 of 44 i - Mark
▪ 3 Previous Next Lab Values. Notes Calculator.
• 4 A
5 A. Hilar lymphadenopathy [3%]
▪ 6 B. Right atrium [5%]
▪ 7
C. Right middle-lobe infiltrate [13%]
▪ 8
▪ 9 D. Right upper-lobe atelectasis [9%]
• 10 E. Right upper-lobe infiltrate [9%]
• 11
itio F. Thymus [61%]
• 12 ME-

• 13
• 14 Explanation:
• 15
• 16
• 17


19
19
wit
20
21
• 22
• 23
• 24
• 25
• 26
- 27
- 28
• 29 Cardiac diameter

• 31
• 32
• 33 Thoracic diameter
• 34
• 35
• 36
• 37
• 39
• 39
• 40
• 41
• 42
• 1
• 2 Item: 30 of 44 11 V- Mark
▪ 3 Previous Next Lab Values. Notes Calculator.
• 4- A

5
The thymus is normally visible on chest x-rays in children age <3 years. It is located in the anterior
▪ 6
mediastinum behind the sternum and in front of the heart, aortic arch, and trachea_ It appears prominent on
▪ 7
infant x-rays due to its relatively large size compared to the infant's small thorax. On frontal views, the thymus
▪ 8
is most commonly recognized as the "sail sign" (Image) due to its triangular shape, scalloped border, and
▪ 9
uniform density_ However, the shape is actually quite variable with phases of respiration as it is a soft
• 10
gland_ It also can shrink during times of stress or illness and rebound to a larger size after recovery (rebound
• 11
hyperplasia).
• 12
• 13 The thymus is an important organ in utero and during infancy and childhood for lymphocyte production and
• 14 maturation_ Its absence on neonatal x-ray could be suggestive of thymic hypoplasia or aplasia (eg,
• 15 DiGeorge syndrome). The thymus normally atrophies and is replaced by fat after puberty, when it has
• 16 completed production of T-lymphocytes. Therefore, adults with mediastinal opacities on x-ray should undergo
• 17 further workup for pathology_ Residual thymic tissue can undergo malignant transformation into a thymoma,
• 18 which can occur in patients with myasthenia gravis. Lymphoma should also be on the differential for an
• 19 anterior mediastinal mass in children and adults_
20
21 Due to its location, the thymus can be misdiagnosed as a variety of pathological findings_
• 22
• 23 (Choice A) Hilar lymphadenopathy is never normal and raises concern for malignancy (eg, lymphoma,
• 24 metastases), infection (eg, tuberculosis, histoplasmosis), or inflammation (eg, sarcoidosis). This infant has
• 25 no symptoms or x-ray findings suggestive of an abnormal mediastinal mass.
• 26
- 27 (Choice B) Right atrial enlargement can give the appearance of cardiomegaly. Infants age <1 year
- 28 normally have a transverse cardiothoracic ratio of 613°A. Children age year and adults should have a
• 29 cardiothoracic ratio of 5135.6. This infant has a normal cardiac silhouette.

(Choices C and El Right middle or upper-lobe infiltrates are typical locations for aspiration pneumonia in
• 31
infants or other patients who are primarily in the supine position. Infants with aspiration from swallowing
• 32
dysfunction typically experience choking or gagging preceding respiratory distress. This infant has no
• 33
infiltrates on x-ray.
• 34
• 35
(Choice 13) Right upper-lobe atelectasis appears as a linear density with associated shifting of the
• 36
mediastinum toward the collapsed lung_
• 37
• 38 Educational objective:
• 39 A large thymic silhouette is a normal finding on frontal chest x-ray in children age <3 years due to its relatively
• 40 i-k" "kil"11 " "k"..1"I

• 41
• 42 End Block
Feedback
• 1
• 2 Item: 30 of 44 11 V- Mark
▪ 3 Previous Next Lab Values. Notes Calculator.
• 4
The thymus is an important organ in utero and during infancy and childhood for lymphocyte production and
▪ 5
maturation_ Its absence on neonatal x-ray could be suggestive of thymic hypoplasia or aplasia (eg,
▪ 6
DiGeorge syndrome). The thymus normally atrophies and is replaced by fat after puberty, when it has
▪ 7
completed production of T-lymphocytes_ Therefore, adults with mediastinal opacities on x-ray should undergo
▪ 8
further workup for pathology_ Residual thymic tissue can undergo malignant transformation into a thymoma,
▪ 9
which can occur in patients with myasthenia gravis. Lymphoma should also be on the differential for an
• 10
anterior mediastinal mass in children and adults_
• 11
• 12
Due to its location, the thymus can be misdiagnosed as a variety of pathological findings_
• 13
• 14 (Choice A) Hilar lymphadenopathy is never normal and raises concern for malignancy (eg, lymphoma,
• 15 metastases), infection (eg, tuberculosis, histoplasmosis), or inflammation (eg, sarcoidosis). This infant has
• 16 no symptoms or x-ray findings suggestive of an abnormal mediastinal mass.
• 17
• 18 (Choice B) Right atrial enlargement can give the appearance of cardiomegaly. Infants age <1 year
• 19 normally have a transverse cardiothoracic ratio of 0% Children age Ll year and adults should have a
20 cardiothoracic ratio of 50136_ This infant has a normal cardiac silhouette.
21
• 22 (Choices C and E) Right middle or upper-lobe infiltrates are typical locations for aspiration pneumonia in
• 23 infants or other patients who are primarily in the supine position_ Infants with aspiration from swallowing
• 24 dysfunction typically experience choking or gagging preceding respiratory distress. This infant has no
• 25 infiltrates on x-ray.
• 26
- 27 (Choice ID) Right upper-lobe atelectasis appears as a linear density with associated shifting of the
- 28 mediastinum toward the collapsed lung_
▪ 29
Educational objective:
A large thymic silhouette is a normal finding on frontal chest x-ray in children age <3 years due to its relatively
• 31
large size compared to the young child's thorax. Opacities in this location in other children should raise
• 32
concern for pneumonia or malignancy, depending on the clinical context_
• 33
• 34
• 35 References:
• 36 1. Clinical and radiologic review of the normal and abnormal thymus: pearls and pitfalls.
• 37
• 38
• 39
Copyright © UWorld Last updated: [9/22/2014]
• 40
• 41
• 42 End Block
Feedback
• -1
• 2 Item: 30 of 44 V-Mark
EF.

▪ 3 Previous Next Lab Values. Notes Calculator


• 4- Media Exhibit
▪ 5
▪ 6 Absent thymic shadow
▪ 7
▪ 8
▪ 9
• 10
• -1 -1
• 12
• 13
• 14
• 15
• 16
• 17
• 19
lassie location
• 19 and shape of
20 thymus in
21 newborn
• 22
• 23
("sail sign")
• 24
• 25
• 26
- 27
- 28
• 29

• 31
• 32
• 33
• 34
• 35
• 36
• 37
• 39
• 39
• 40
• 41
• 42 End Block
Feedback
• -1
• 2 Item: 30 of 44 V- Mark
EF.

▪ 3 Previous Next Lab Values. Notes Calculator


• 4- Media Exhibit
▪ 5
▪ 6 Metastatic neuroblastoma
▪ 7
▪ 8
▪ 9
• 10


-1 -1
12
L
• 13
RW
• 14
• 15
• 16
• 17
• 19
• 19
20
21
• 22
• 23


24
25
-4.1-
• 26
• 27
• 28
• 29
30
• 31
• 32
• 33
• 34
35
36
37
39
• 39
• 40


41
42
Feedback
a
End Block
• -1
• 2 Item: 30 of 44 V-Mark
EF.

▪ 3 Previous Next Lab Values. Notes Calculator


• 4- Media Exhibit
▪ 5
▪ 6 Right atrial enlargement
▪ 7
▪ 8
▪ 9
• 10
• -1 -1
• 12
• 13
• 14
• 15
• 16
• 17
• 19 I
Right atrial
• 19 Pacemaker lead
enlargement

• 22
Cardiac diameter
• 23
• 24
• 25
• 26
[esc
- 27 Thoracic diameter
- 28
▪ 29

7 1.
• 32
• 33
• 34
• 35
• 36
• 37
• 30
• 39
• 40
• 41
• 42 End Block
Feedback

• 2 Item: 30 of 44 V-Mark
EF. 11
.

▪ 3 Previous Next Lab Values Notes Calculator


• 4- Media Exhibit
▪ 5
▪ 6 Right middle lobe pneumonia
▪ 7
▪ 8
▪ 9
• 10
• 1-1
• 12
• 13
• 14
• 15
• 16
• 17
• 19
• 19
20
21
• 22
• 23
• 24
• 25
• 26
- 27
- 28
• 29

• 31
• 32
• 33
• 34
• 35
• 36
• 37 06


39
39
3
• 40 II
• 41
• 42 End Block
Feedback
• -1
• 2 Item: 30 of 44 V- Mark
-4(1 ra77

▪ 3 Previous Next Lab Values Notes Calculator


• 4- Media Exhibit
▪ 5
▪ 6 Right middle lobe pneumonia
▪ 7
▪ 8
▪ 9
• 10
• 1-1
• 12
• 13
• 14
Tracheostorny
• 15 tube
• 16
• 17
1111111-
• 19
• 19
20
21
• 22
• 23
• 24
• 25
• 26
• 27
• 28
▪ 29

• 31
• 32
• 33
• 34
• 35
• 36 J
• 37


39
39
3
• 40


41
42
Feedback
a
End Block
• -1
• 2 Item: 30 of 44 V-Mark
11
.

▪ 3 Previous Next Lab Values Notes Calculator


• 4- Media Exhibit X
▪ 5
▪ 6 1111 collapse
▪ 7
▪ 8
▪ 9
• 10
• -1 -1
• 12
• 13
• 14
• 15
• 16
• 17
• 19
• 19
20
21
• 22
• 23
• 24
• 25
• 26
• 27
• 28
• 29

• 31
• 32
• 33
• 34
• 35
• 36
• 37
• 39
• 39
• 40
• 41
• 42
'1
-1

2 Item: 30 of 44 MI F. Mark 11
.
3 Previous Next Lab Values Notes Calculator
4- Media Exhibit
5
6 RUL collapse
7
8
9
10
-1-1

12
13
14
15
16
17
18
19
20
21
• 22
• 23
• 24
• 25
• 26
- 27
- 28
▪ 29

• 31
• 32
• 33
• 34
• 35
• 36
• 37
• 39
• 39
• 40 ii


41
42
Feedback
a
End Block
• 1
• 2 Item: 31 of 44 F' Mark
▪ 3 Previous Next Lab Values. Notes Calculator.
• 4-
5
▪ 6 A 3-day-old boy is in the neonatal intensive care unit for management of prematurity. For the past few days,
▪ 7 he has had decreased spontaneous movements, decreased tone, seizures, and rapidly increasing head
▪ 8 circumference. The boy was delivered vaginally at 3D weeks gestation by a multiparous woman with cervical
▪ 9 incompetence. Birth weight was 1.36 kg (3 lb). Prenatal laboratory studies and ultrasounds were normal.
• 10 Rupture of membranes occurred 3 hours prior to birth. Amniotic fluid was clear, and there was no maternal
• 11 fever. The neonate's vital signs show intermittent bradycardia and apnea_ Examination shows a lethargic
• 12 neonate with a weak and high-pitched cry, tense fontanels, and generalized hypotonia. Complete blood count
• 13 shows anemia C-reactive protein is normal_ Head ultrasound is shown below.
• 14
• 15
• 16
• 17
• 18
• 19
20
21
• 22
• 23
• 24
• 25
• 26
• 27
• 28 I
29
• 30

• 32
• 33
• 34
• 35
• 36
• 37
• 38
• 39
What is the most likely cause of the patient's condition?
• 40
• 41
• 42 End Block
Feedback
• 1
• 2 Item: 31 of 44 F' Mark
▪ 3 Previous Next Lab Values. Notes Calculator.
• 4-
▪ 5 •
▪ 6
▪ 7 COR ANT-POST
▪ 8
▪ 9
What is the most likely cause of the patient's condition?
• 10
• 11
• 12 A. Arnold-Chiari malformation [10°A]
• 13 B. Choroid plexus cysts [79.6]
• 14
C. Craniopharyngioma [1%]
• 15
• 16 D. Dandy-Walker malformation [8%]
• 17 E Intraventricular hemorrhage [73%]
• 18
Meningitis [1%]
• 19
20
21 Explanation:
• 22
• 23
• 24 Complications of prematurity
• 25
• 26
• Respiratory distress syndrome
• 27
• 28 • Patent ductus arteriosus
• 29 • Bronchopulmonary dysplasia
• 30
• Intraventricular hemorrhage
• 32 • Necrotizing enterocolitis
• 33 • Retinopathy of prematurity
• 34
• 35 gitl5MLArOtiCk. uc
• 36
• 37
Premature and underweight neonates are at high risk of multisystem comorbidities and death (Table).
• 38
Intraventricular hemorrhage (IVH) is a common complication in neonates born at <30 weeks gestation or
• 39
<1500 g (3.3 lb). The susceptibility is due to capillary fragility of the subependymal germinal matrix and
• 40
• 41
• 42 End Block
Feedback
1
• 2 Item: 31 of 44 V-Mark
3 Previous Next Lab Values. Notes Calculator.
• 4- A
Premature and underweight neonates are at high risk of multisystem comorbidities and death (Table).
5
Intraventricular hemorrhage (IVH) is a common complication in neonates born at <30 weeks gestation or
6
<1500 g (3.3 lb). The susceptibility is due to capillary fragility of the subependymal germinal matrix and
7
immature autoregulation of cerebral blood flow.
8
9 Screening for IVH with serial head ultrasounds is necessary, as 25%-5G% of cases are asymptomatic.
• 10 This patient's presentation (lethargy, hypotonia, high-pitched cry, rapidly increasing head circumference.
• 11 bulging fontanels) is consistent with severe hemorrhage_ The coronal view of this patient's cranial
• 12 ultrasound shows bilateral IVH and dilated ventricles. Communicating (nonobstructive) hydrocephalus is a
• 13 complication in one-third of cases, as accumulating blood irritates the arachnoid villi, impairing its ability to
• 14 absorb cerebrospinal fluid. These patients are at greatest risk of death; those who survive high-grade bleeds
• 15 often suffer from significant neurodevelopmental disability (eg, cerebral palsy). Prevention of preterm labor
• 16 and antenatal administration of maternal corticosteroids are the only interventions that can reduce the
• 17 incidence of IVH and improve overall mortality_
• 18
• 19 (Choices A and D) Arnold-Chiari and Dandy-Walker malformations are congenital brain malformations that
20 cause noncommunicating hydrocephalus due to obstruction of cerebrospinal fluid flow in the posterior fossa.
21 However, the precipitous neurologic decline and acute hydrocephalus of this patient are more consistent with
• 22 IVH.
• 23
• 24 (Choice B) Choroid plexus cysts are commonly identified on second-trimester ultrasound as a soft marker
• 25 for fetal aneuploidy. They usually regress spontaneously by the third trimester but are asymptomatic and
I
• 26 benign even if they persist after birth.
• 27
• 28
(Choice C) Craniopharyngioma is a slow-growing tumor that typically presents at age 5-14 years and rarely
• 29
presents at birth.
• 30
(Choice F) Neonatal meningitis can present with nonspecific symptoms, such as the cardiopulmonary
instability and neurologic decline seen in this patient Communicating hydrocephalus is a common
• 32
complication affecting one-fourth of neonates with meningitis_ However, the lack of infectious risk factors (eg,
• 33
maternal fever, prolonged rupture of membranes), presence of anemia and intraventricular blood on
• 34
ultrasound, and normal inflammatory markers (eg, C-reactive protein, white blood cell count) make infection
• 35
less likely than IVH.
• 36
• 37
Educational objective:
• 38
Intraventricular hemorrhage is a common complication in premature and underweight neonates.
• 39
Accumulation of blood in the subarachnoid space can impair the ability of the arachnoid villi to absorb
• 40
• 41
• 42 End Block
Feedback
A
• 1
• 2 Item: 31 of 44 .11 V-Mark
▪ 3 Previous Next Lab Values. Notes Calculator.
• 4- often suffer from significant neurodevelopmental disability (eg, cerebral palsy). Prevention of preterm labor
▪ 5 and antenatal administration of maternal corticosteroids are the only interventions that can reduce the
▪ 6 incidence of IVH and improve overall mortality.
▪ 7
▪ 8 (Choices A and D) Arnold-Chiari and Dandy-Walker malformations are congenital brain malformations that
▪ 9 cause noncommunicating hydrocephalus due to obstruction of cerebrospinal fluid flow in the posterior fossa.
• 10 However, the precipitous neurologic decline and acute hydrocephalus of this patient are more consistent with
• 11 IVH.
• 12
• 13
(Choice B) Choroid plexus cysts are commonly identified on second-trimester ultrasound as a soft marker
• 14
for fetal aneuploidy. They usually regress spontaneously by the third trimester but are asymptomatic and
• 15
benign even if they persist after birth.
• 16
(Choice C) Craniopharyngioma is a slow-growing tumor that typically presents at age 5-14 years and rarely
• 17
presents at birth.
• 18
• 19
(Choice F) Neonatal meningitis can present with nonspecific symptoms, such as the cardiopulmonary
20 instability and neurologic decline seen in this patient Communicating hydrocephalus is a common
21
complication affecting one-fourth of neonates with meningitis. However, the lack of infectious risk factors (eg,
• 22
maternal fever, prolonged rupture of membranes), presence of anemia and intraventricular blood on
• 23
ultrasound, and normal inflammatory markers (eg, C-reactive protein, white blood cell count) make infection
• 24
less likely than IVH.
• 25
• 26 Educational objective:
• 27 Intraventricular hemorrhage is a common complication in premature and underweight neonates.
• 28 Accumulation of blood in the subarachnoid space can impair the ability of the arachnoid villi to absorb
• 29 cerebrospinal fluid, resulting in communicating hydrocephalus_ Affected neonates are at risk for permanent
• 30 neurodevelopmental problems and death.

• 32
References:
• 33
• 34 1. Intracranial hemorrhage in the preterm infant: understanding it, preventing it.
• 35 2. The diagnosis, management, and postnatal prevention of intraventricular hemorrhage in
• 36 the preterm neonate.
• 37
• 38
• 39
Copyright © UWorld Last updated: [10/9/2014]
• 40
• 41
• 42 End Block
Feedback
1
2 Item: 31 of 44 .111 II Mark
ra77

3 Previous Next Lab Values. Notes Calculator


4- Media Exhibit
5
6 Intraventricular hemorrhage of prematurity
7
8
9
10
-1 -1

12
13
14
15
16
17
19
19
20
21
• 22
• 23
• 24
• 25
• 26
• 27
• 28
Is
• 29
1.
• 30

• 32
• 33
• 34
• 35
• 36
• 37
• 39
• 39
• 40
• 41
• 42
v
• 1
• 2 Item: 32 of 44 V- Mark
▪ 3 Previous Next Lab Values. Notes Calculator.
• 4-
▪ 5
▪ 6 A 6-year-old female with Down syndrome is brought to the physician for behavioral changes. Over the past
▪ 7 few weeks, she has begun refusing to do her usual activities. Her parents also report that she seems dizzy
▪ 8 and state that she has developed urinary incontinence. On examination, she is hypotonic but hyperreflexic
▪ 9 with a positive Babinski reflex_ Her gait is ataxic. Which of the following is the most likely diagnosis?
• 10
• 11
• 12
A. Alzheimer disease [7%]
• 13 ▪ B. Atlantoaxial instability [57%]
• 14
C. Hypothyroidism [8%]
• 15
• 16
ID. Mental retardation [3%]
• 17 E Spinal cord infarction [25%]
• 18
• 19
20
Explanation:
21
Atlantoaxial instability is a malformation seen in 10-15% of patients with Down syndrome, and most commonly
• 22
occurs due to excessive laxity in the posterior transverse ligament, which causes increased mobility between
• 23
the atlas (Cl) and the axis (C2). Fortunately, only 1-2% of Down syndrome patients with atlantoaxial instability
• 24
are symptomatic. Symptoms usually progress over several weeks and result from compression of the spinal
• 25
cord. Presenting symptoms include behavioral changes, torticollis, urinary incontinence, and vertebrobasilar
• 26
symptoms such as dizziness, vertigo, and diplopia. On examination, upper motor neuron symptoms such as
• 27
leg spasticity, hyperreflexic, a positive Babinski sign, and clonus are often present Patients with Down
• 28
syndrome are normally hypotonic, and they may remain hypotonic or have increased tone with symptomatic
• 29
30 atlantoaxial instability.
• 31
Atlantoaxial instability is suspected on physical examination and diagnosed with lateral radiographs of the
cervical spine in flexion, extension, and in a neutral position. Open mouth radiographs can also be helpful in
• 33
visualizing the odontoid. Treatment consists of surgical fusion of the first cervical vertebrae (C1)to the
• 34
second (C2).
• 35
• 36 (Choice A) Although there is a higher prevalence of Alzheimer disease among individuals with Down
• 37 syndrome, it usually presents around ages 40-51
• 38
• 39 (Choice C) The risk of hypothyroidism is increased in individuals with Down syndrome. These patients
• 40 nr-oc.ani- ierith Aar nr-roArFft lealnr ihr ierair,kt nmin r nIrl ini-nlar.mnr r nr rlar lininn crknnl

• 41
• 42 End Block
Feedback
• 1
• 2 Item: 32 of 44 V-Mark
▪ 3 Previous Next Lab Values. Notes Calculator.

• 4-
Explanation:
5
▪ 6
Atlantoaxial instability is a malformation seen in 10-15% of patients with Down syndrome, and most commonly
▪ 7
occurs due to excessive laxity in the posterior transverse ligament, which causes increased mobility between
▪ 8
the atlas (Cl) and the axis (C2). Fortunately, only 1-2% of Down syndrome patients with atlantoaxial instability
▪ 9
are symptomatic. Symptoms usually progress over several weeks and result from compression of the spinal
• 10
cord. Presenting symptoms include behavioral changes, torticollis, urinary incontinence, and vertebrobasilar
• 11
symptoms such as dizziness, vertigo, and diplopia. On examination, upper motor neuron symptoms such as
• 12 leg spasticity, hyperreflexia, a positive Babinski sign, and clonus are often present Patients with Down
• 13
syndrome are normally hypotonic, and they may remain hypotonic or have increased tone with symptomatic
• 14
atlantoaxial instability.
• 15
• 16 Atlantoaxial instability is suspected on physical examination and diagnosed with lateral radiographs of the
• 17 cervical spine in flexion, extension, and in a neutral position. Open mouth radiographs can also be helpful in
• 18 visualizing the odontoid. Treatment consists of surgical fusion of the first cervical vertebrae (C1)to the
• 19 second (C2).
20
21 (Choice A) Although there is a higher prevalence of Alzheimer disease among individuals with Down
• 22 syndrome, it usually presents around ages 40-50.
• 23
• 24 (Choice C) The risk of hypothyroidism is increased in individuals with Down syndrome. These patients
• 25 present with decreased growth velocity, weight gain, cold intolerance, fatigue, constipation, or declining school
• 26 performance_ Neurologic symptoms can include delayed reflexes, but upper motor neuron signs should not
• 27 be present.
• 28
• 29 (Choice 0) Mental retardation is seen in individuals with Down syndrome and can vary in its severity.
• 31h: However, progressive neurologic symptoms are not seen with mental retardation alone_
• 31
(Choice E) Spinal cord infarction presents with the acute onset of severe pain, weakness, and paresthesias.
This condition is rare and more commonly seen in adults, and there is no increased risk in patients with Down
• 33
syndrome.
• 34
• 35
Educational objective:
• 36
Atlantoaxial instability should be suspected in any patient with Down syndrome who presents with upper motor
• 37
neuron findings_
• 39
• 39
Copyright © LIWorld Last updated: [12/29/2014]
• 40
• 41
• 42 End Block
Feedback
1
• 2 Item: 33 of 44 F' Mark
▪ 3 Previous Next Lab Values. Notes Calculator.
• 4-
▪ 5
▪ 6 A 2-year-old boy is brought by his mother to the emergency department because of a high-grade fever which
▪ 7 "does not go away" with acetaminophen. For the last four days, the child has been very irritable and is crying
▪ 8 a lot He is also pulling his ear and not eating well. He has been generally well, other than the occasional sore
▪ 9 throat this season_ His temperature is 38.8° C (102.2° F), blood pressure is 90160 mm Hg, pulse is 119/min,
• 10 and respirations are 24/min. He appears well nourished, but is irritable_ Physical examination reveals
• 11 enlarged cervical lymph nodes and splenomegaly. The tympanic membranes are inflamed. CBC shows:
• 12
WBC 81,1113 mm3
• 13
Hemoglobin 8.0 gidL
• 14
Hematocrit 25%
• 15
Platelets 16,000 mm3
• 16
Blast forms 80%
• 17
Prolymphocytes 10%
• 18
Lymphocytes 10%
• 19
20
The blast cells have condensed nuclear chromatin, small nucleoli and scant agranular cytoplasm_
21
Subsequent histochemical staining reveals strongly positive periodic acid Schiff (PAS) reaction. No Auer rods
• 22
were seen. Which of the following is the most likely diagnosis?
• 23
• 24
• 25 A. Burkitt lymphoma [6%]
• 26 B. Acute myelocytic leukemia [8%]
• 27
C. Prolymphocytic leukemia [4%]
• 28
• 29 ► ID. Acute lymphoblastic leukemia [79%]
• 30 E. Myelodysplastic syndrome [3%]
• 31
• 32
Explanation:
• 34
• 35 Acute lymphoblastic leukemia (ALL) is the predominant type of leukemia in children from ages 2-10 years.
• 36 Approximately 30-50% of patients present with infections, and about half present with lymphadenopathy and
• 37 splenomegaly_ Lymphoblasts are typically seen on the peripheral smear. Varying degrees of anemia,
• 39 neutropenia, and thrombocytopenia have been noted. The presence of more than 25% lymphoblasts in the
• 39 bone marrow is diagnostic_ Lymphoblasts lack peroxidase positive granules but often contain cytoplasmic
• 40 aggregates of periodic acid Schiff (PAS) positive material. lmmunostaining for terminal
• 41
• 42 End Block
Feedback
• 1
• 2 Item: 33 of 44 Mark
▪ 3 Previous Next Lab Values. Notes Calculator.
Le- I 13111 14,11,11-.1.11-lie 14-.41111.4-.11111-4
• 4-
▪ 5 • • E Myelodysplastic syndrome [3%]
▪ 6
▪ 7
Explanation:
▪ 8
▪ 9
Acute lymphoblastic leukemia (ALL) is the predominant type of leukemia in children from ages 2-10 years.
• 10
Approximately 30-50% of patients present with infections, and about half present with lymphadenopathy and
• 11
splenomegaly_ Lymphoblasts are typically seen on the peripheral smear. Varying degrees of anemia,
• 12
neutropenia, and thrombocytopenia have been noted. The presence of more than 25% lymphoblasts in the
• 13
bone marrow is diagnostic. Lymphoblasts lack peroxidase positive granules but often contain cytoplasmic
• 14
aggregates of periodic acid Schiff (PAS) positive material_ Immunostaining for terminal
• 15
deoxynucleotidyltransferase (TdT) is positive in more than 95% of patients. TdT is expressed only by pre B
• 16
and pre T lymphoblasts.
• 17
• 18 Myeloblasts on the other hand contain peroxidase positive material.
• 19
20 (Choice A) Burkitt lymphoma is a neoplasm of mature B cells. It is associated with the Epstein-Barr virus
21 infection. Most patients present with either a mass involving the mandible or abdominal viscera. High mitotic
• 22 index is typical. Histological examination shows characteristic "starry sky appearance". It is a very
• 23 aggressive tumor but responds well to the high dose chemotherapy.
• 24
• 25 (Choice B) In acute myelocytic leukemia (AML), the predominant cells are of myeloid origin. Auer rods are
• 26 specific for AML.
• 27
• 28 (Choice C) Prolymphocytic leukemia is a CLL variant. It is characterized by massive splenomegaly. high
• 29 lymphocytosis, and lymphadenopathy.
• 30
(Choice E) Myelodysplastic syndromes (MIDS) are clonal stem cell disorders which may progress to acute
• 31
leukemias. These are usually seen in elderly patients, and are characterized by pancytopenia.
• 32

Educational Objective:
• 34
Acute lymphoblastic leukemia is predominantly a disease of children_ Lymphoblasts lack peroxidase positive
• 35
granules but often contain cytoplasmic aggregates of periodic acid Schiff (PAS) positive material_
• 36
Immunostaining for terminal deoxynucleotidyltransferase (TdT) is positive in more than 95% of patients. TdT
• 37
is expressed only by pre B and pre T lymphoblasts.
• 39
• 39
Copyright © UWorld Last updated: [8/19/2014]
• 40 1
• 41
• 42 End Block
Feedback
• 1
• 2 Item: 34 of 44 V-Mark
▪ 3 Previous Next Lab Values. Notes Calculator.
• 4-
▪ 5
▪ 6 A 2-year-old boy is brought to the emergency department due to a cough and a "whistling" sound with
▪ 7 breathing. Two days ago, he developed rhinorrhea, fever, a hoarse cry and a progressively worsening, harsh.
▪ 8 "barkc cough. His immunizations are up-to-date. His 6-year-old brother also has cold symptoms. His
▪ 9 temperature is 37.5° C (99.5° F), pulse is 14G/min, and respirations are 36/min. On examination, he is alert.
• 10 in mild respiratory distress, has a dry barking cough, hoarse cry, and some clear mucoid rhinorrhea_ His
• 11 pharynx is slightly injected, but without enlargement or asymmetry. The lungs are clear on auscultation.
• 12 Lateral neck x-rays reveal a mildly narrowed subglottic region. What is the most likely diagnosis?
• 13
• 14
• 15
A. Epiglottitis [5%]
• 16 B. Croup [88%]
• 17
C. Laryngotracheobronchopneumonitis [4%]
• 18
• 19
ID. Foreign body aspiration [1%]
20 E Laryngeal diphtheria [11.6]
21
• 22
• 23 Explanation:
• 24
Croup, also known as laryngotracheitis or laryngotracheobronchitis (LTB), is characterized by laryngeal
• 25
inflammation that results in hoarseness, a barking cough, and varying degrees of respiratory distress over
• 26
time. The typical patient is less than 3 years of age, and the most common causative agent is Parainfluenza
• 27
virus. Croup is usually a clinical diagnosis. Lateral neck x-rays show subglottic narrowing.
• 28
• 29
(Choice A) Epiglottitis is seen in older children. Furthermore, its presentation is different The cough is not
• 30
"croupy", and the patient appears more toxic, with high-grade fever, tachypnea and tachycardia. Stridor
1:=
31
and excessive drooling are often found. The three characteristic findings on lateral neck x-rays are: swollen
• 32
epiglottis (thumb sign), thickened aryepiglottic folds, and obliteration of the vallecula.
• 33

e (Choice C) Bacterial laryngotracheobronchopneumonitis is characterized by signs of lower respiratory


• 35
involvement, such as wheezing and interstitial changes on chest x-rays. Hypoxia secondary to the lower
r• 36 I
airway disease often occurs.
• 37
• 38 (Choice 0) Foreign body aspiration should be considered in cases of sudden onset stridor without a fever_
• 39
• 40 fIr+6..m.:•••••. 14- L-11-1"1-1.-•11.-..1-1. 14-
se
• 41
• 42 End Block
Feedback
1
• 2 Item: 34 of 44 V-Mark
-4Z1

3 Previous Next Lab Values. Notes Calculator.


LaLGI al I IUL,P. ay-.3 1 G Aruai a l l inuly iiai I L.ITYULI I IUL 1J LI 1G I 114....L !
• 4-
5
6 A. Epiglottis [5%]
7
0 B. Croup [88%]
8
9
C. Laryngotracheobronchopneumonitis [4%]
• 10 ID. Foreign body aspiration [1%]
• 11
E Laryngeal diphtheria [1°4]
• 12
• 13
• 14 Explanation:
• 15
• 16 Croup, also known as laryngotracheitis or laryngotracheobronchitis (LTB), is characterized by laryngeal
• 17 inflammation that results in hoarseness, a barking cough, and varying degrees of respiratory distress over
• 18 time. The typical patient is less than 3 years of age, and the most common causative agent is Parainfluenza
• 19 virus. Croup is usually a clinical diagnosis. Lateral neck x-rays show subglottic narrowing.
20
21 (Choice A) Epiglottis is seen in older children. Furthermore, its presentation is different The cough is not
• 22 "croupy", and the patient appears more toxic, with high-grade fever, tachypnea and tachycardia. Stridor
• 23 and excessive drooling are often found. The three characteristic findings on lateral neck x-rays are: swollen
• 24 epiglottis (thumb sign), thickened aryepiglottic folds, and obliteration of the vallecula.
• 25
• 26 (Choice C) Bacterial laryngotracheobronchopneumonitis is characterized by signs of lower respiratory
• 27 involvement, such as wheezing and interstitial changes on chest x-rays. Hypoxia secondary to the lower
• 28 airway disease often occurs.
• 29
(Choice 0) Foreign body aspiration should be considered in cases of sudden onset stridor without a fever_
• 30
• 31
(Choice E) Although laryngeal diphtheria is rare, it should still be considered because it sometimes presents
• 32
with a croup-like syndrome, known as membranous croup. In this case, however, the patient's immunization
• 33
history rules out this diagnosis_
• 35
Educational Objective:
• 36
Croup is caused by Parainfluenza virus and commonly presents in children younger than 3 years with
• 37
hoarseness, a barking cough, and varying degrees of respiratory distress_
• 38
• 39
Copyright © LIWorld Last updated: [16/23/2014]
• 40
• 41
• 42 End Block
Feedback
• 1
• 2 Item: 35 of 44 V-• Mark
▪ 3 Previous Next Lab Values. Notes Calculator.
• 4-
▪ 5
You are called to examine a newly delivered infant, who is 41 weeks gestation and a product of an
▪ 6
uncomplicated pregnancy. Physical examination reveals a matted mass of edematous bowel loops
▪ 7
protruding from the abdomen_ There is no covering over the bowel loops. The umbilical cord appears
▪ 8
normal. Which of the following is the best next step in the management of this patient?
▪ 9
• 10 A. Intravenous nutrition [11C]
• 11 B. Sterile wrapping of exposed bowel [63%]
• 12
C. Immediate surgery [22%]
• 13
• 14 ID. Look for associated anomalies [119C]
• 15 E Initiate broad-spectrum antibiotics [3%]
• 16
• 17
• 18 Explanation:
• 19
20
Diagnosing congenital abdominal wall defects can be made antenatally with maternal serum-AFP
21 measurements and ultrasonography. The two types of abdominal wall defects are described in the table below:
• 22 Gastroschisis Omphalocele
• 23 1.Bowel protrudes through a
• 24 defect on the right side of the
• 25 1.Intra-abdominal contents protrude through the umbilical) riig_
umbilical cord.
• 26 The abdominal herniation usually includes small bowel and
2. Bowel is not covered by a
• 27 may include large bowel and liver_
protective membrane.
• 28 2. Bowel is covered by an amnioperitoneal membrane.
3. Bowel looks "angry" and matted.
• 29 3. Can be associated with other congenital abnormalities (e.g..
4. Not typically associated with
• 30 heart, kidney).
any other abnormalities outside
• 31 the GI tract
• 32
• 33
In this case, the neonate's congenital abdominal wall defect is gastroschisis. If the diagnosis was made
• 34
antenatally, management would begin with planned spontaneous vaginal delivery at a tertiary neonatal surgical
unit Caesarean section is performed only in the presence of the usual obstetric indications. After delivery, the
• 36
next step in management is to immediately wrap the exposed bowel with sterile saline dressings and cover
• 37
with plastic wrap in order to prevent insensible heat and large fluid losses. The neonate is then maintained in a
• 39
thermo-neutral environment
• 39
• 40
• 41
• 42 End Block
Feedback
• 1
• 2 Item: 35 of 44 V-Mark
▪ 3 Previous Next Lab Values. Notes Calculator.
• 4- 4. Not typically associated with
heart, kidney).
▪ 5 any other abnormalities outside
▪ 6 the GI tract
▪ 7
▪ 8 In this case, the neonate's congenital abdominal wall defect is gastroschisis. If the diagnosis was made
▪ 9 antenatally, management would begin with planned spontaneous vaginal delivery at a tertiary neonatal surgical
• 10 unit Caesarean section is performed only in the presence of the usual obstetric indications. After delivery, the
• 11 next step in management is to immediately wrap the exposed bowel with sterile saline dressings and cover
• 12 with plastic wrap in order to prevent insensible heat and large fluid losses_ The neonate is then maintained in a
• 13 thermo-neutral environment
• 14
• 15 (Choices A and El Further steps in the management of gastroschisis include inserting an orogastric tube (to
• 16 decompress the stomach), establishing peripheral intravenous access (to provide fluids), and administration of
• 17 broad-spectrum antibiotics.
• 18
• 19 (Choice C) Definitive treatment is indeed surgical; however, surgery should be preceded by the above
20 measures_ Primary closure is successful in 70% of neonates, but if it is unsuccessful, staged closure with
21 Silastic silo can be used.
• 22
• 23 (Choice ID) Looking for associated anomalies is not routinely required in gastroschisis. It is usually done in
• 24 patients with omphalocele, which is commonly associated with other anomalies. Fetal chromosomal analysis
• 25 by amniocentesis should be offered antenatally in such cases_
• 26
The management of omphalocele is similar to that of gastroschisis_ Delivery is also planned at a tertiary care
• 27
center, and caesarean section is reserved for the usual obstetric indications. (This is based on the lack of
• 28
evidence that caesarean delivery improves the outcome of patients with uncomplicated omphalocele.)
• 29
Immediate care of the newborn with omphalocele also involves sterile wrapping of the bowel (to preserve heat
• 30
and minimize insensible fluid loss), insertion of an orogastric tube (to decompress the stomach), stabilizing the
• 31
airway (to ensure adequate ventilation), and establishing peripheral intravenous access. Small defects (< 2
• 32
cm) can be repaired with primary closure, but most defects will require a staged procedure with Silastic silo.
• 33
• 34
Educational Objective:
Know how to differentiate between the different congenital abdominal wall defects. The first step in the
• 36
management of gastroschisis is to immediately wrap the exposed bowel in sterile saline dressing and plastic
• 37
wrap in order to minimize heat and fluid losses_
• 39
• 39
Copyright © LIWorld Last updated: [9/20/2014]
• 40 Ii
• 41
• 42 End Block
Feedback
• 1
• 2 Item: 36 of 44 F' Mark U.P
▪ 3 Previous Next Lab Values, Notes Calculator.


4-
5

▪ 6 An 8-year-old African American child is brought to the office for the evaluation of a 1-day history of fever and
▪ 7 back pain. He has sickle cell disease, and has had 5 hospitalizations for similar painful crises. His laboratory
▪ 8 report shows normocytic anemia, reticulocytosis and leukocytosis. What finding is most likely to be present
▪ 9 on this patient's peripheral smear?
• 10
• 11
• 12
A. Bite cells [MC]
• 13 B. Helmet cells [12%]
• 14
▪ C. Howell Jolly bodies [64%]
• 15
• 16
ID. Heinz bodies [10%]
• 17 E Basophilic stippling [4%]
• 18
• 19
20
Explanation:
21
Sickle cell patients usually have infarcted spleens by the first 18 to 36 months of life_ Repeated
• 22
microinfarctions from clumping of sickle cells can destroy tissues, thereby leading to microvascular beds that
• 23
promote sickling. The peripheral smear characteristically reveals Howell-Jolly bodies. These bodies are
• 24
nuclear remnants of red blood cells which are generally removed by a functional spleen; therefore, their
• 25
presence suggests splenectomy or functional asplenia. Functional asplenia renders a patient susceptible to
• 26
infections, particularly from capsulated organisms such as pneumococci.
• 27
• 28
In this case, the patient has several years history of sickle cell disease and multiple, previous, similar, painful
• 29
crises. By this time, these repetitive occlusive events must have already damaged his spleen, leading to
• 30
functional asplenia. Howell Jolly bodies are most likely to be seen in his peripheral smear.
• 31
• 32 (Choices A and D) Heinz bodies are aggregates of denatured hemoglobin, and are commonly seen in
• 33 patients with hemolysis due to G6PD deficiency and thalassemia. When phagocytes extract this rigid
• 34 precipitate, they form characteristic bite cells.
35
I 36 (Choice B) Helmet cells are fragmented red blood cells. Their presence is suggestive of traumatic hemolytic
• 37 conditions such as DID, HUB and TTP.
• 30
• 39 (Choice E) Basophilic stippling are ribosomal precipitates which appear as blue granules of various sizes
• 40 dispersed throughout the cytoplasm of the red cell. These are often seen with thalassemias, as well as lead se
• 41
• 42 End Block
Feedback
• 1
• 2 Item: 36 of 44 VI/lark
▪ 3 Previous Next Lab Values. Notes Calculator.
• 4-
A. Bite cells [9%]
▪ 5
▪ 6
B. Helmet cells [12%]
▪ 7 • C. Howell Jolly bodies [64%]
▪ 8
ID. Heinz bodies [10%]
▪ 9
• 10
E Basophilic stippling [4%]
• 11
• 12
Explanation:
• 13
• 14 Sickle cell patients usually have infarcted spleens by the first 18 to 36 months of life_ Repeated
• 15 microinfarctions from clumping of sickle cells can destroy tissues, thereby leading to microvascular beds that
• 16 promote sickling. The peripheral smear characteristically reveals Howell-Jolly bodies. These bodies are
• 17 nuclear remnants of red blood cells which are generally removed by a functional spleen; therefore, their
• 18 presence suggests splenectomy or functional asplenia. Functional asplenia renders a patient susceptible to
• 19 infections, particularly from capsulated organisms such as pneumococci.
20
21 In this case, the patient has several years history of sickle cell disease and multiple, previous, similar, painful
• 22 crises. By this time, these repetitive occlusive events must have already damaged his spleen, leading to
• 23 functional asplenia. Howell Jolly bodies are most likely to be seen in his peripheral smear.
• 24
• 25 (Choices A and D) Heinz bodies are aggregates of denatured hemoglobin. and are commonly seen in
• 26 patients with hemolysis due to G6PD deficiency and thalassemia. ..I.Then phagocytes extract this rigid
• 27 precipitate, they form characteristic bite cells.
• 28
• 29 (Choice B) Helmet cells are fragmented red blood cells. Their presence is suggestive of traumatic hemolytic
• 30 conditions such as DID, HUS and TTP.
• 31
(Choice E) Basophilic stippling are ribosomal precipitates which appear as blue granules of various sizes
• 32
dispersed throughout the cytoplasm of the red cell. These are often seen with thalassemias, as well as lead
• 33
or heavy metal poisoning.
• 34
• 35
Educational Objective:
I 36
Howell-Jolly bodies are nuclear remnants of the red blood cells which are generally removed by a functional
• 37
spleen; therefore, their presence in a peripheral smear suggests functional asplenia in sickle cell patients.
• 30
• 39
Copyright © LIWorld Last updated: [9/5/2014]
• 40
• 41
• 42 End Block
Feedback
• 1
• 2 Item: 37 of 44 F' Mark -4(1
U.P
▪ 3 Previous Next Lab Values, Notes Calculator.
• 4-
5
▪ 6 A 1-day-old girl is in the newborn nursery with swollen hands and feet She was born at term by
▪ 7 uncomplicated vaginal delivery_ Her mother had mild pre-eclampsia and swelling of her hands and feet during
▪ 8 pregnancy. The pregnancy was otherwise uncomplicated. Vital signs are normal. Examination shows a girl
▪ 9 with a short webbed neck, dysplastic nails, and bilateral, non-pitting carpal and pedal edema_ Ultrasound of
• 10 the abdomen shows a horseshoe kidney. Which of the following is the most likely cause of the edema?
• 11
• 12
• 13
A. Decreased albumin synthesis [4%]
• 14 B. Decreased cardiac contractility [5%]
• 15
C. Dysgenesis of the lymphatic network [64%]
• 16
• 17
D. Increased capillary permeability [5%]
• 18 E Increased sodium retention [10%]
• 19
• Increased urinary loss of protein [12%]
20
21
• 22 Explanation:
• 23
• 24
• 25 Causes of edema
• 26
• 27
Primary mechanism Examples
• 28
• 29
• 30 • Heart failure
• 31 t Capillary hydrostatic • Glomerulonephritis, renal failure
• 32 pressure • Venous obstruction (eg, cirrhosis,
• 33 venous insufficiency)
• 34
• 35
• 36 • Protein loss (eg, nephrotic syndrome,
a 37 Capillary oncotic
protein-losing enteropathy)
• 38 pressure
(hypoalbuminemia) • Decreased albumin synthesis (eg,
• 39
• 40
cirrhosis, malnutrition) V

• 41
• 42 End Block
Feedback
• -1
• 2 Item: 37 of 44 F'Mark
▪ 3 Previous Next Lab Values. Notes Calculator.
• 4-
▪ 5
▪ 6 Causes of edema
▪ 7
▪ 8
Primary mechanism Examples
▪ 9
• 10
• -1-1 • Heart failure
• 12 t Capillary hydrostatic • Glomerulonephritis, renal failure
• 13 pressure • Venous obstruction (eg, cirrhosis,
• 14 venous insufficiency)
• 15
• 115
• 17 • Protein loss (eg, nephrotic syndrome,
• 1E
Capillary oncotic protein-losing enteropathy)
• 19
pressure
C hypoalbuminemia) • Decreased albumin synthesis (eg,
20
21
cirrhosis, malnutrition)
• 22
• 23 • Burns, trauma & sepsis
• 24 f Capillary permeability • Allergic reactions
• 25
• 26
• Other systemic inflammatory processes
• 27
• 28
• Malignancy & related treatment
• 29 Lymphatic obstruction • Hypothyroidism
• 30
• 31
• Congenital lymphedema
• 32
LISM LEWur I d, LLC
• 33
• 34
This girl's clinical presentation (eg, webbed neck, carpal and pedal edema, nail dysplasia, horseshoe
• 35
kidney) are strongly suggestive of Turner syndrome_ The edema is likely due to congenital lymphedema
• 36
from abnormal development of the lymphatic network. The dysfunctional lymphatic system causes
a 37
accumulation of protein-rich interstitial fluid in the hands, feet, and neck (webbed neck). Severe obstruction of
• 39
lymphatic vessels can result in cystic hygroma of the neck and fetal hydrops.
• 39
• 40 I +V. an. Irk; “rii-Irk ma-%
v
• 41
• 42 End Block
Feedback
• 1
• 2 Item: 37 of 44 F'Mark
▪ 3 Previous Next Lab Values. Notes Calculator.
• 4
▪ 5 • Malignancy & related treatment
▪ 6 Lymphatic obstruction • Hypothyroidism
▪ 7 • Congenital lymphederna
▪ 8
▪ 9 ©LISMLEWorld, LLC
• 10
• 11 This girl's clinical presentation (eg, webbed neck, carpal and pedal edema, nail dysplasia, horseshoe
• 12 kidney) are strongly suggestive of Turner syndrome_ The edema is likely due to congenital lymphedema
• 13 from abnormal development of the lymphatic network. The dysfunctional lymphatic system causes
• 14 accumulation of protein-rich interstitial fluid in the hands, feet, and neck (webbed neck). Severe obstruction of
• 15 lymphatic vessels can result in cystic hygroma of the neck and fetal hydrops.
• 16
• 17
Lymphedema is generally nonpitting as opposed to the pitting edema seen with liver failure (Choice A).
• 18
congestive heart failure (Choice B), or nephrotic syndrome (Choice F). In addition. hepatomegaly is seen in
• 19
right heart failure and liver failure, but not in lymphedema.
20
(Choice 0) Pre-eclampsialeclampsia can exacerbate extremity swelling in pregnancy due to increased
21
capillary permeability, decreased albumin, and renal dysfunction. These faulty mechanisms are not the
• 22
primary problem in congenital lymphedema.
• 23
• 24
(Choice E) Increased sodium and fluid retention is seen in normal pregnancy (total body volume expansion)
• 25
as well as renal pathology and heart failure.
• 26
• 27 Educational objective:
• 28 Turner syndrome should be suspected in newborn girls who have a webbed neck, horseshoe kidney, carpal
• 29 and pedal edema, and nail dysplasia. Congenital lymphedema occurs due to abnormal development of the
• 30 lymphatic system.
• 31
• 32
References:
• 33
• 34 1. The role of lymphoscintigraphy in the diagnosis of lymphedema in Turner syndrome.
• 35 2. Lymphedema: a primer on the identification and management of a chronic condition in
•36 oncologic treatment.

• 38
• 39
Copyright © LIWorld Last updated: [9/4/2014]
• 40
• 41
• 42 End Block
Feedback
• 1
• 2 Item: 37 of 44 lark
-•<1
1171
:)•*
Previous Next Lab Values Notes Calculator
• 3
4 Media Exhibit
5
6 Turner syndrome
7
8 Turner syndrome
▪ 9
• 10
• 11 Narrow, high
• 12 arched palate Low set
• 13
ears
• 14
• 15
Low
• 16
• 17
hairline __
• 18
• 19 Webbed
20
neck
21
• 22 Coarctation
• 23 Broad chest of aorta
• 24
with widely
• 25
• 26
spaced nipples
• 27
• 28
Cubitus valgus
• 29
• 30
• 31
• 32
• 33
• 34
• 35
• 36
I 37
• 1
• 2 Item: 37 of 44 P 1'2 ilark
-•<1 OP- ,
11171 4
Previous Next Lab Values. Notes Calculator.
• 3
4 Media Exhibit X
5
6 Turner syndrome
7
1
8
spaced nipples Bicuspid
▪ 9 aortic valve
• 10
• 11 Cubitus valgus
• 12
• 13
• 14
• 15
• 16
• 17
• 18
• 19
20 Streak ovaries,
21 ameriorrhea,
• 22
• 23
& infertility
• 24
• 25
• 26
• 27
• 28
• 29
• 30
• 31
Short I
• 32 stature
• 33
• 34
• 35
• 36
C)USMLEWorld, LLC V
37

Feedback End Block


• 1
• 2 Item: 38 of 44 F'Mark
-4(1

▪ 3 Previous Next Lab Values. Notes Calculator.


• 4-
5
▪ 6 A 14-year-old girl is brought to the physician for evaluation of facial puffiness, fatigue, and decreased appetite
▪ 7 for the past few days. The patient recently immigrated from China to the United States. Temperature is 36.7
▪ 8 C (98 F), blood pressure is 110/70 mm Hg, pulse is 80/min, and respirations are 18/min. Physical
▪ 9 examination shows periorbital and pretibial edema Serum laboratory results are as follows:
• 10
Creatinine 0.9 mg/dL
• 11
Albumin 2.2 mg/dL
• 12
Total bilirubin G.5 mg/dL
• 13
Aspartate aminotransferase 56 UIL
• 14
Alanine aminotransferase 64 WI_
• 15
Alkaline phosphatase 97 LEL
• 16
HBsAg Positive
• 17
HBeAg Positive
• 18
Anti-HRsAg antibodies Negative
• 19
Anti-HCV antibodies Negative
20
Anti-HIV antibodies Negative
21
• 22
Urinalysis shows 4+ proteinuria, no red blood cells, and no casts. Which of the following is the most likely
• 23
diagnosis in this patient?
• 24
• 25
• 26 A. Focal segmental glomerulosclerosis [14%]
• 27 B. Membranoproliferative glomerulonephritis [20%]
• 28
C. Membranous nephropathv [40%]
• 29
• 30 ID. Minimal change disease [25%]
• 31 E Poststreptococcal glomerulonephritis [1%]
• 32
• 33
• 34 Explanation:
▪ 35
▪ 36
- 37 Nephrotic versus nephritic syndrome
• 38
- 39
- 40 Nephrotic syndrome Nephritic syndrome V

▪ 41
▪ 42 End Block
Feedback
A'S

• 2 Item: 38 of 44 F'Mark
-4(1

▪ 3 Previous Next Lab Values. Notes Calculator.


• 4-
▪ 5
▪ 6 Nephrotic versus nephritic syndrome
▪ 7
▪ 8
▪ 9
Nephrotic syndrome Nephritic syndrome
• 10
• 1-1 • Edema • Hypertension
• 12 • Fatigue • Oliguria
• 13 Clinical
• Proteinuria • Hematuria
• 14 features
• Absence of hematuria • Proteinuria
• 15
• 16
• Hypoalbuminemia • Casts
• 17
• 19 • Minimal change disease • Hemolytic uremic syndrome
Pediatric
• 19 • Poststreptococcal
20
etiologies
glonierulonephrilis
21
• 22
• 23
• FSGS • IgA nephropathy
• 24 Adult • Membranous nephropathy • Crescentic glonnerulonephritis
• 25 etiologies • Membranoproliferative • klembranoproliferative
• 26 glornerulonephrilis glonierulonephritis
• 27
• 28 0U LEViorld, LLC

• 29
• 30 This patient's edema, hypoalbuminemia, and markedly elevated urine protein are consistent with
• 31 nephrotic syndrome (NS). Common causes of NS include minimal change disease in young children and
• 32 focal segmental glomerulosclerosis (FSGS) and membranous nephropathy in adolescents and adults.
• 33
Although membranous nephropathy is less common in children, hepatitis B infection is a significant risk
• 34
factor. Universal vaccination has dramatically reduced rates of hepatitis B virus-associated membranous
▪ 35
nephropathy (HBVMN), and unvaccinated children who have immigrated from endemic areas should be
▪ 36
screened for hepatitis B. This patient is positive for HRsAg and HBeAg and negative for anti-HRsAg antibody,
- 37
• 38
findings consistent with active hepatitis B infection. The pathogenesis of HBVMN may involve deposition of
HI3eAg or its corresponding antibody in the glomeruli.
- 39
V
- 40
▪ 41
▪ 42 End Block
Feedback
1
2 F'Mark U.P
3 ' s Next Lab Values, Notes Calculator.
4- 0-',11.i I L • memuranclus riepri.rupainy • giumeruiuriepurius
LieSUU111.14.;

5 etiologies • Membranoproliferative • klembranoproliferative


6 g lonnerulonephrilis glomerulonephritis
7
8 OUSNILEWorld,L_C

9
10 This patient's edema, hypoalbuminemia, and markedly elevated urine protein are consistent with
11 nephrotic syndrome (NS). Common causes of NS include minimal change disease in young children and
12 focal segmental glomerulosclerosis (FSGS) and membranous nephropathy in adolescents and adults.
13
14
Although membranous nephropathy is less common in children, hepatitis B infection is a significant risk
15
factor. Universal vaccination has dramatically reduced rates of hepatitis B virus-associated membranous
16
nephropathy (HBVMN), and unvaccinated children who have immigrated from endemic areas should be
17
screened for hepatitis B. This patient is positive for HRsAg and HBeAg and negative for anti-HRsAg antibody,
18
findings consistent with active hepatitis B infection. The pathogenesis of HBVMN may involve deposition of
19
HI3eAg or its corresponding antibody in the glomeruli.
20
Additional workup for this patient should include a 24-hour urine sample (protein excretion >3 gIday is
21
consistent with NS), serum C3 (typically low with HBVMN), antinuclear antibody (elevated in lupus), and renal
22
biopsy_
23
24 (Choice A) FSGS is a common cause of NS in adults and adolescents_ HIV is the most commonly
25 associated infection.
26
27 (Choice B) Membranoproliferative glomerulonephritis can cause NS or nephritic syndrome, most commonly
28 in adults, and has been associated with hepatitis B infection. However, it is significantly less common than
29 membranous nephropathy_
30
31 (Choice ID) Minimal change disease is the most common cause of NS in preadolescent children. This
32 patient is a teenager and has active hepatitis B infection, making membranous nephropathy more likely.
33
34 (Choice E) Poststreptococcal glomerulonephritis is the most common cause of nephritic syndrome in
▪ 35 children. The absence of hypertension, oliguria, hematuria, and casts makes nephritic syndrome unlikely.
▪ 36
- 37
Educational objective:
• 38 Membranous nephropathy is a common cause of nephrotic syndrome (edema, proteinuria, and
- 39
hypoalbuminemia) in adolescents and adults. Active hepatitis B infection is an important risk factor, and
- 40
vaccination reduces this risk. so
▪ 41
▪ 42
A'S
Feedback
a
End Block
• 1
• 2 Item: 38 of 44 V-Mark
▪ 3 Previous Next Lab Values. Notes Calculator.

• 4 Although membranous nephropathy is less common in children, hepatitis B infection is a significant risk
▪ 5 factor. Universal vaccination has dramatically reduced rates of hepatitis B virus-associated membranous
▪ 6 nephropathy (HBYMN), and unvaccinated children who have immigrated from endemic areas should be
▪ 7 screened for hepatitis B. This patient is positive for HBsAg and HBeAg and negative for anti-HEsAg antibody,
▪ 8 findings consistent with active hepatitis B infection. The pathogenesis of HBWIN may involve deposition of
▪ 9 HBeAg or its corresponding antibody in the glomeruli.
• 10
• 11 Additional workup for this patient should include a 24-hour urine sample (protein excretion >3 giday is
• 12 consistent with NS), serum C3 (typically low with HBVMN), antinuclear antibody (elevated in lupus), and renal
• 13 biopsy_
• 14
• 15 (Choice A) FSGS is a common cause of NS in adults and adolescents. HIV is the most commonly
• 16 associated infection.
• 17
• 18 (Choice B) Membranoproliferative glomerulonephritis can cause NS or nephritic syndrome, most commonly
• 19
in adults, and has been associated with hepatitis B infection. However, it is significantly less common than
20
membranous nephropathy_
21
(Choice ID) Minimal change disease is the most common cause of NS in preadolescent children. This
• 22
patient is a teenager and has active hepatitis B infection, making membranous nephropathy more likely.
• 23
• 24
(Choice E) Poststreptococcal glomerulonephritis is the most common cause of nephritic syndrome in
• 25
children_ The absence of hypertension, oliguria, hematuria, and casts makes nephritic syndrome unlikely.
• 26
• 27 Educational objective:
• 28 Membranous nephropathy is a common cause of nephrotic syndrome (edema, proteinuria, and
• 29 hypoalbuminemia) in adolescents and adults. Active hepatitis B infection is an important risk factor, and
• 30 vaccination reduces this risk.
• 31
• 32 References:
• 33
• 34 1. Universal hepatitis B vaccination reduces childhood hepatitis B virus-associated
35 membranous nephropathy.
36 2. Membranous nephropathy in children: clinical presentation and therapeutic approach.
37
• 38
- 39
Copyright © UWorld Last updated: [9/4/2014]
- 40 Ii
• 41
• 42 End Block
Feedback
• 1
• 2 Item: 39 of 44 F Mark
▪ 3 Previous Next Lab Values. Notes Calculator.
• 4-
▪ 5
▪ 6 A 1-week-old boy in the neonatal intensive care unit has central cyanosis. He was born at 40 weeks gestation
▪ 7 to a healthy 25-year-old woman who had an uncomplicated pregnancy and delivery. Family history is notable
▪ 8 for a maternal uncle who died of hypoplastic left heart syndrome shortly after birth. Auscultation shows a III
▪ 9 holosystolic murmur that is loudest at the left lower sternal border_ The lungs are clear bilaterally. Chest
• 10 radiograph reveals decreased pulmonary vascular markings and a normal-sized heart. Left axis deviation and
• 11 tall, peaked P waves are shown in the electrocardiogram below.
• 12
• 13
• 14
• 15
• 16
• 17
• 18
• 19
20
21
• 22
• 23
• 24
• 25
• 26
• 27


28
29
II a VI_ V2
(u
• 30
• 31
• 32
• 33
• 34

n ty,
III V6

I
se

Feedback
a
End Block
• 1
• 2 Item: 39 of 44 F'Mark
▪ 3 Previous Next Lab Values. Notes Calculator.
• 4-
▪ 5
▪ 6
▪ 7
▪ 8 11
▪ 9
• 10
• 11
• 12
• 13
• 14
• 15
• 16
• 17 III
• 18
• 19
20
21
USluill World, LLC
• 22
• 23
What is the most likely diagnosis in this patient?
• 24
• 25
• 26 A. Complete atrioventricular canal defect [6%]
• 27 B. Ebstein's anomaly [11%]
• 28
C. Tetralogy of Fallot [29%]
• 29
• 30 D. Total anomalous pulmonary venous return [13%]
• 31 E Tricuspid valve atresia [341.6]
• 32
• Truncus arteriosus [816]
• 33
• 34
Explanation:

Tricuspid atresia
III r inn 11 I
I I I I I I
se

Feedback End Block


• 1
• 2 Item: 39 of 44 V-• Mark -4(1

▪ 3 Previous Next Lab Values. Notes Calculator.


• 4 Tricuspid atresia
▪ 5
▪ 6
▪ 7 Minimal 13 waves
▪ 8
▪ 9
• 10
• 11 ▪ _ 1--

• 12 Left axis
• 13 deviation
• 14
• 15
Tall, peaked
• 16
P waves
• 17 1 Minimal R waves
• 18
• 19
20 II
21
• 22
! • I I
• 23 I 1 I

• 24
Left axis
11_11




25
26
27
28
fI
j deviation Short R waves !

• 29 iii
• 30
• 31
• 32
• 33
USMLEWorld, LLC
• 34
• 35
• 36
In the normal fetus, blood is shunted away from the lungs by the patent ductus arteriosus, and the systemic
• 37
circulation relies primarily on the right heart. As a result, newborns are born with a larger right ventricle
• 38
compared to the left ventricle, which appears as physiologic right axis deviation and R waves in the precordial
leads (Y1-V3) on electrocardiogram (ECG).
• 40
se
• 41
• 42 End Block
Feedback
A'S
• 1
• 2 Item: 39 of 44 F' Mark
▪ 3 Previous Next Lab Values. Notes Calculator.
4g IJIvILLWOrld, LLIL
• 4-
5
▪ 6
In the normal fetus, blood is shunted away from the lungs by the patent ductus arteriosus, and the systemic
▪ 7
circulation relies primarily on the right heart As a result, newborns are born with a larger right ventricle
▪ 8
compared to the left ventricle, which appears as physiologic right axis deviation and R waves in the precordial
▪ 9
leads (VI -V3) on electrocardiogram (ECG).
• 10
Tricuspid valve atresia (TVA) should be suspected in a cyanotic infant with left axis deviation and small or
• 11
absent R waves in the precordial leads. The lack of communication between the right heart chambers results
• 12
in a hypoplastic right ventricle and diminished right ventricular forces on ECG. The lack of blood flow to the
• 13
right ventricle and pulmonary outflow tract consequently results in underdevelopment of the pulmonary
• 14
valve and/or artery. Pulmonary undercirculation is the reason for decreased pulmonary markings on
• 15
chest x-ray. Associated atrial and ventricular septa/ defects are necessary for survival, allowing for mixing
• 16
of oxygenated and deoxygenated blood to provide some oxygenated blood for the systemic circulation. The
• 17
atrial septal defect permits increased blood flow to the right atrium and subsequent enlargement, resulting in
• 18
tall, peaked P waves on ECG. The ventricular septal defect manifests as a holosystolic murmur that is
• 19
loudest at the left lower sternal border.
20
21
The cause of TVA is unknown, but it can occur in any patient with congenital heart disease risk factors (eg,
• 22
congenital rubella syndrome, Down syndrome, maternal diabetes, family history of congenital heart
• 23
disease). In the absence of surgical intervention, most children will die in the first year of life. Surgical repair
• 24
improves 10-year survival rates to 80%.
• 25
• 26 The following congenital heart defects also cause cyanosis but with different ECG and chest x-ray findings.
• 27
• 28 (Choice A) Complete atrioventricular canal defect is strongly associated with Down syndrome. Chest
• 29 x-ray findings include increased pulmonary markings and cardiomegaly from excessive pulmonary blood flow
• 30 and biventricular volume overload. respectively, making this diagnosis less likely.
• 31
• 32 (Choice B) Ebstein's anomaly is associated with maternal lithium use during pregnancy. The primary
• 33 problem is displacement of a malformed tricuspid valve into the right ventricle. The droopy tricuspid valve
• 34 results in severe tricuspid regurgitation and right atrial enlargement, resulting in tall P waves and right axis
• 35 deviation on ECG. Extreme cardiomegaly from heart failure can be seen on chest x-ray.
• 36
• 37
(Choice C) Both tetralogy of Fallot and TVA can have a paucity of vascular markings on chest x-ray due to
• 38
pulmonary outflow tract obstruction. Although the "boot-shaped heart" finding from right ventricular
hypertrophy may take time to manifest in untreated tetralogy of Fallot, affected patients would have right axis
• 40
deviation on ECG, as expected in newborns. V

41
• 42 End Block
Feedback
A'S
• 1
• 2 Item: 39 of 44 V-Mark
▪ 3 Previous Next Lab Values. Notes Calculator.
• 4
▪ 5 (Choice A) Complete atrioventricular canal defect is strongly associated with Down syndrome. Chest
▪ 6 x-ray findings include increased pulmonary markings and cardiomegaly from excessive pulmonary blood flow
▪ 7 and biventricular volume overload, respectively, making this diagnosis less likely.
▪ 8
▪ 9 (Choice B) Ebstein's anomaly is associated with maternal lithium use during pregnancy. The primary
• 10 problem is displacement of a malformed tricuspid valve into the right ventricle. The droopy tricuspid valve
• 11 results in severe tricuspid regurgitation and right atrial enlargement, resulting in tall P waves and right axis
• 12 deviation on ECG. Extreme cardiomegaly from heart failure can be seen on chest x-ray.
• 13
• 14
(Choice C) Roth tetralogy of Fallot and TVA can have a paucity of vascular markings on chest x-ray due to
• 15
pulmonary outflow tract obstruction. Although the "boot-shaped heart" finding from right ventricular
• 16
hypertrophy may take time to manifest in untreated tetralogy of Fallot, affected patients would have right axis
• 17
deviation on ECG, as expected in newborns.
• 18
(Choice 0) Total anomalous pulmonary venous return is a defect in which all 4 pulmonary veins fail to
• 19
make their normal connection to the left atrium. The right atrium receives blood from both the pulmonary and
20
systemic venous systems, resulting in right atrial and ventricular enlargement ECG can show right
21
ventricular hypertrophy and right axis deviation that may be difficult to distinguish from expected findings in
• 22
newborns. Chest x-ray can show increased pulmonary markings due to pulmonary overcirculation.
• 23
• 24
(Choice F) Truncus arteriosus is strongly associated with DiGeorge syndrome. ECG can be normal in the
• 25
neonate. Chest x-ray typically shows cardiomegaly and increased pulmonary vascular markings from heart
• 26
failure and pulmonary overcirculation_
• 27
• 28 Educational objective:
• 29 Left axis deviation on neonatal electrocardiogram is never normal. Tricuspid valve atresia is a cyanotic
• 30 congenital heart defect characterized by left axis deviation on electrocardiogram and decreased pulmonary
• 31 markings on chest radiograph due to hypoplasia of the right ventricle and pulmonary outflow tract
• 32
• 33
References:
• 34
• 35 1. The pediatric electrocardiogram. Part I: Age-related interpretation.
• 36 2. Diagnosis and management of cyanotic congenital heart disease: part I.
• 37
• 38

Copyright © Morld Last updated: [10/15/2014] se


• 40
• 41
• 42 End Block
Feedback
A'S
• -1
• 2 Item: 39 of 44 F'Mark
▪ 3 Previous Next Lab Values Notes Calculator
• 4- Media Exhibit
▪ 5
▪ 6 Tricuspid atresia
▪ 7
▪ 8
Tricuspid atresia
▪ 9
• 10
• 1-1
• 12
• 13
• 14
Aorta
• 15
• 16
• 17 Pulmonary artery
• 19
• 19
20 Patent
21 foramen
• 22
ovate Pulmonary stenosis
• 23
• 24 or
• 25
• 26 atrial
• 27 septal
• 28 defect Ventricular
• 29
septal
• 30
• 31
defect
• 32
• 33
• 34
• 35
Absent
tricuspid Left
• 36
• 37 e ventricle
• 30

Feedback
a
End Block
• 1
• 2 Item: 39 of 44 11 1'2 ilark W
o 4
Previous Next Lab Values, Notes Calculator
▪ 3
4 Media Exhibit Ai
5
6 Tricuspid atresia 1
7
8
▪ 9
Pulmonary artery
• 10
• 11
• 12 Patent
• 13
foramen
• 14
• 15 ovale Pulmonary stenosis
• 16
or
• 17
• 18 atrial
• 19 septal
20
defect Ventricular
21
• 22 septal
• 23 defect
• 24
• 25
• 26
• 27 Absent
• 28 tricuspid Left
• 29 ventricle
valve
• 30
• 31
▪ 32
• 33 Hypoplastic right ventricle
▪ 34
• 35
• 36 O USMLEWorld, LLC
• 37


38
39
a
Feedback End Block
• -1
• 2 Item: 39 of 44 111
.
RI I - Mark 1
▪ 3 Previous Next Lab Values Notes Calculator
• 4- Media Exhibit
▪ 5
▪ 6 Complete atrioventricular mnal defect
▪ 7
I
▪ 8
▪ 9
• 10
• 1-1
Aorta
• 12
• 13
• 14
• 15 Pulmonary artery
• 16
• 17
• 19
• 19
Left at
20
Atrial
21
septal
• 22
• 23 defect
• 24
Right
• 25 Underdeveloped
• 26 atri LIM
at rioventricul a r
• 27
valves
• 28
• 29
• 30 Ventricular
Right
• 31 ventricle septa) defect
• 32
• 33
• 34 Left 1
35 ventricle
36
LI SiiLEW0 I L LE
37
30
• 39
• 40
• 41
• 42 End Block
4.* Feedback
• -1
• 2 Item: 39 of 44 V-. Mark
111
.
1
▪ 3 Previous Next Lab Values Notes Calculator
• 4- Media Exhibit
▪ 5
▪ 6 Ebstein's anomaly
▪ 7
▪ 8 Ebstein's anomaly
▪ 9
• 10
• -1 -1
• 12
• 13
• 14
• 15
• 16
• 17
• 19
• 19
20
21
Atrial septa!
• 22
defect
• 23
• 24
Right
• 25
atnurn
• 26
• 27
• 28 Atrialized
• 29 right ventricle
• 30
• 31
Magormed
• 32
tricuspid valve
• 33
• 34
• 35
• 36 .D 1_15.VILCWIErprkl.,11C
• 37
• 39
• 39
• 40
v


41
42
Feedback
a
End Block
• 1 A

• 2 Item: 39 of 44 r'Mark 11.P


Previous Next Lab Values, Notes
• 3
. 4 Media Exhibit
• 5
6 Tetralogy of Fallot
7
Tetralogy of Fallot
0
• 10
• 11
• 12
• 13 Overriding Aorta
• 14
• 15
• 16
• 17
Pulmonary
• 18
• 19
stenosis
20
21
. 22
• 23
• 24
• 25 Ventricular
• 26 septal
• 27 defect
• 28
• 29
• 30
• gl
• 32
• 33
. 34
• 35 /
• 36 Right ventricular hypertrophy
• 37
• 38
• 39
• -1
• 2 Item: 39 of 44 V-Mark
▪ 3 Previous Next Lab Values Notes Calculator
• 4- Media Exhibit
▪ 5
▪ 6 Tetralogy of Fallot
▪ 7
▪ 8
▪ 9
• 10
• 1-1
• 12
• 13
• 14
• 15
• 16
• 17
• 19
• 19
20
21
• 22
• 23
• 24
• 25
• 26
• 27
• 28
• 29
• 30
• 31
• 32
• 33
• 34
35
36
37 V
30
• 39
• 40
• 41
• 42 End Block
4.* Feedback
• -1
• 2 Item: 39 of 44 V-Mark
111
.

▪ 3 Previous Next Lab Values Notes Calculator


• 4- Media Exhibit
▪ 5
▪ 6 Tetralogy of Fallot
▪ 7
▪ 8
▪ 9
• 10
• 1-1
• 12
• 13
• 14
• 15
• 16
• 17
• 19
• 19
20
21
• 22
• 23
• 24
• 25
• 26
• 27
• 28
• 29
• 30
• 31
• 32
• 33
• 34
35
36
37
30
• 39
• 40
• 41
• 42 End Block
4.* Feedback
• -1
• 2 Item: 39 of 44 'Mark
11
.

▪ 3 Previous Next Lab Values Notes Calculator


• 4- Media Exhibit
▪ 5
▪ 6 Supraimrdiac total anomalous pulmonary venous retu
▪ 7
▪ 8 Supracardiac total anomalous pulmonary venous return
▪ 9
• 10
• -1 -1 Anomalous pulmonary
• 12 venous drainage into
• 13 superior vena cava
• 14
• 15
Aorta
• 16
• 17
• 19
Pulmonary artery
• 19
20
21
• 22 Pulmonary
• 23 veins
• 24
Atrial septa!
• 25
defect
• 26
• 27 Left atrium
• 28
Right atrium
• 29
• 30
• 31
• 32
• 33 Left
• 34 Right ventricle ventricle
• 35
• 36
• 37 v
• 39
• 39
• -1
• 2 Item: 39 of 44 AlM Mark
11
.

▪ 3 Previous Next Lab Values Notes Calculator


• 4- Media Exhibit 1%1
▪ 5
▪ 6 Supraimrdiac total anomalous pulmonary venous retu
▪ 7
▪ 8 An oma Ions pulmonary
▪ 9 venous drainage into
• 10 superior vena cava
• 11
• 12
Aorta
• 13
• 14
• 15
Pulmonary artery
• 16
• 17
• 19
• 19 Pulmonary
20 veins
21
Atrial septa!
• 22
defect
• 23
• 24 Left atrium
• 25
Right atrium
• 26
• 27
• 28
• 29
• 30 Left
• 31 Right ventricle ventricle
• 32
• 33
• 34
35
36 (E) U SMLEY.for Iti. LL(
• 37
• 39
• 39 J


2 Item: 39 of 44 V-Mark
-4(1 ra77

▪ 3 Previous Next Lab Values Notes Calculator


• 4- Media Exhibit
▪ 5
▪ 6 Truncus arteriosus
▪ 7
▪ 8 Truncus arteriosus
▪ 9
• 10
• 11
• 12
• 13
• 14
• 15 Aorta
• 16
• 17
• 19 Pulmonary
• 19
artery
20
21
• 22
• 23
• 24
• 25 Truncus
• 26 arteriosus
• 27
• 28
• 29
• 30 Ventricular
• 31 septa
• 32
defect
• 33
• 34
35
36
37 V
39
• 39
V
• 40
• 41
• 42 End Block
4.* Feedback
• 1 A

• 2 Item: 39 of 44 r•Mark 11
.P
Previous Next Lab Values, Notes Calculator
• 3
. 4 Pedia Exhibit
• 5
6 Truncus arteriosus
7 A
8
Aorta
0
• 10
• 11
• 12 Pulmonary
• 13 artery
• 14
• 15
• 16
• 17
• 18
Truncus
• 19
arteriosus
20
21
• 22
• 23
Ventricular
• 24
• 25
septal
• 26 defect
• 27
• 28
• 29
• 30
• gl
• 32
• 33
. 34
Ventricles
• 35
• 36 USMLEWorkl. L LC
• 37
• 38
• 39

Feedback End Block


• 1
• 2 Item: 40 of 44 V- Mark
▪ 3 Previous Next Lab Values. Notes Calculator.
• 4-
5
▪ 6 A 7-year-old boy is brought to the emergency department for a suspected fracture. His mother says that he
▪ 7 was running around the house and fell on the carpet an hour ago. The boy has been crying and complaining
▪ 8 of thigh pain since the fall_ He did not hit his head or lose consciousness. His past medical history is notable
▪ 9 for mild hearing loss and multiple fractures after seemingly minor injuries. On physical examination, he has
• 10 decreased muscle tone throughout The right thigh is markedly tender to palpation with obvious deformity.
• 11 Multiple bruises in various stages of healing are present on his extremities. His eye examination is shown
• 12 below.
• 13
• ..
• 14
• 15
• 16
• 17
• 18
• 19
20
21
• 22
• 23
• 24
• 25
• 26
• 27
• 28
• 29
• 30
• 31
• 32
• 33
• 34
• 35
• 36
• 37 Which of the following is most likely associated with this patient's underlying condition?
• 30
• 39 A. Aortic root dilatation [189C] V

41
42 End Block
Feedback
A'S
• 1 I Item: 40 of 44
• 2 F' Mark
▪ 3 Previous Next Lab Values. Notes Calculator.
• 4-
▪ 5
▪ 6 A 7-year-old boy is brought to the emergency department for a suspected fracture. His mother says that he
▪ 7 was running around the house and fell on the carpet an hour ago. The boy has been crying and complaining
▪ 8 of thigh pain since the fall. He did not hit his head or lose consciousness. His past medical history is notable
▪ 9 for mild hearing loss and multiple fractures after seemingly minor injuries. On physical examination, he has
• 10 decreased muscle tone throughout The right thigh is markedly tender to palpation with obvious deformity.
• 11 Multiple bruises in various stages of healing are present on his extremities. His eye examination is shown
• 12 below.
• 13
• 14
• 15
• 16
• 17
• 18
• 19
20
21
• 22
• 23
• 24
• 25
• 26
• 27
• 28
• 29
• 30
• 31
• 32
• 33
• 34
• 35
• 36
• 37 1.1.Thich of the following is most likely associated with this patient's underlying condition?
• 39
• 39
A. Aortic root dilatation [18%] se
r

41
42 End Block
Feedback
A'S
• 1
• 2 Item: 40 of 44 F' Mark U.P
▪ 3 Previous Next Lab Values, Notes Calculator.
• 4-
▪ 5
▪ 6
▪ 7
▪ 8
▪ 9
• 10
• 11
• 12
• 13
• 14
• 15
• 16
• 17 Which of the following is most likely associated with this patient's underlying condition?
• 18
• 19
A. Aortic root dilatation [1896]
20
21 B. Intellectual disability [3'36]
• 22 C. Opalescent teeth [689C]
• 23
ID. Retinal hemorrhages [4%]
• 24
• 25 E Subdural hematoma [3%]
• 26 E Vitamin ID deficiency [4%]
• 27
• 28
• 29 Explanation:
• 30
• 31
• 32
• 33
• 34
• 35
• 36
• 37
• 39
• 39
se
41
42
A'S
Feedback
a
End Block
• 1 I Item: 40 of 44
• 2 [I V- • Mark
▪ 3 Previous Next Lab Values. Notes Calculator.
• 4-
▪ 5 E Subdural hematoma [3%]
▪ 6 E Vitamin D deficiency [4%]
▪ 7
▪ 8
▪ 9 Explanation:
• 10
• 11
• 12
• 13
• 14
• 15
• 16
• 17
• 18
• 19
20
21
• 22
• 23
• 24
• 25
• 26
• 27
• 28
• 29
• 30
• 31
• 32
• 33
This patient has osteogenesis imperfecta (01), a connective tissue disorder most commonly inherited from an
• 34
autosomal dominant mutation of COL1A1. The disorder has a varying spectrum of severity, from mild (type
• 35
I), moderate (types 111-1X), to fatal perinatal (type II) disease. Patients with all types of Cl have osteopenia, and
• 36
the diagnosis should be suspected in any patient with blue sclerae. Other manifestations depend on the
• 37
severity of the disorder and can include recurrent fractures, easy bruisability, hypotonia, and hearing loss.
• 39
Many patients with 01 also have dentinogenesis imperfecta, an opalescent blue-gray to yellow-brown
• 39
discoloration caused by discolored dentin shining through the translucent and weak enamel. Both primary V

41
42 End Block
Feedback
A'S
• 1 I Item: 40 of 44
• 2 F' Mark
▪ 3 Previous Next Lab Values. Notes Calculator.
• 4
▪ 5
▪ 6
▪ 7
▪ 8
▪ 9
• 10
• 11
• 12
• 13
This patient has osteogenesis imperfecta (01), a connective tissue disorder most commonly inherited from an
• 14
autosomal dominant mutation of COL1A1. The disorder has a varying spectrum of severity, from mild (type
• 15
I), moderate (types 111-1X), to fatal perinatal (type II) disease. Patients with all types of Cl have osteopenia, and
• 16
the diagnosis should be suspected in any patient with blue sclerae. Other manifestations depend on the
• 17
severity of the disorder and can include recurrent fractures, easy bruisability, hypotonia, and hearing loss.
• 18
Many patients with Cl also have dentinogenesis impel-recta, an opalescent blue-gray to yellow-brown
• 19
discoloration caused by discolored dentin shining through the translucent and weak enamel. Both primary
20
and permanent teeth are affected.
21
• 22
(Choice A) Cardiac anomalies such as aortic root dilatation are not usually seen in 01. Aortic root dilatation
• 23
can be seen in other connective tissue disorders such as Marian syndrome_
• 24
• 25 (Choice B) Patients with 01 typically have normal intelligence.
• 26
• 27 (Choices D and E) Although fractures and bruises in various stages of healing should raise suspicion for
• 28 physical abuse, this diagnosis is less likely as the blue sclerae suggest an underlying collagen disorder. In
• 29 addition, inflicted bruises typically occur in central areas (eg, back, buttocks, neck, cheeks); accidental
• 30 bruises of childhood tend to occur on extremities. However, if abuse is suspected or if the child has altered
• 31 mental status or neurologic deficits, evaluation for retinal hemorrhages and subdural hematomas should be
• 32 pursued.
• 33
• 34 (Choice F) Vitamin D deficiency can lead to rickets and pathologic fractures. However, scleral abnormalities
35 typically do not occur, making this associated problem less likely.
36
37
Educational objective:
38
Osteogenesis imperfecta is associated with blue sclerae, hearing loss, recurrent fractures, and opalescent
39
teeth. Patients with osteogenesis imperfecta have normal intelligence.
se

Feedback End Block


• 1
• 2 Item: 40 of 44 it Mark
▪ 3 Previous Next Lab Values. Notes Calculator.
I I iia 1.4,41.14-.1 IL I iva 4Jaw4J4 4 iti aia 11111,4,114,4,LL4 L-4. 11, L4 4:4,1 II 14-.44.1.14,-. uaa4A4. 4411.-,411 4414,1 I I 14.0.-DL 4:4,1 I II 114.0111y 11111,1 ILL-.441 II 4.01 I I L411
• 4
▪ 5
autosomal dominant mutation of COL/M. The disorder has a varying spectrum of severity, from mild (type
▪ 6
I), moderate (types 111-1X), to fatal perinatal (type II) disease. Patients with all types of Cl have osteopenia, and
the diagnosis should be suspected in any patient with blue sclerae. Other manifestations depend on the
▪ 7
severity of the disorder and can include recurrent fractures, easy bruisability, hypotonia, and hearing loss.
▪ 8
Many patients with 01 also have dentinogenesis imperfecta, an opalescent blue-gray to yellow-brown
▪ 9
discoloration caused by discolored dentin shining through the translucent and weak enamel. Both primary
• 10
• 11
and permanent teeth are affected.
• 12
(Choice A) Cardiac anomalies such as aortic root dilatation are not usually seen in 01. Aortic root dilatation
• 13
can be seen in other connective tissue disorders such as Marfan syndrome_
• 14
• 15 (Choice 13) Patients with 01 typically have normal intelligence.
• 16
• 17 (Choices ID and E) Although fractures and bruises in various stages of healing should raise suspicion for
• 18 physical abuse, this diagnosis is less likely as the blue sclerae suggest an underlying collagen disorder. In
• 19 addition, inflicted bruises typically occur in central areas (eg, back, buttocks, neck, cheeks); accidental
20 bruises of childhood tend to occur on extremities. However, if abuse is suspected or if the child has altered
21 mental status or neurologic deficits, evaluation for retinal hemorrhages and subdural hematomas should be
• 22 pursued.
• 23
• 24 (Choice F) Vitamin ID deficiency can lead to rickets and pathologic fractures_ However, scleral abnormalities
• 25 typically do not occur, making this associated problem less likely.
• 26
• 27 Educational objective:
• 28 Osteogenesis imperfecta is associated with blue sclerae. hearing loss. recurrent fractures. and opalescent
• 29 teeth. Patients with osteogenesis imperfecta have normal intelligence.
• 30
• 31 References:
• 32
1. Osteogenesis imperfecta.
• 33
• 34 2. Osteogenesis imperfecta: diagnosis and treatment.
35 3. Pediatric dental management of a patient with osteogenesis imperfecta and
36 dentinogenesis imperfecta.
37
38
39
Copyright © LIWorld Last updated: [9/9/2014]
• 40
• 41
• 42 End Block
Feedback
• -1
• 2 Item: 40 of 44 V-Mark
EF. 11
.

▪ 3 Previous Next Lab Values Notes Calculator


• 4- Media Exhibit
▪ 5
▪ 6 Dentinogenesis imperfecta
▪ 7
▪ 8
▪ 9
• 10
• 11
• 12
• 13
• 14
• 15
• 16
• 17
• 19
• 19
20
21
• 22
• 23
• 24
• 25
• 26
• 27
• 28
• 29
• 30
• 31
• 32
• 33
• 34
35
36
37
39
39
• 40
• 41
• 42 End Block
v Feedback
• 1
• 2 Item: 41 of 44 V- Mark
▪ 3 Previous Next Lab Values. Notes Calculator.
• 4

A 6-week-old boy is brought to the physician with persistent, worsening vomiting over the past 2 weeks. The
emesis occurs with every feed, is nonbilious, and is projectile in nature_ The emesis has persisted even
▪ 8 though his mother has tried small, frequent feeds and holding the infant upright after feeds. On physical
▪ 9 examination, peristaltic waves are seen over the upper abdomen, and an olive-shaped mass is palpated in the
• 10 right upper quadrant Laboratory results are as follows:
• 11
Serum chemistry
• 12
Sodium 133 mEq/L
• 13
Potassium 2.8 mEq/L
• 14
Chloride 90 mEq/L
• 15
Bicarbonate 36 mEq/L
• 16
Blood urea nitrogen 18 mgldL
• 17
Creatinine 0_6 mg/dL
• 18
Glucose 100 mgicIL
• 19
20
Which of the following is the most appropriate next step in management of this patient?
21
• 22
• 23 A. Elective surgery later in childhood [1%]
• 24 11 Immediate surgery [19N
• 25
▪ C. Intravenous hydration and potassium replacement [80%]
• 26
• 27 ID. Oral metoclopramide [0%]
• 28 E Switch to a hydrolyzed formula [0%]
• 29
• 30
• 31 Explanation:
• 32
• 33 Laboratory derangements In pyloric sten osis
34
35
36 Vomiting
37
38
39 4
• 40
so
41
• 42 End Block
Feedback
• -1
• 2 Item: 41 of 44 F'Mark -4(1
.4111°.
▪ 3 Previous Next Lab Values. Notes Calculator.
• 4-
▪ 5 Laboratory derangements In pyloric stenosis
▪ 6
▪ 7
▪ 8 Vomiting
▪ 9
• 10
• -1 -1
• 12
• 13 L Loss of gastric HI Loss of Na I, FI20
• 14
• 15
• 16
Initiation of Hypovolennia
• 17
metabolic alkalosis,
• 19
hypochloremia
• 19
20 Renal perfusion
21
pressure
• 22
• 23
• 24
• 25 t Angiotensin II
• 26
• 27
• 28
• 29
HCO3 Aldosterone K4 secretion
• 30
• 31
reabsorption
• 32
• 33
+ I-I' secretion Hypokaleriiia
• 34
• 35
• 36 Maintenance of
• 37 metabolic alkalosis
• 33
ClUSMIEVolorld.i.LC
• 39
• 40
se
41
42 End Block
Feedback
• 1
• 2 Item: 41 of 44 I'Mark
▪ 3 Previous Next Lab Values. Notes Calculator.
• 4- T A

▪ 5 f Angiotensin II
▪ 6
▪ 7
▪ 8
▪ 9
HCCE3 Aldosterone + 10- secretion
• 10
• 11
reabsorption
• 12
• 13
+ I-I' secretion Hypokalemia
• 14
• 15
• 16 ivlaintenance of
• 17 metabolic alkalosis
• 18
USAREWoeld, LLC
• 19
20
Infantile hyperkrophic pyloric stenosis is most common in first-born boys and typically begins at age 3-5
21
weeks with projectile, nonbilious vomiting after every feed. Classic examination findings include a
• 22
palpable olive-shaped mass in the right upper quadrant A peristaltic wave moving from left to right across
• 23
the upper abdomen may also be seen immediately prior to emesis. Laboratory abnormalities include
• 24
hypochloremic, hypokalemic metabolic alkalosis. Diagnosis is made by abdominal ultrasound, which
• 25
shows an elongated, thickened pylorus.
• 26
• 27 Although pyloromyotomy is the treatment of choice, infants with signs of dehydration or laboratory
• 28 abnormalities should be admitted for intravenous rehydration and normalization of electrolytes prior to
• 29 definitive surgical treatment. Normalization of electrolytes and correction of alkalosis prior to surgery (Choice
• 30 B) have been shown to decrease the risk of postoperative apnea and improve overall outcomes.
• 31
• 32 (Choice A) Unlike reducible umbilical hernias and asymptomatic atrial septal defects, surgical repair of
• 33 pyloric stenosis should not be delayed due to risk of worsening dehydration and poor weight gain.
▪ 34
• 35 (Choice 0) Metoclopramide has no role in the treatment of pyloric stenosis as the emesis is a result of a
• 36 fixed gastric outlet obstruction rather than poor gastric motility.
• 37
• 39 (Choice E) Milk-protein allergy may present with blood-streaked stools and non-projectile emesis. Switching
• 39 the infant to a hydrolyzed formula would be appropriate if a milk-protein allergy is suspected, but it is
• 40 inappropriate in this case. so

42 End Block
Feedback
• 1
• 2 Item: 41 of 44 V-Mark
▪ 3 Previous Next Lab Values. Notes Calculator.
• 4 gi.LKIMLEWbrid, LLC
▪ 5
▪ 6 Infantile hypertrophic pyloric stenosis is most common in first-born boys and typically begins at age 3-5
▪ 7 weeks with projectile, nonbilious vomiting after every feed. Classic examination findings include a
▪ 8 palpable olive-shaped mass in the right upper quadrant A peristaltic wave moving from left to right across
▪ 9 the upper abdomen may also be seen immediately prior to emesis. Laboratory abnormalities include
• 10 hypochloremic, hypokalemic metabolic alkalosis. Diagnosis is made by abdominal ultrasound, which
• 11 shows an elongated, thickened pylorus.
• 12
• 13 Although pyloromyotomy is the treatment of choice, infants with signs of dehydration or laboratory
• 14 abnormalities should be admitted for intravenous rehydration and normalization of electrolytes prior to
• 15 definitive surgical treatment. Normalization of electrolytes and correction of alkalosis prior to surgery (Choice
• 16 B) have been shown to decrease the risk of postoperative apnea and improve overall outcomes.
• 17
• 18
(Choice A) Unlike reducible umbilical hernias and asymptomatic atrial septal defects, surgical repair of
• 19
pyloric stenosis should not be delayed due to risk of worsening dehydration and poor weight gain.
20
(Choice 0) Metoclopramide has no role in the treatment of pyloric stenosis as the emesis is a result of a
21
fixed gastric outlet obstruction rather than poor gastric motility.
• 22
• 23
(Choice El Milk-protein allergy may present with blood-streaked stools and non-projectile emesis. Switching
• 24
the infant to a hydrolyzed formula would be appropriate if a milk-protein allergy is suspected, but it is
• 25
inappropriate in this case.
• 26
• 27 Educational objective:
• 28 Infantile hypertrophic pyloric stenosis presents with projectile, nonbilious emesis and an olive-shaped
• 29 abdominal mass. Prolonged vomiting causes a hypochloremic, hypokalemic metabolic alkalosis. Treatment
• 30 consists of intravenous rehydration and normalization of electrolytes prior to pyloromyotomy to decrease the
• 31 risk of postoperative apnea.
• 32
• 33
References:
34
35 1. Electrolyte profile of pediatric patients with hypertrophic pyloric stenosis.
36 2. Pyloric stenosis in pediatric surgery: an evidence-based review.
37
38
39
Copyright © UWorld Last updated: [10/15/2014]
• 40
41
• 42 End Block
v Feedback

• 2 Item: 41 of 44 .111M Mark FF.
▪ 3 Previous Next Lab Values Notes Calculator
• 4- Media Exhibit
▪ 5
▪ 6 Hypertrophic pyloric stenosis
▪ 7
▪ 8
▪ 9
• 10
• 1-1
• 12
• 13
• 14
• 15
• 16
• 17
• 19
• 19
20
21
• 22
• 23
• 24
• 25
• 26
• 27
• 28
• 29
• 30
• 31
• 32
• 33
• 34
• 35
• 36
• 37
• 30
• 39
• 40

• 42 End Block
V Feedback
1
• 2 Item: 42 of 44 F' Mark a'
▪ 3 Previous Next Lab Values, Notes Calculator.
• 4-
▪ 5
▪ 6 A newborn boy is in the delivery room with respiratory distress. He was born vaginally at 28-weeks gestation
▪ 7 due to maternal pre-eclampsia. Examination shows a cyanotic tachypneic boy with intercostal and subcostal
▪ 8 retractions and nasal flaring. On lung auscultation, coarse breath sounds are heard bilaterally. Respiratory
▪ 9 support with continuous positive airway pressure is provided. The patient is admitted to the neonatal intensive
• 10 care unit where he is intubated due to worsening respiratory status. An orogastric tube is placed to
• 11 decompress the stomach. A chest radiograph is obtained. In addition to prematurity, which of the following
• 12 is a risk factor for the development of respiratory distress syndrome?
• 13
• 14
• 15
A. Antenatal corticosteroids [191)]
• 16 B. Intrauterine growth restriction [33%]
• 17
C. Maternal diabetes mellitus [41%]
• 18
• 19
ID. Maternal hypertension [10%]
20 E. Prolonged rupture of membranes [14%]
21
F. Vaginal delivery [196]
• 22
• 23
• 24 Explanation:
• 25
• 26
• 27
• 28
• 29
• 30
• 31
Endotracheal
• 32
• 33
tube —11111 1 '
• 34



35
36
37
ii.
• 39
• 39
• 40
se
• 41

Feedback
a
End Block
-1

2 Item: 42 of 44 V-• Mark


3 Previous Next Lab Values Notes Calculator
4- Media Exhibit
▪ 5
▪ 6 Media 1 of 1
▪ 7 A
▪ 8
▪ 9
• 10
• 11
• 12
• 13
• 14
• 15
• 16
• 17
• 19
• 19
20
21
• 22
• 23
• 24
• 25
• 26
• 27
• 28
• 29
• 30
• 31
• 32
• 33
• 34
• 35
• 36
• 37
.225m
39
• 39
3
• 40
• 41

4e# Feedback End Block



=.41.1
• 2 Item: 42 of 44 F' Mark
a VI ...

▪ 3 Previous Next Lab Values Notes Calculator


• 4-
▪ 5
Explanation:
▪ 6
▪ 7
▪ 8
▪ 9
• 10
• -1 -1
• 12
• 13
• 14
• 15
• 16
• 17
• 19
• 19
20
21
• 22
• 23
• 24
• 25
• 26
• 27
• 28
• 29
• 30
• 31
• 32
• 33
• 34
• 35
• 36
• 37
• 39
• 39
v
• 40
• 41

Feedback End Block


1
• 2 Item: 42 of 44 r Mark a'
3 Previous Next Lab Values. Notes Calculator.
• 4- urogastric ingi
5
tube
6
7
8
9
• 10
This patient most likely has respiratory distress syndrome (RIDS), a pulmonary condition caused by immature
• 11
lungs and surfactant deficiency. RIDS incidence is inversely proportional to gestational age_ The most
• 12
important risk factor for RIDS is prematurity; other factors that increase RIDS risk include male sex, perinatal
• 13
asphyxia, maternal diabetes, and cesarean section without labor. Maternal diabetes increases the incidence
• 14
of RIDS by delaying the maturation of pulmonary surfactant production. Maternal hyperglycemia causes fetal
• 15
hyperglycemia, which in turn triggers fetal hyperinsulinism. High levels of circulating insulin antagonize
• 16
cortisol and block the maturation of sphingomyelin, a vital component of surfactant.
• 17
• 18 RIDS presents with tachypnea, retractions, grunting, nasal flaring, and cyanosis at birth. Despite initial
• 19 resuscitation and respiratory support, patients can continue to decompensate and require intubation. Chest
20 radiograph shows a diffuse reticulogranular pattern ("ground-glass opacities") and air bronchograms
21 (Image). Treatment consists of antenatal prevention with corticosteroids and postnatal treatment with
• 22 exogenous surfactant and respiratory support.
• 23
• 24 (Choice A) Antenatal corticosteroids are given to women in preterm labor to enhance fetal lung maturity prior
• 25 to delivery, resulting in decreased incidence of RIDS.
• 26
• 27 Intrauterine growth restriction (Choice B). maternal hypertension (Choice D), and chronic intrauterine stress
• 28 from prolonged rupture of membranes (Choice E) decrease the risk of RDS. Intrauterine stress is thought to
• 29 stimulate early fetal lung maturity_
• 30
• 31 (Choice F) Cesarean delivery without labor is associated with increased risk of RDS. However, this baby
• 32 was born vaginally and had the benefit of stress from labor.
• 33
Educational objective:
• 34
Respiratory distress syndrome is caused by surfactant deficiency. Important risk factors include prematurity
• 35
and maternal diabetes mellitus.
• 36
• 37




38
39
40
41
r References:
1. Preconception care for diabetic women for improving maternal and infant health.

Feedback End Block


• 1
• 2 Item: 42 of 44 F Mark
▪ 3 Previous Next Lab Values. Notes Calculator.
• 4-
▪ 5
▪ 6 This patient most likely has respiratory distress syndrome (RIDS), a pulmonary condition caused by immature
▪ 7 lungs and surfactant deficiency. RIDS incidence is inversely proportional to gestational age_ The most
▪ 8 important risk factor for RIDS is prematurity; other factors that increase RIDS risk include male sex, perinatal
▪ 9 asphyxia, maternal diabetes, and cesarean section without labor. Maternal diabetes increases the incidence
• 10 of RIDS by delaying the maturation of pulmonary surfactant production. Maternal hyperglycemia causes fetal
• 11 hyperglycemia, which in turn triggers fetal hyperinsulinism. High levels of circulating insulin antagonize
• 12 cortisol and block the maturation of sphingomyelin, a vital component of surfactant.
• 13
• 14 RIDS presents with tachypnea, retractions, grunting, nasal flaring, and cyanosis at birth. Despite initial
• 15 resuscitation and respiratory support, patients can continue to decompensate and require intubation. Chest
• 16 radiograph shows a diffuse reticulogranular pattern ("ground-glass opacities") and air bronchograms
• 17 (Image}_ Treatment consists of antenatal prevention with corticosteroids and postnatal treatment with
• 18 exogenous surfactant and respiratory support.
• 19
20
(Choice A) Antenatal corticosteroids are given to women in preterm labor to enhance fetal lung maturity prior
21
to delivery, resulting in decreased incidence of RIDS_
22
Intrauterine growth restriction (Choice B), maternal hypertension (Choice D), and chronic intrauterine stress
23
from prolonged rupture of membranes (Choice E) decrease the risk of RDS. Intrauterine stress is thought to
24
stimulate early fetal lung maturity_
25
26
(Choice F) Cesarean delivery without labor is associated with increased risk of RIDS. However, this baby
27
was born vaginally and had the benefit of stress from labor.
28
29 Educational objective:
30 Respiratory distress syndrome is caused by surfactant deficiency. Important risk factors include prematurity
31 and maternal diabetes mellitus.
32
33
References:
34
35 1. Preconception care for diabetic women for improving maternal and infant health.
36 2. Care of the infant of the diabetic mother.
37
38
39
Copyright @ UWorld Last updated: [9/19/2014]
40
• 41
a 42 End Block
Feedback
• 1
• 2 Item: 43 of 44 V-• Mark
▪ 3 Previous Next Lab Values. Notes Calculator.

• 4-
5
▪ 6 An overweight 12-year-old boy presents with left knee pain that has been going on intermittently for the past
▪ 7 three months. Physical activity, especially stair climbing, exacerbates the pain. The boy's mother also points
▪ 8 out that he has been limping recently. On physical examination, his anterior left hip is moderately tender to
▪ 9 palpation, and when he is asked to stand on his left leg, the right half of his pelvis tilts downward. Which of the
• 10 following best explains this finding?
• 11
• 12
• 13
A. Tensor fascia lata weakness [1%]
• 14 B. Psoas muscle weakness [8*]
• 15
C. Quadratus lumborum weakness [6*]
• 16
• 17
D. Quadriceps muscle weakness [1%]
• 18 E. Gluteus muscle weakness [71%]
• 19
20
21 Explanation:
• 22 NORMAL TRENDELENBURG
• 23 SIGN
• 24
• 25
• 26
• 27
• 28
• 29
• 30
• 31
• 32
• 33
• 34
• 35
• 36
• 37
• 39 The physical exam finding described is the Trendelenburg sign, a drooping of the contralateral pelvis that
• 39 occurs when the patient stands on one foot. The associated Trendelenburg gait is waddling in quality, caused
• 40 by the trunk's rnEkinn to rnninensAte fnr this nelvir rirnnninn riurinn the. stAnro nhAse of nAit
• 41
• 42 End Block
Feedback
• 1
• 2 Item: 43 of 44 .11M Mark
▪ 3 Previous Next Lab Values. Notes Calculator.

• 4-
5
▪ 6

▪ 8
▪ 9
• 10
• 11
• 12
• 13
• 14
• 15
• 16 1-1
• 17
• 18 The physical exam finding described is the Trendelenburg sign, a drooping of the contralateral pelvis that
• 19 occurs when the patient stands on one foot The associated Trendelenburg gait is waddling in quality, caused
20 by the trunk's rocking to compensate for this pelvic drooping during the stance phase of gait.
21
• 22 Normally, the gluteus medius and gluteus minimus muscles, which are both innervated by the superior gluteal
• 23 nerve, function to abduct the thigh at the hip when standing on one foot or during normal ambulation when the
• 24 body's weight rests on only one foot Weakness of these muscles, as can occur in neuromuscular disease,
• 25 impingement of or trauma to the superior gluteal nerve, or inflammatory myopathies, results in a positive
• 26 Trendelenburg sign and g ait
1

• 27
(Choice Al The tensor fascia lata spans from the iliac crest to the fascia lata_ It is a small muscle that
• 28
assists in hip abduction and maintenance of knee extension.
• 29
• 30
(Choice B) The psoas major muscle spans from the transverse processes of the lumbar vertebrae to the
• 31
lesser trochanter of the femur. It functions to flex and laterally rotate the thigh.
• 32
• 33 (Choice C) The quadratus lumborum spans from the iliac crest to the 12th rib and the transverse processes
• 34 of the first four lumbar vertebrae. It functions in rib cage fixation and in lateral flexion of the trunk.
• 35
• 36 (Choice 0) The quadriceps femoris muscles all function in leg extension at the knee_ The rectus femoris
• 37 also functions as a hip flexor.
L 39 I
Educational objective:
nrnnninn of the rnntmH-eml heminelvis helnw its nnrmA hnn7nntA level eliirinn n-innnneriA stanre rnntitiiteq V
• 40 I
• 41
• 42 End Block
Feedback
• 1
• 2 Item: 43 of 44 .11M Mark
▪ 3 Previous Next Lab Values. Notes Calculator.
• 4-
5
▪ 6
▪ 7
▪ 8
▪ 9
• 10
• 11
• 12
• 13
The physical exam finding described is the Trendelenburg sign, a drooping of the contralateral pelvis that
• 14
occurs when the patient stands on one foot. The associated Trendelenburg gait is waddling in quality, caused
• 15
by the trunk's rocking to compensate for this pelvic drooping during the stance phase of gait
• 16
• 17 Normally, the gluteus medius and gluteus minimus muscles, which are both innervated by the superior gluteal
• 18 nerve, function to abduct the thigh at the hip when standing on one foot or during normal ambulation when the
• 19 body's weight rests on only one foot Weakness of these muscles, as can occur in neuromuscular disease,
20 impingement of or trauma to the superior gluteal nerve, or inflammatory myopathies, results in a positive
21 Trendelenburg sign and gait
• 22
• 23 (Choice A) The tensor fascia lata spans from the iliac crest to the fascia lata_ It is a small muscle that
• 24 assists in hip abduction and maintenance of knee extension.
• 25
• 26 (Choice B) The psoas major muscle spans from the transverse processes of the lumbar vertebrae to the
• 27 lesser trochanter of the femur. It functions to flex and laterally rotate the thigh.
• 28
• 29 (Choice C) The quadratus lumborum spans from the iliac crest to the 12th rib and the transverse processes
• 30 of the first four lumbar vertebrae. It functions in rib cage fixation and in lateral flexion of the trunk.
• 31
(Choice CI) The quadriceps femoris muscles all function in leg extension at the knee. The rectus femoris
• 32
also functions as a hip flexor.
• 33
• 34
Educational objective:
• 35
Drooping of the contralateral hemipelvis below its normal horizontal level during monopedal stance constitutes
• 36
a positive Trendelenburg sign. It is caused by weakness or paralysis of the gluteus medius and minimus
• 37
muscles, which are innervated by the superior gluteal nerve.
• 39
• 39
• 40
Copyright © UWorld Last updated: [9/19/2014]
1
• 41
• 42 End Block
Feedback
• 1
• 2 Item: 44 of 44 V- Mark
▪ 3 Previous Next Lab Values. Notes Calculator.
• 4-
▪ 5
▪ 6 A 14—month-old boy is admitted to the hospital for treatment of pneumonia. He has a fever, cough, and
▪ 7 increased work of breathing. Sick contacts include multiple children in day care who have had a 'cold." His
▪ 8 mother is concerned that he is "frequently ill" as he had 3 episodes of bronchiolitis during infancy and
▪ 9 pneumococcal pneumonia 2 months ago that required intubation and hospitalization in the intensive care
• 10 unit He also had tympanostomy tubes placed recently for recurrent otitis media. He takes no medications
• 11 and his vaccinations are up to date. Weight is 7 kg (15.4 Ib, <3rd percentile). Temperature is 39.5 C (103.2
• 12 F), blood pressure is 90/50 mm Hg, pulse is 128/min, and respirations are 38/min. On examination, the
• 13 patient appears listless and is in mild respiratory distress_ Both ear canals contain purulent
• 14 drainage. Crackles are heard in the left lung base. Laboratory results are shown below.
• 15
Complete blood count
• 16
Hemoglobin 12f gJdL
• 17
Platelets 260,000/pL
• 18
Leukocytes 9,000/pL
• 19
Neutrophils 40%
20
Lymphocytes 50%
21
• 22
Immunoglobulins
• 23
IgG 250 mgidL (normal = 700-1500 mg.iciL)
• 24
IgA 24 mg/dL (normal = 60-400 mgfdL)
• 25
IgM 450 mg/dL (normal = 60-300 mg/I:IL)
• 26
• 27
CD4ICD8 ratio 2.2 (normal = 1-4)
• 28
HIV-1 antibody negative
• 29
• 30
Which of the following is the most likely diagnosis?
• 31
• 32
• 33 A. Bruton agammaglobulinemia [10%]
• 34 B. Common variable immunodeficiency [9%]
• 35
C. Hyper-IgM syndrome [73%]
• 36
• 37 ID. IgA deficiency [6%]
• 39 E Munchausen syndrome by proxy [0%]
• 39
• Selective IgG subclass deficiencies [1%] V
• 40
• 41
• 42 End Block
Feedback
-1

2 Item: 44 of 44 Y- Mark
3 Previous Next Lab Values, Notes Calculator.
4-
5 Which of the following is the most likely diagnosis?
6
7 A. Bruton agammaglobulinemia [10'36]
8
B. Common variable immunodeficiency [9%]
9
10 • C. Hyper-IgM syndrome [73%]
-1 -1 D. IgA deficiency [6%]
12
E Munchausen syndrome by proxy [0%]
13
14 • Selective IgG subclass deficiencies [M]
15 G. Transient hypogammaglobulinemia of infancy [1136]
16
17
19 Explanation:
19
20
21 Primary humoral deficiencies
• 22
• 23
• 24
Diagnosis Clinical features Laboratory findings
• 25
• 26 X-linked 4, or absent B-cells
• 27 agammaglobulinemia 4,Iminnunoglobulins
• 28
• 29 Common variable Normal B cells
• 30 immunodeficiency 4,1minnunoglobulins
• 31
Recurrent &icor severe
• 32 Normal B cells
• 33
IgA deficiency siriopulmonary infections
4,IgA
• 34 with viruses and
• 35 encapsulated bacteria Normal B cells
• 36
Hyper-IgM syndrome 4,IgG & IgA
• 37
Igrvl
• 39
• 39
IgG subclass Normal B cells se
• 40


41
42
Feedback
a
End Block
Item: 44 of 44 r.Mark
Previous , Next . Lab Values. Notes Calculator.
A

6 Primary humoral deficiencies


7
8
• 9 Diagnosis Clinical features Laboratory findings
• 10
• 11 X-linked 4, or absent B-cells
• 12
agammaglobulinemia 4,1rninnunoglobulins
• 13
• 14
Common variable Normal B cells
• 15
immunodeficiency 4'Irninnunoglobulins
• 15
• 17 Recurrent &for severe
Normal B cells
• 18 IgA deficiency sinopuirnonary infections 4,1gA
• 19 with viruses and
20 encapsulated bacteria Normal B cells
21
Hyper-10A syndrome 4,IgG & IgA
• 22
• 23 t Igtvl
• 24
IgG subclass Normal B cells
deficiency 4, Ig

• 25
Fl. 27 LISM LEWar I d, LLC
• 28
• 29 Mild respiratory infections are common in childhood and can occur up to 10 times per year. especially with
• 30 day care exposure. This child has severe and recurrent sinopulmonary infections (intubation. need for
.31 early tympanostomy tubes. poor growth) that are concerning for humoral immunodeficiency. Complete
• 32 blood count with differential, lymphocyte T and E cell subsets, and serum immunoglobulins (Ig) G. A. and M
.33 are important screening tests. HIV infection should also be excluded.
• 34
• 35 The low serum IgA and IgG in combination with the markedly elevated Ignil in this case is concerning for
• 36 hyper-IgM syndrome (HIM). which is caused by an X-linked genetic defect in the CD40 ligand. The CD40
• ligand is present on T-cells: it binds to CD40 expressed on B-cells, which induces a change in B-cell
• 38 production of IgM to other immunoglobulins (class switching). The absence of the CD40 ligand prevents
• 39 class switching. leading to elevated IgM levels and a deficiency of all other immunoglobulin types. CD4G
• 40 lioanrl riafirionru alcn inkihitQ nlacma roll forma tion in/kirk rnntrihi itoc to nnnr raQnnnQa to infortinn and se
• 41
• 42
Feedback, End Block .
• 1
• 2 Item: 44 of 44 V-Mark
▪ 3 Previous Next Lab Values. Notes Calculator.
• 4-
5 IgG subclass Normal B cells
deficiency 4,IgG
▪ 7 OUSNILEWeirld, LLC
▪ 8
▪ 9 Mild respiratory infections are common in childhood and can occur up to 10 times per year, especially with
• 10 day care exposure. This child has severe and recurrent sinopulmonary infections (intubation, need for
• 11 early tympanostomy tubes, poor growth) that are concerning for humoral immunodeficiency. Complete
• 12 blood count with differential, lymphocyte T and 13 cell subsets, and serum immunoglobulins (Ig) G, A, and M
• 13 are important screening tests. HIV infection should also be excluded.
• 14
• 15 The low serum IgA and IgG in combination with the markedly elevated 101 in this case is concerning for
• 16 hyper-IgM syndrome (HIM), which is caused by an X-linked genetic defect in the CD40 ligand. The CD40
• 17 ligand is present on T-cells; it binds to CD4G expressed on B-cells, which induces a change in B-cell
• 18 production of IgM to other immunoglobulins (class switching). The absence of the CD40 ligand prevents
• 19 class switching, leading to elevated IgM levels and a deficiency of all other immunoglobulin types. CD4D
20 ligand deficiency also inhibits plasma cell formation, which contributes to poor response to infection and
21 immunization.
• 22
• 23 Patients with HIM have recurrent sinopulmonary infections (eg, acute otitis media, pneumonia, sinusitis) with
• 24 encapsulated bacteria. They also tend to have more frequent viral infections and increased risk of
• 25 opportunistic infections, such as PrIeurnocystis proveci pneumonia. Growth impairment can result from high
• 26 energy expenditure and poor intake during illness. Treatment includes antibiotic prophylaxis and interval
• 27 administration of intravenous immunoglobulin.
• 28
• 29 (Choice A) Bruton agammaglobulinemia, also known as X-linked agammaglobulinemia presents in a similar
• 30 manner to the other humoral immunodeficiency syndromes_ It is characterized by low Ig, IgM, and IgA and
• 31 low or absent 13-lymphocytes_
• 32
(Choice B) Common variable immunodeficiency is characterized by low levels of Ig, IgM, and IgA in the
• 33
setting of a normal B-lymphocyte count
• 34
• 35
(Choice 13) Selective IgA deficiency is a relatively common immunodeficiency. Most patients are
▪ 36
asymptomatic but some have recurrent respiratory, gastrointestinal, and urogenital infections. Serum IgA is
• 37
low while IgM is normal.
▪ 38
• 39 (Choice E) Munchausen syndrome by proxy is a form of child abuse during which a caregiver induces illness
• 40
se
• 41
• 42 End Block
Feedback
• 1
• 2 Item: 44 of 44 V-Mark
▪ 3 Previous Next Lab Values. Notes Calculator.
• 4
▪ 5 (Choice A) Bruton agammaglobulinemia, also known as X-linked agammaglobulinemia presents in a similar
▪ 6 manner to the other humoral immunodeficiency syndromes_ It is characterized by low IgG, IV, and IgA and
▪ 7 low or absent B-lymphocytes_
▪ 8
(Choice B) Common variable immunodeficiency is characterized by low levels of IgG, IgM, and IgA in the
▪ 9
setting of a normal B-lymphocyte count
• 10
• 11
(Choice ID) Selective IgA deficiency is a relatively common immunodeficiency. Most patients are
• 12
asymptomatic but some have recurrent respiratory, gastrointestinal, and urogenital infections. Serum IgA is
• 13
low while IV is normal.
• 14
• 15 (Choice El Munchausen syndrome by proxy is a form of child abuse during which a caregiver induces illness
• 16 (eg, putting contaminated material in a child's ears) or fabricates a history of recurrent symptoms_ These
• 17 children may undergo repeated procedures, medication courses, and/or hospitalizations with no objective
• 18 abnormalities detected_
• 19
20 (Choice F) Patients with selective IgG subclass deficiencies usually have recurrent infections, low total IgG,
21 and normal IgM_
• 22
• 23 (Choice G) The normal physiologic nadir of maternally-derived antibody is 3-6 months of age in term infants.
• 24 Low serum IgG levels after age 6 months is termed transient hypogammaglobulinemia of infancy as it usually
• 25 resolves by age 12 months. B and T lymphocyte counts and IgA and IgM levels are normal_
• 26
• 27 Educational objective:
• 28 Primary humoral immune deficiency syndromes present with recurrent or severe sinopulmonary infections.
• 29 Hyper-IgM syndrome is caused by a defect in the CND ligand and is characterized by high Igl••rl levels, low or
• 30 absent IgG and IgA, and normal lymphocyte populations_
• 31
• 32 References:
• 33
1. Back to basics: primary immune deficiencies: windows into the immune system
• 34
• 35 2. Update an the hyperimmunaglobulin Pi syndromes
• 36 3. History of respiratory infections in the first 12 yr among children from a birth cohort.
• 37
• 39
• 39
Copyright @ Morld Last updated: [10/29/2014]
• 40
• 41
• 42 End Block
Feedback

Anda mungkin juga menyukai